colecção de problemas de electromagnetismo e...

128
Colecção de Problemas de Electromagnetismo e Óptica Introdução teórica, problemas resolvidos e problemas propostos com soluções Mestrado em Engenharia Electrotécnica e Computadores (MEEC) versão: 1 o semestre de 2013-14 Fernando Carvalho Barão, Luís Filipe Mendes Departamento de Física do Instituto Superior Técnico Versão: Dezembro/2013

Upload: duongdiep

Post on 31-Aug-2018

219 views

Category:

Documents


0 download

TRANSCRIPT

Page 1: Colecção de Problemas de Electromagnetismo e Ópticafcomp.ist.utl.pt/EO.web/Book.29_01_2014.pdf · Preâmbulo Esta colectânea de problemas resulta da experiência dos autores na

Colecção de Problemas de

Electromagnetismo e ÓpticaIntrodução teórica, problemas resolvidos e problemas propostos com soluções

Mestrado em Engenharia Electrotécnica e Computadores (MEEC)versão: 1o semestre de 2013-14

Fernando Carvalho Barão, Luís Filipe MendesDepartamento de Física do Instituto Superior Técnico

Versão: Dezembro/2013

Page 2: Colecção de Problemas de Electromagnetismo e Ópticafcomp.ist.utl.pt/EO.web/Book.29_01_2014.pdf · Preâmbulo Esta colectânea de problemas resulta da experiência dos autores na
Page 3: Colecção de Problemas de Electromagnetismo e Ópticafcomp.ist.utl.pt/EO.web/Book.29_01_2014.pdf · Preâmbulo Esta colectânea de problemas resulta da experiência dos autores na

Preâmbulo

Esta colectânea de problemas resulta da experiência dos autores na docência da disciplina de Electro-magnetismo e Óptica no mestrado integrado de Electrotecnia e Computadores, no Instituto SuperiorTécnico ao longo de vários anos. O conjunto de problemas aqui apresentado, beneficiou do traba-lho de inúmeros colegas, quer pela contribuição directa para a disciplina em causa, quer pela viade problemas herdados. A todos devotamos os nossos agradecimentos. Adicionalmente, destaca-mos também a contribuição dos alunos para esta colectânea de problemas, quer pelo apontar deincorrecções ou incongruências quer pelo estímulo que nos foi dado para a sua execução.

A colectânea de problemas encontra-se dividida em vários capítulos. Nestes, faz-se uma breveintrodução teórica que pretende resumir a matéria necessária à resolução dos problemas propostos.Segue-se um conjunto de exercícios resolvidos que pretende ser um guia da metodologia a seguir naresolução dos exercícios propostos. Por último, todos os problemas apresentados possuem soluções.

Este documento encontra-se ainda a ser trabalhado, nomeadamente no que respeita às intro-duções teóricas de alguns dos capítulos. No entanto e apesar de provisório, achámos útil a suadisponibilização aos alunos. Resta o desejo de que ele cumpra os objectivos de permitir uma melhorassimilação dos conteúdos ensinados na disciplina de Electromagnetismo e Óptica.

Lisboa, 16 de Dezembro de 2009

Fernando Carvalho BarãoLuís Filipe Mendes

Page 4: Colecção de Problemas de Electromagnetismo e Ópticafcomp.ist.utl.pt/EO.web/Book.29_01_2014.pdf · Preâmbulo Esta colectânea de problemas resulta da experiência dos autores na
Page 5: Colecção de Problemas de Electromagnetismo e Ópticafcomp.ist.utl.pt/EO.web/Book.29_01_2014.pdf · Preâmbulo Esta colectânea de problemas resulta da experiência dos autores na

Índice

Constantes Físicas 1

Formulário Matemático 3

Formulário de Electromagnetismo e Óptica 5

1 Electrostática 71.1 Introdução . . . . . . . . . . . . . . . . . . . . . . . . . . . . . . . . . . . 71.2 Exercícios Resolvidos . . . . . . . . . . . . . . . . . . . . . . . . . . . . . . 18

1.2.1 Distribuição discreta de carga eléctrica . . . . . . . . . . . . . . . . . . 181.2.2 Distribuição contínua de carga eléctrica num fio . . . . . . . . . . . . . 211.2.3 Distribuição contínua de carga eléctrica num plano infinito . . . . . . . . 241.2.4 Condutores em equilíbrio electrostático . . . . . . . . . . . . . . . . . . 251.2.5 Campo eléctrico num dieléctrico . . . . . . . . . . . . . . . . . . . . . 271.2.6 Condensador plano . . . . . . . . . . . . . . . . . . . . . . . . . . . 291.2.7 Associação de condensadores em série . . . . . . . . . . . . . . . . . . 311.2.8 Condensador cilíndrico . . . . . . . . . . . . . . . . . . . . . . . . . . 33

1.3 Exercícios Propostos . . . . . . . . . . . . . . . . . . . . . . . . . . . . . . 361.4 Soluções dos exercícios propostos . . . . . . . . . . . . . . . . . . . . . . . . 45

2 Corrente eléctrica 492.1 Introdução . . . . . . . . . . . . . . . . . . . . . . . . . . . . . . . . . . . 492.2 Exercícios Resolvidos . . . . . . . . . . . . . . . . . . . . . . . . . . . . . . 54

2.2.1 Velocidade de deriva dos electrões . . . . . . . . . . . . . . . . . . . . 542.2.2 Resistência de um condutor cilíndrico . . . . . . . . . . . . . . . . . . 552.2.3 Adição de resistências em série . . . . . . . . . . . . . . . . . . . . . . 562.2.4 Adição de resistências em paralelo . . . . . . . . . . . . . . . . . . . . 582.2.5 Corrente eléctrica numa coroa esférica . . . . . . . . . . . . . . . . . . 59

2.3 Exercícios Propostos . . . . . . . . . . . . . . . . . . . . . . . . . . . . . . 602.4 Soluções . . . . . . . . . . . . . . . . . . . . . . . . . . . . . . . . . . . . 62

3 Magnetostatica 633.1 Introdução . . . . . . . . . . . . . . . . . . . . . . . . . . . . . . . . . . . 633.2 Exercícios Propostos . . . . . . . . . . . . . . . . . . . . . . . . . . . . . . 703.3 Resoluções . . . . . . . . . . . . . . . . . . . . . . . . . . . . . . . . . . . 743.4 Soluções . . . . . . . . . . . . . . . . . . . . . . . . . . . . . . . . . . . . 77

Page 6: Colecção de Problemas de Electromagnetismo e Ópticafcomp.ist.utl.pt/EO.web/Book.29_01_2014.pdf · Preâmbulo Esta colectânea de problemas resulta da experiência dos autores na

4 Partículas e campos 794.1 Introdução . . . . . . . . . . . . . . . . . . . . . . . . . . . . . . . . . . . 794.2 Exercícios Resolvidos . . . . . . . . . . . . . . . . . . . . . . . . . . . . . . 82

4.2.1 Partículas aceleradas . . . . . . . . . . . . . . . . . . . . . . . . . . 824.2.2 Condutor em movimento . . . . . . . . . . . . . . . . . . . . . . . . 834.2.3 Efeito de Hall . . . . . . . . . . . . . . . . . . . . . . . . . . . . . . 84

4.3 Exercícios Propostos . . . . . . . . . . . . . . . . . . . . . . . . . . . . . . 854.4 Soluções . . . . . . . . . . . . . . . . . . . . . . . . . . . . . . . . . . . . 88

5 Campo Magnético Variável 895.1 Exercícios Propostos . . . . . . . . . . . . . . . . . . . . . . . . . . . . . . 905.2 Resoluções . . . . . . . . . . . . . . . . . . . . . . . . . . . . . . . . . . . 955.3 Soluções . . . . . . . . . . . . . . . . . . . . . . . . . . . . . . . . . . . . 99

6 Circuitos eléctricos 1016.1 Exercícios Propostos . . . . . . . . . . . . . . . . . . . . . . . . . . . . . . 1026.2 Resoluções . . . . . . . . . . . . . . . . . . . . . . . . . . . . . . . . . . . 1056.3 Soluções . . . . . . . . . . . . . . . . . . . . . . . . . . . . . . . . . . . . 107

7 Equações de Maxwell e ondas electromagnéticas 1097.1 Exercícios Propostos . . . . . . . . . . . . . . . . . . . . . . . . . . . . . . 1107.2 Resoluções . . . . . . . . . . . . . . . . . . . . . . . . . . . . . . . . . . . 1137.3 Soluções . . . . . . . . . . . . . . . . . . . . . . . . . . . . . . . . . . . . 115

8 Óptica 1178.1 Introdução . . . . . . . . . . . . . . . . . . . . . . . . . . . . . . . . . . . 1178.2 Exercícios Propostos . . . . . . . . . . . . . . . . . . . . . . . . . . . . . . 1188.3 Exercícios Resolvidos . . . . . . . . . . . . . . . . . . . . . . . . . . . . . . 1218.4 Soluções . . . . . . . . . . . . . . . . . . . . . . . . . . . . . . . . . . . . 122

Page 7: Colecção de Problemas de Electromagnetismo e Ópticafcomp.ist.utl.pt/EO.web/Book.29_01_2014.pdf · Preâmbulo Esta colectânea de problemas resulta da experiência dos autores na

Constantes Físicas

massa do electrão me 9, 10 × 10−31 kg

massa do protão mp 1, 67 × 10−27 kg

carga elementar e 1, 6 × 10−19 C

permitividade eléctrica do vácuo 14πε0

9 × 109 N.m2.C−2

permitividade magnética do vácuo µ0

4π10

−7 N.A−2

constante de Planck h 6, 6 × 10−34 J.s

número de Avogadro NA 6, 022 × 1023

velocidade da luz no vácuo c 3 × 108 m.s−1

Page 8: Colecção de Problemas de Electromagnetismo e Ópticafcomp.ist.utl.pt/EO.web/Book.29_01_2014.pdf · Preâmbulo Esta colectânea de problemas resulta da experiência dos autores na

2

Page 9: Colecção de Problemas de Electromagnetismo e Ópticafcomp.ist.utl.pt/EO.web/Book.29_01_2014.pdf · Preâmbulo Esta colectânea de problemas resulta da experiência dos autores na

Formulário matemático

Formulário de Matemática para Electromagnetismo e Óptica, MEEC (2008-2013)

Algumas Primitivas

dx

(x2 + b)3/2=

1

b

x√

x2 + b

xdx

(x2 + b)3/2= − 1

x2 + b∫

xdx√

x2 + b=

x2 + b

dx√

x2 + b= ln

(

x +√

x2 + b)

dx

x(x + a)=

1

aln(

x

x + a)

Para o cálculo analítico de integrais pode ser consultado o endereço web: http://integrals.wolfram.com

Coordenadas cartesianas (x, y, z)

d~ℓ = dx ~ux + dy ~uy + dz ~uz

dS = dx dy

dV = dx dy dz

~∇F =

(

∂F

∂x,∂F

∂y,∂F

∂z

)

~∇ · ~A =∂Ax

∂x+

∂Ay

∂y+

∂Az

∂z

~∇ × ~A =

(

∂x,

∂y,

∂z

)

× (Ax, Ay, Az)

Coordenadas polares (r, θ)

d~ℓ = dr ~ur + r dθ ~uθ

dS = r dr dθ

Coordenadas cilíndricas (r, θ, z)

d~ℓ = dr ~ur + r dθ ~uθ + dz ~uz

dV = r dr dθ dz

~∇F =

(

∂F

∂r,1

r

∂F

∂θ,∂F

∂z

)

~∇ · ~A =1

r

∂(r Ar)

∂r+

1

r

∂Aθ

∂θ+

∂Az

∂z

~∇ × ~A =

(

1

r

∂Az

∂θ− ∂Aθ

∂z

)

~ur +

(

∂Ar

∂z− ∂Az

∂r

)

~uθ +

(

1

r

∂(r Aθ)

∂r− 1

r

∂Ar

∂θ

)

~uz

Coordenadas esféricas (r, θ, φ)

d~ℓ = dr ~ur + r dθ ~uθ + r senθ dφ ~uφ

dV = r2 dr senθ dθ dφ

~∇F =

(

∂F

∂r,1

r

∂F

∂θ,

1

rsenθ

∂F

∂φ

)

~∇ · ~A =1

r2

∂r

(

r2Ar)

+1

rsenθ

∂θ(senθAθ) +

1

rsenθ

∂φ

(

)

~∇ × ~A =

[

1

rsenθ

∂(senθAφ)

∂θ− ∂(senθAθ)

∂φ

]

~ur +1

r

[

1

senθ

∂Ar

∂φ− ∂(rAφ)

∂r

]

~uθ +1

r

[

∂(rAθ)

∂r− ∂Ar

∂θ

]

~uφ

Teorema da Divergência∫

V

~∇ · ~A dV =

S

~A · ~n dS

3

Page 10: Colecção de Problemas de Electromagnetismo e Ópticafcomp.ist.utl.pt/EO.web/Book.29_01_2014.pdf · Preâmbulo Esta colectânea de problemas resulta da experiência dos autores na

Teorema da Stokes

S

~∇ × ~A · d~S =

Γ

~A · d~ℓ

Identidades vectoriais

~∇ · ( ~A × ~B) = ~B · (~∇ × ~A) − ~A · (~∇ × ~B)

~∇ · (~∇ × ~A) = 0

~∇ × (~∇ × ~A) = ~∇(~∇ · ~A) − ∇2 ~A

F.Barao, L.F.Mendes Dep. de Física, IST

4

Page 11: Colecção de Problemas de Electromagnetismo e Ópticafcomp.ist.utl.pt/EO.web/Book.29_01_2014.pdf · Preâmbulo Esta colectânea de problemas resulta da experiência dos autores na

Formulário de Electromagnetismo eÓptica

Formulário de Electromagnetismo e Óptica, MEEC (2008-2013)

5

Page 12: Colecção de Problemas de Electromagnetismo e Ópticafcomp.ist.utl.pt/EO.web/Book.29_01_2014.pdf · Preâmbulo Esta colectânea de problemas resulta da experiência dos autores na

Electrostática

• ~E =1

4πε0

q

r2~ur

• 1

4πε0= 9 × 109N.m2.C−2

•∮

Γ

~E · d~ℓ = 0

∇ × ~E = 0

•∮

S

~D · ~n dS =

Vρliv dv

~∇ · ~D = ρliv

•∮

S

~P · ~n dS = −∫

Vρpoldv

ρpol = −~∇ · ~P

σpol = ~P · ~next

• φP =

∫ Ref

P

~E · d~ℓ

~E = −~∇φ

• ~D = ~P + ε0 ~E

~D = ε0(1 + χE)~E = ε ~E

• Q = CV

• UE =

[

1

2

]

i

qi φi

• uE =1

2εE2

UE =

VuEdv

• ~Fs = ±dUE

ds~us

Corrente eléctrica estacionária

• ~J = Nq~v

• ~J = σc ~E

• I =

S

~J · ~n dS

• p = ~J · ~E

•∮

S

~J · ~n dS = − d

dt

Vρdv

~∇ · ~J = −dρ

dt

Magnetostática

• ~B =

Γ

µ0

Id~ℓ × ~ur

r2

µ0

4π= 10−7H/m

• d~F = Id~ℓ × ~B

•∮

S

~B · ~n dS = 0

~∇ · ~B = 0

•∮

Γ

~H · d~ℓ =

S

~J · ~n dS

~∇ × ~H = ~J

• ~B = µ0( ~M + ~H)

~B = µ0(1 + χm) ~H = µ ~H

•∮

Γ

~M · d~ℓ =

S

~JM · ~n dS

~JM = ~∇ × ~M

~J′

M = ~M × ~next

Interacção de partículas e campos

• ~F = q(

~E + ~v × ~B)

Campos variáveis e indução

•∮

Γ

~E · d~ℓ = − d

dt

S

~B · ~n dS

~∇ × ~E = −∂ ~B

∂t

• Φi = LiIi + MijIj

• UM =

[

1

2

]

i

ΦiIi

• uM =1

2

B2

µ

UM =

VuMdv

• ~Fs = ±dUM

ds~us

•∮

Γ

~H · d~ℓ =

S

~J · ~n dS +d

dt

S

~D · ~n dS

~∇ × ~H = ~J +∂ ~D

∂tOndas electromagnéticas

• ~S = ~E × ~H

• ~n =~κ

κ=

~E

~B

B

• E

B= v

• v =1

√εµ

• u = uE + uM

• I =⟨

~S · ~n⟩

Óptica

• n1senθ1 = n2senθ2

• tgθB =n2

n1

interferência entre fendas

• dsenθmax = mλ

• dsenθmin = mλ +λ

m′(m

′ ≤ N e par)

difracção

• asenθmin = mλ

F.Barao, L.F.Mendes Dep. de Física, IST

6

Page 13: Colecção de Problemas de Electromagnetismo e Ópticafcomp.ist.utl.pt/EO.web/Book.29_01_2014.pdf · Preâmbulo Esta colectânea de problemas resulta da experiência dos autores na

Capítulo 1

Capítulo 1

Electrostática

1.1 Introdução

A interacção electromagnética, uma das quatro interacções fundamentais da Na-tureza, está associada à presença de cargas eléctricas. A existência de interacçõestraduz-se no aparecimento de forças a actuar os corpos. As quatro forças são a forçagravítica, que rege o movimentos dos planetas por exemplo, a força fraca, respon-sável pelas desintegrações radioactivas, a força electromagnética, que é dominanteao nível atómico (ligações químicas) e a força forte, responsável pela coesão dosnúcleos onde existem protões a repelir-se electromagneticamente.

Interacções

Tal como foi experimentalmente constatado por Coulomb em 1784, a força entreduas cargas eléctricas Q1 e Q2 separadas por uma distância r é proporcional aoproduto das cargas Q1 Q2, inversamente proporcional ao quadrado da distância r2

e a sua direcção encontra-se ao longo da linha que une as duas cargas:

~F =1

4πε0

Q1 Q2

r2~ur =

1

4πε0

Q1 Q2

r3~r (1.1)

O vector ~r aponta da carga Q1 para a carga Q2 no caso de se querer aforça aplicada pela carga 1 sobre a 2, ou vice-versa no caso contrário. Ofactor 1

4πε0aparece devido ao sistema SI de unidades escolhido (MKSA),

onde a carga se expressa em Coulomb (C) e a distância em metros (m).

Esta lei experimental é a lei fundamental da electrostática no sentido em queas outras leis são dela derivadas.

Lei de Coulomb

A constante ε0 é conhecida como constante dieléctrica ou permitividade dovácuo. O seu valor poderia ser determinado a partir da medição da força entreduas cargas pontuais colocadas a uma distância conhecida, mas isso não é prático.Podemos mais facilmente fazer a sua determinação experimental a partir da medidada capacidade de um condensador, como se verá mais adiante.

ε0 = 8, 8542× 10−12 [F.m−1]

4πε0 ≃ 1

9× 109[F.m−1]

Permitividade eléctrica

F.Barão, L.F.Mendes Electromagnetismo e Óptica (MEEC-IST) 7

Page 14: Colecção de Problemas de Electromagnetismo e Ópticafcomp.ist.utl.pt/EO.web/Book.29_01_2014.pdf · Preâmbulo Esta colectânea de problemas resulta da experiência dos autores na

Capítulo 1

Quando existem mais que duas cargas, a força total sentida por uma das cargas Qresulta da soma vectorial das forças aplicadas por cada uma das cargas existentes àsua volta Qi (princípio da sobreposição):

~F = ~F1 + ~F2 + ~F3 + · · · = 1

4πε0

Q Q1

r21

~ur1 +1

4πε0

Q Q2

r22

~ur2 + · · ·

= Q∑

i

1

4πε0

Qi

r2i

~uri (1.2)

Princípio da Sobreposição

É útil passar da ideia de força eléctrica entre cargas para a noção de campo eléctricoproduzido por cargas. Admitamos que uma carga de teste estacionária Q0 colocadanuma dada região do espaço sente uma força ~F (aqui o facto da carga estar emrepouso é importante porque evita a existência da força magnética como se verámais adiante). Pode-se então afirmar que nessa região do espaço existe um campoeléctrico definido por:

~E =~F

Q0

[V.m−1] ≡ [N.C−1] (1.3)

As cargas eléctricas são pois as responsáveis pela existência do campo eléctrico. Ocampo eléctrico produzido por uma carga Q1 situada à distância r da carga de teste,é então dado por:

~E1 =1

4πε0

Q1

r2~ur (1.4)

Um conjunto de cargas pontuais Q1,Q2, · · · ,Qi colocadas à distânciar1, r2, · · · , ri de um ponto P produzirão o seguinte campo eléctrico, obtido apartir do princípio da sobreposição:

~E =1

4πε0

(Q1

r21

~ur1 +Q2

r22

~ur2 + · · ·)

=1

4πε0

i

Qi

r2i

~uri (1.5)

Campo Eléctrico

A visualização do campo elétrico pode ser feita recorrendo a uma representaçãográfica com linhas de campo, inventada por Faraday. Estas linhas são desenhadasde forma a que em cada ponto o campo eléctrico seja tangente à linha de campo.São também chamadas linhas de força uma vez que uma carga de teste colocadanuma linha de campo, sentirá uma força também tangente à linha. A representaçãográfica do campo eléctrico permite-nos ter quer uma visão da magnitude do campo,através da densidade de linhas de campo, quer uma visão da sua direcção, atravésda orientação das linhas. As linhas aproximam-se em regiões de campo mais intensoe afastam-se em regiões onde o campo seja menor.

Linhas de Campo

E

F.Barão, L.F.Mendes Electromagnetismo e Óptica (MEEC-IST) 8

Page 15: Colecção de Problemas de Electromagnetismo e Ópticafcomp.ist.utl.pt/EO.web/Book.29_01_2014.pdf · Preâmbulo Esta colectânea de problemas resulta da experiência dos autores na

Capítulo 1

Quando a carga eléctrica se distribui de forma contínua, seja ao longo de um fio (λ[C.m−1]), de uma superfície (σ [C.m−2]) ou de um volume (ρ [C.m−3]), o campoeléctrico pode ser calculado dividindo a carga em pequenos elementos dq. Cada umdestes elementos de carga, à distância r de um ponto P, produz um campo eléctricoinfinitesimal nesse ponto:

d~E =1

4πε0

dq

r2~ur (1.6)

Para se obter o campo eléctrico total no ponto P, devem ser adicionados vectorial-mente os campos eléctricos infinitesimais (d~E) criados por todos os elementos decarga dq (isto é, integrados): ~E =

i d~Ei →

∫d~E.

~E =1

4πε0

∫dq

r2~ur com : dq = λ dx, σ dS, ρdV (1.7)

A simplicidade da resolução de problemas com distribuição contínua de cargas passapor uma escolha criteriosa do sistema de coordenadas a usar nas definições dasdistâncias e carga.

Distribuição contínua decarga

dq

r

P

Como a força electrostática só depende da posição, o campo electrostático é dotipo conservativo. Isto pode ser facilmente verificado calculando o trabalho da forçaeléctrica que uma carga +Q realiza sobre uma carga de teste +Q0 num caminhofechado:

W =

Γ

~F · d~ℓ =

Γ

Q0

Q

4 π ε0

1

r2d~ℓ = Q0

Q

4 π ε0

∫ P

P

dr

r2

= Q0

Q

4 π ε0

[

−1

r

]P

P

= 0

O princípio da sobreposição permite-nos generalizar esta conclusão para o campoelectrostático criado por um conjunto arbitrário de cargas. Assim para um caminhomacroscópico fechado podemos escrever,

Γ

~F · d~ℓ = 0 ⇒∮

Γ

~E · d~ℓ = 0

e localmente (em cada ponto do espaço), utilizando o teorema de Stokes,

~∇ × ~E = 0

É precisamente o facto de o campo electrostático ser conservativo que nos permitiráfalar de energia potencial electrostática e de potencial eléctrico.

Campo conservativo

A energia potencial electrostática associada a um sistema de cargas eléctricas cor-responde ao trabalho realizado para o constituir. Considera-se habitualmente que ascargas são trazidas do infinito. Nesta situação não há interacção entre as cargas e,portanto, a energia potencial electrostática é nula. No entanto, a escolha do infinitocomo ponto de refência é arbitrária, isto é, pode-se escolher um outro ponto doespaço e aí considerar a energia potencial nula. Em termos físicos o que é relevanteé a variação da energia potencial do sistema.

Energia Potencial

F.Barão, L.F.Mendes Electromagnetismo e Óptica (MEEC-IST) 9

Page 16: Colecção de Problemas de Electromagnetismo e Ópticafcomp.ist.utl.pt/EO.web/Book.29_01_2014.pdf · Preâmbulo Esta colectânea de problemas resulta da experiência dos autores na

Capítulo 1

Comecemos por considerar uma carga pontual estacionária +Q1 colocada numaregião livre de campo eléctrico. Esta pode ser deslocada desde o ponto de referênciasem necessidade de aplicar uma força. A sua energia electrostática é portanto nula.Uma segunda carga, +Q2, é trazida até um ponto P a uma distância rP da primeira.Neste caso foi necessário vencer a força de repulsão existente entre as cargas e aplicaruma força contrária à força eléctrica ~Fa = −Q2

~E. A energia do sistema de duascargas corresponde então à energia gasta para trazer a segunda carga desde infinito:

UE = W =

∫ P

~Fa · d~ℓ = −Q1Q2

4πε0

∫ rP

dr

r2= +Q2

Q1

4πε0

1

rP(1.8)

A expressão da energia potencial associada ao sistema de duas cargas dada pelaexpressão 1.8 pode ser escrita em termos de um potencial eléctrico φ criado pelacarga Q1 e da carga Q2:

UE = Q2φ (1.9)

Assim, em geral, o potencial eléctrico num dado ponto P do espaço corresponde aotrabalho (por unidade de carga) que teria que ser realizado para trazer uma cargaaté aí desde um ponto de referência:

φP =

∫ P

Ref

−~E · d~ℓ =

∫ Ref

P

~E · d~ℓ (1.10)

O ponto de referência do potencial é o ponto onde se considera que este se anula(φ(Ref) = 0). Para cargas pontuais ou localizadas em regiões finitas do espaço,a escolha do inifinito como ponto de referência é conveniente; a escolha de umoutro ponto implicaria o aparecimento de uma constante na expressão do potencial.

O potencial eléctrico num ponto P a uma distância r de uma carga pontualQ é dado por:

φP =

∫ ∞

r

~E · d~ℓ =

∫ ∞

r

1

4πε0

Q

r2dr =

1

4πε0Q

[

−1

r

]∞

r

(1.11)

=1

4πε0

Q

r

Da mesma forma que para o campo eléctrico, se tivermos uma distribuição de car-gas discreta ou contínua, aplicaremos o princípio da sobreposição para se obter opotencial eléctrico:

φP =∑

i

φi →∫

1

4πε0

dq

r(1.12)

Potencial eléctrico

A diferença de potencial entre dois pontos A e B é dada por:

VAB ≡ φA − φB

=

∫ Ref

A

~E · d~ℓ −∫ Ref

B

~E · d~ℓ

=

∫ Ref

A

~E · d~ℓ +∫ B

Ref

~E · d~ℓ

Diferença de Potencial

F.Barão, L.F.Mendes Electromagnetismo e Óptica (MEEC-IST) 10

Page 17: Colecção de Problemas de Electromagnetismo e Ópticafcomp.ist.utl.pt/EO.web/Book.29_01_2014.pdf · Preâmbulo Esta colectânea de problemas resulta da experiência dos autores na

Capítulo 1

VAB =

∫ B

A

~E · d~ℓ (1.13)

Note-se que a escolha do ponto de referência não é relevante uma vez que nãocontribui para a diferença de potencial.

Tendo em conta a definição de diferença de potencial entre dois pontos A e B,

∫ B

A

~E · d~ℓ = φA − φB = −∫ B

A

dφ = −∫ B

A

d~ℓ· d~ℓ

obtém-se uma relação entre o campo eléctrico e o potencial eléctrico:

~E = −dφ

d~ℓ≡ −~∇φ (1.14)

As superfícies equipotenciais definem-se então como conjuntos de pontos ao longodos quais o potencial se mantém constante: dφ = 0. Conhecendo as linhas decampo eléctrico, as equipotenciais constróem-se facilmente unindo pontos vizinhos(d~ℓ) em direcções perpendiculares ao campo eléctrico (d~ℓ ⊥ ~E).

Relação entre o campo epotencialeléctrico

E

dl

As moléculas são bons exemplos de objectos electricamente neutros mas que noentanto produzem um campo eléctrico e interagem com campos eléctricos externos.O sistema mais simples deste tipo é o dipolo eléctrico que consiste em duas cargasiguais de sinal oposto ±Q e distanciadas de d. Uma aproximação habitualmentefeita ao dipolo eléctrico é a de que a distância entre as cargas d é desprezávelface à distância onde se pretende calcular o campo eléctrico (aproximação dipolar).Esta configuração de cargas pode ser caracterizada através do seu momento dipolar,definido como:

~p =∑

i

Qi~di = Q

(

~d+ − ~d−

)

= Q~d (1.15)

O cálculo do campo eléctrico produzido por um dipolo num ponto P qualquer a umadistância r >> d, pode ser feito a partir da expressão do potencial eléctrico em Pproduzido por ambas as cargas:

φP = φ+ +φ− =Q

4πε0

(1

r+− 1

r−

)

≃ Qd cos θ

4πε0r2=

1

4πε0

~p · ~rr3

(1.16)

O campo eléctrico pode ser obtido tendo em conta a relação entre o potencialeléctrico e o campo eléctrico, dada por 1.14:

Er = ∂φ(r,θ)∂r

= 14πε0

2p cos θr3

Eθ = ∂φ(r,θ)r∂θ

= 14πε0

psenθr3

(1.17)

Trata-se pois de um campo que, não se podendo considerar já nulo (quem estiverrealmente distante vê uma carga eléctrica nula!), decresce no entanto mais rapida-mente que o campo eléctrico criado por cada uma das cargas pontuais.

Dipolo eléctrico

d −d+

r + r −

P

r

θ

Q Qp

F.Barão, L.F.Mendes Electromagnetismo e Óptica (MEEC-IST) 11

Page 18: Colecção de Problemas de Electromagnetismo e Ópticafcomp.ist.utl.pt/EO.web/Book.29_01_2014.pdf · Preâmbulo Esta colectânea de problemas resulta da experiência dos autores na

Capítulo 1

Quando um dipolo eléctrico é colocado na presença de um campo eléctrico uniforme~E = E0~ux, sente uma força total nula. Existe, no entanto, um momento da forçaeléctrica ~N =

i=± ~ri × ~Fi que o tenderá a alinhar com o campo eléctrico. Aenergia potencial do dipolo, calculada como sendo o trabalho realizado por umaforça externa (contra o campo, portanto!) para levar o dipolo da posição angularθ = 90 até uma posição angular qualquer (θ), é:

UE =

∫ θ

90

(

−~FE

)

· d~ℓ = 2

∫ θ

90

QE0 cosαℓ

2dθ

=

∫ θ

90

Q E0 senθ ℓ dθ = −Q ℓ E0 cosθ = −~p · ~E

O ponto de partida θ = 90 é escolhido uma vez que para esse ângulo a energiapotencial do dipolo é nula: as cargas, simétricas, encontram-se ao mesmo potencialeléctrico (ver equação 1.12). O sinal negativo significa que o dipolo ao alinhar-secom o campo, diminui a sua energia potencial; esta energia é mínima quando omomento dipolar eléctrico se encontra paralelo ao campo eléctrico externo.

Energia potencial de umdipolo eléctrico num

campo eléctrico exteriorFE

FE

θ E

−Q

+Q

p

A lei de Gauss é uma consequência da lei de Coulomb e resulta do facto do campoeléctrico criado por um elemento de carga dq diminuir com 1/r2, enquanto queo elemento de área (em coordenadas esféricas) aumenta com r2. Desta forma, oproduto da carga pela área é independente da distância do elemento de área à cargaeléctrica. Esta lei relaciona o fluxo do campo eléctrico (ΦE) que atravessa umasuperfície fechada com a carga eléctrica total existente no seu interior (Qint):

~E · ~n dS =Qint

ε0(1.18)

Esta lei pode ser escrita na forma local como:

~∇ · ~E =ρ

ε0(1.19)

A lei de Gauss permite:

• determinar a carga eléctrica contida numa superfície fechada, se conhecermoso fluxo do campo eléctrico que a atravessa.

• determinar o módulo do campo eléctrico para distribuições de carga comsimetria espacial, usando para tal uma superfície fechada conveniente.

No cálculo do campo eléctrico, a aplicabilidade da lei de Gauss encontra-se reduzidaa três geometrias tipo, tal como se mostra adiante nos exemplos de aplicação:

• simetria esférica

• simetria cilíndrica

• plano infinito

Lei de Gauss

F.Barão, L.F.Mendes Electromagnetismo e Óptica (MEEC-IST) 12

Page 19: Colecção de Problemas de Electromagnetismo e Ópticafcomp.ist.utl.pt/EO.web/Book.29_01_2014.pdf · Preâmbulo Esta colectânea de problemas resulta da experiência dos autores na

Capítulo 1

No interior de um condutor em equilíbrio electrostático, o campo eléctrico tem de sernulo (~E = 0), caso contrário ter-se-ia uma força eléctrica aplicada às suas cargaslivres e estas mover-se-iam. Como se pode verificar facilmente com recurso à leide Gauss, um fluxo nulo do campo eléctrico através de qualquer superfície fechadadefinida no interior do condutor implica a não existência de carga eléctrica no seuinterior. Desta forma, uma carga eléctrica introduzida num meio condutor em equilí-brio electrostático desloca-se para a sua superfície exterior. Os tempos de relaxaçãoda carga dependem da condutividade eléctrica do meio, sendo da ordem de 10−18

segundos num material bom condutor. O campo eléctrico é normal à superfíciedo condutor, dependendo a sua intensidade da densidade de carga superficial (σ) epode ser derivado a partir da lei de Gauss como:

~E ≡ ~E⊥ =σ

ε0~u⊥ (1.20)

Equilíbrio electrostáticonos condutores

Nenhum material é um isolante perfeito. Os materiais que se caracterizampor muito baixas condutividades eléctricas são chamados materiais dieléctri-cos. Um campo eléctrico aplicado a um dieléctrico causa um deslocamentoespacial da carga eléctrica positiva e negativa associada aos átomos ou mo-léculas (em sentidos opostos!), fazendo aparecer dipolos eléctricos. O vectorpolarização ~P descreve a polarização do material num dado ponto do es-paço e corresponde ao momento dipolar por unidade de volume (d~p/dV ).

Um material dieléctrico é caracterizado pela sua susceptibilidade eléctrica χE

(grandeza adimensional) e é do tipo linear se a susceptibilidade não depender docampo aplicado, do tipo homogéneo se a susceptibilidade não depender da posiçãono dieléctrico e do tipo isotrópico caso a susceptibilidade não dependa da direcção.A polarização no dieléctrico é proporcional ao campo eléctrico a que este estásujeito (~E) e que resulta da soma do campo exterior com o campo produzido peladistribuição de carga dipolar:

~P =d~p

dV=

i ~pi |dVd V

= ε0 χE~E [C.m−2] (1.21)

Em geral, existe carga de polarização à superfície de um dieléctrico. A densidadede carga de polarização superficial existente num dado ponto da superfície é dadapor:

σpol = ~P · ~next (1.22)

sendo ~next a normal exterior um vector unitário que aponta para o exterior dodieléctrico.

Pode também existir carga de polarização em volume no caso de o dieléctrico não seruniforme ou no caso de existir carga eléctrica introduzida (que não é de polarização)no interior do dieléctrico. A densidade de carga de polarização em volumeexistente numa dada região do dieléctrico é dada por:

ρpol = −~∇ · ~P (1.23)

e a carga de polarização contida numa superfície fechada é dada por,

Qpol =

V

ρpol dV = −∮

S

~P · ~n dS (1.24)

Campo eléctrico em mate-riais dieléctricos

F.Barão, L.F.Mendes Electromagnetismo e Óptica (MEEC-IST) 13

Page 20: Colecção de Problemas de Electromagnetismo e Ópticafcomp.ist.utl.pt/EO.web/Book.29_01_2014.pdf · Preâmbulo Esta colectânea de problemas resulta da experiência dos autores na

Capítulo 1

Portanto, um campo eléctrico ~E0 aplicado a um dieléctrico origina uma polarizaçãodo dieléctrico, isto é, induz uma dada carga de polarização seja em superfície, sejaem volumezação induzida no material.Recorrendo à lei Gauss, podemos calcular o campo eléctrico separando as cargaseléctricas em termos de cargas livres (Qliv) e cargas de polarização (Qpol):

S

~E · ~n dS =

i Qi

ε0=

Qliv + Qpol

ε0(1.25)

O cálculo do campo eléctrico através da expressão 1.25 implica o conhecimento prévioquer das cargas livres, quer das cargas de polarização. No entanto, se substituirmosna equação 1.25 a carga polarização expressa em 1.24, obtem-se:

S

(ε0 ~E + ~P ) · ~n dS = Qliv (1.26)

Definindo o vector Deslocamento do Campo Eléctrico ~D como:

~D = ε0 ~E + ~P = ε0(1 + χE)~E = ε ~E (1.27)

obtém-se a lei de Gauss generalizada na forma integral:∮

S

~D · ~n dS = Qliv (1.28)

e na forma local:

~∇ · ~D = ρliv (1.29)

A permitividade eléctrica do meio, ε, caracteriza a polarizabilidade do material.

Lei de Gauss generalizada

A energia associada a um sistema de múltiplas cargas é, como se referiu anterior-mente, o trabalho necessário para o constituir. Por exemplo, a energia de um sistemade três cargas pode ser calculada como se fez para o sistema de duas cargas (equação1.8) tendo agora em conta que a terceira é trazida contrariando a repulsão exercidapelas outras duas:

UE =1

4πε0

(

Q2

Q1

r12+ Q3

Q1

r13+ Q3

Q2

r12

)

= Q2φ12 + Q3 (φ13 + φ23)

Este resultado pode ser reescrito em termos das cargas (Qi) e dos potenciais a quecada uma se encontra (φi) devido à presença das outras duas:

UE =1

2(Q1φ1 + Q2φ2 + Q3φ3)

Generalizando para um sistema de N cargas:

U =1

2

i

Qi φi (1.30)

Energia Potencial electros-tática de um sistema de car-gas

F.Barão, L.F.Mendes Electromagnetismo e Óptica (MEEC-IST) 14

Page 21: Colecção de Problemas de Electromagnetismo e Ópticafcomp.ist.utl.pt/EO.web/Book.29_01_2014.pdf · Preâmbulo Esta colectânea de problemas resulta da experiência dos autores na

Capítulo 1

Um condutor carregado gera um campo eléctrico à sua volta ficando a um certopotencial φ em relação ao infinito. Define-se a capacidade do condutor como arelação entre a sua carga eléctrica Q e o potencial a que se encontra:

C =Q

φ[F ] (1.31)

Esta expressão traduz a relação de linearidade entre o campo eléctrico e a carga quelhe dá origem.

Num condensador, sistema de dois condutores ou armaduras que se encontram auma diferença de potencial V , a capacidade é definida por:

C =Q

V(1.32)

sendo Q a carga armazenada em cada um dos condutores (+Q e −Q). A capacidadede um condensador só depende da sua geometria e do meio que os separa.

Condensadores

A energia (potencial electrostática) de um condensador corresponde ao trabalhonecessário para o carregar. Para a calcular, tem que se contabilizar o trabalhorealizado sobre cada elemento de carga dq para o transportar de uma armadura paraa outra, contra a força eléctrica existente (cada vez maior, à medida que se carregao condensador!).

UE =

~F · d~ℓ =

dq ~E · d~ℓ︸ ︷︷ ︸

V

=

∫ Q

0

V (q) dq =

∫ Q

0

q

Cdq (1.33)

Daqui resulta:

UE =1

2

Q2

C=

1

2C V 2 (1.34)

Energia armazenada porum condensador

E+Q −Q

dq

V

A capacidade equivalente de uma associação em série de n condensadores é dadapor:

1

Ceq

=1

C1

+1

C2

+ · · ·+ 1

Cn

(1.35)

e uma associação em paralelo de condensadores é dada por:

Ceq = C1 + C2 + · · · + Cn (1.36)

Associação de condensado-res

F.Barão, L.F.Mendes Electromagnetismo e Óptica (MEEC-IST) 15

Page 22: Colecção de Problemas de Electromagnetismo e Ópticafcomp.ist.utl.pt/EO.web/Book.29_01_2014.pdf · Preâmbulo Esta colectânea de problemas resulta da experiência dos autores na

Capítulo 1

Sendo a carga eléctrica a criadora do campo electrostático e sendo necessária arealização de trabalho para constituir as distribuições de carga, ou seja, para criaros campos eléctricos, é natural que a própria expressão do campo eléctrico nos dêinformações sobre a energia de um sistema. Com efeito, associado a um campoeléctrico ~E existe associada uma densidade de energia dada por

uE =1

2ε E2 [J/m3] (1.37)

e num volume V existe uma energia

UE =

V

uE dV [J] (1.38)

Energia do campo eléctrico

Consideremos um sistema de cargas isoladas e o campo eléctrico por estas produzido.Para se modificar a geometria do sistema movendo d~ℓ uma certa quantidade de carga,é necessário realizar trabalho contra a força eléctrica ~FE que actua essa carga:

dW = ~Fa · d~ℓ = −~FE · d~ℓ

Sendo o campo eléctrico conservativo e não havendo variação de energia cinética,esse trabalho traduz-se necessariamente na variação da energia potencial do sistema:

dU = −~FE · d~ℓ ⇒ ~FE = −dU

d~ℓ

De forma mais simplificada, a força eléctrica numa dada direcção ~us pode ser cal-culada por:

(

~FE

)

s= −dU(s)

ds~us (1.39)

Força e Energia Potencial

Consideremos agora que o sistema não está isolado e que existem fontes que man-têm as diferenças de potencial entre os componentes do sistema. Neste caso, aomodificar-se a geometria do sistema, as fontes realizam trabalho pondo e retirandocarga de forma a manter os potenciais eléctricos. Contabilizando este trabalho adi-cional das fontes obtém-se:

(

~FE

)

s= +

dU(s)

ds~us (1.40)

Estas expressões são um modo alternativo para calcular forças eléctricas quando seconhece a energia potencial do sistema. A uma alteração da geometria do sistemasegundo a direcção em que se pretende calcular a força eléctrica corresponde umavariação da energia potencial. Sendo este um método de deslocamentos virtuais,qualquer uma das expressões anteriores 1.39, 1.40 conduzirá ao mesmo resultado.

F.Barão, L.F.Mendes Electromagnetismo e Óptica (MEEC-IST) 16

Page 23: Colecção de Problemas de Electromagnetismo e Ópticafcomp.ist.utl.pt/EO.web/Book.29_01_2014.pdf · Preâmbulo Esta colectânea de problemas resulta da experiência dos autores na

Capítulo 1

F.Barão, L.F.Mendes Electromagnetismo e Óptica (MEEC-IST) 17

Page 24: Colecção de Problemas de Electromagnetismo e Ópticafcomp.ist.utl.pt/EO.web/Book.29_01_2014.pdf · Preâmbulo Esta colectânea de problemas resulta da experiência dos autores na

Capítulo 1

1.2 Exercícios Resolvidos

1.2.1 Distribuição discreta de carga eléctrica

Duas cargas eléctricas positivas Q1 e Q2 encontram-se a uma distância d umada outra. Na sua vizinhança existe uma carga eléctrica de teste q0 num ponto P ,de coordenadas (x, 0).

• Campo eléctrico produzido pela carga Q1 no ponto PPela lei de Coulomb, o campo eléctrico vem:

~E1 =Q1

4πε0

~r1

r31

com o vector posição ~r1, que une a carga Q1 ao ponto P , a ser dado por:

~r1 = x~ux +d

2~uy

Desta forma, vem para o campo eléctrico:

~E1 =Q1

4πε0

x~ux + d2~uy

[

x2 +(d2

)2] 3

2

• Campo eléctrico total em PO campo eléctrico total existente em P resulta da soma dos campos eléc-tricos produzidos pelas cargas Q1 e Q2 em P . Tem-se então:

~E = ~E1 + ~E2 =Q1

4πε0

x~ux + d2~uy

[

x2 +(d2

)2] 3

2

+Q2

4πε0

x~ux − d2~uy

[

x2 +(d2

)2] 3

2

=1

4πε0

(Q1 + Q2)

x[

x2 +(d2

)2] 3

2

~ux +

+ (Q1 − Q2)d2

[

x2 +(d2

)2] 3

2

~uy

Vemos portanto que o campo eléctrico tem componentes Ex e Ey. Noentanto, no caso em que Q1 = Q2, o campo eléctrico reduz-se umaexpressão mais simples, possuindo somente componente Ex:

~E =Q

2πε0

x[

x2 +(d2

)2] 3

2

~ux

• Força sobre a carga de teste quando Q1 = Q2

~F = q0 ~E = q0Q

2πε0

x[

x2 +(d2

)2] 3

2

~ux

++

Q1 Q2

r1 r 2

d/2d/2

P

x

y

F.Barão, L.F.Mendes Electromagnetismo e Óptica (MEEC-IST) 18

Page 25: Colecção de Problemas de Electromagnetismo e Ópticafcomp.ist.utl.pt/EO.web/Book.29_01_2014.pdf · Preâmbulo Esta colectânea de problemas resulta da experiência dos autores na

Capítulo 1

• Linhas do campo eléctrico quando Q1 = Q2

As linhas do campo eléctrico associadas a cada uma das cargas positivas,caso estas estivessem isoladas, corresponderiam a linhas radiais representa-das na figura a traço interrompido. O campo eléctrico resultado das duascargas, será diferente, tal como se calculou anteriormente. As linhas decampo eléctrico devem reflectir o facto de o campo eléctrico ser muitopequeno ou nulo, na região entre as duas cargas.

• Trabalho realizado contra o campo eléctrico para trazer a carga q0para o ponto PAs interacções existentes no campo eléctrico estão associadas a variações deenergia. O trabalho realizado contra o campo eléctrico produzido pela duascargas positivas +Q, para transportar a carga de teste q0 desde infinitoaté uma distância x do eixo das cargas por um caminho ao longo do eixodos XX, calcula-se da seguinte forma:

W =

∫ x

(

−~F)

· d~ℓ = −q0

∫ x

~E · d~ℓ

= −q0Q

2πε0

∫ x

x[

x2 +(d2

)2] 3

2

~ux · d~ℓ

= −q0Q

2πε0

∫ x

x[

x2 +(d2

)2] 3

2

dx

= −q0Q

2πε0

− 1√

x2 +(d2

)2

x

= q0Q

2πε0

1√

x2 +(d2

)2

Vemos então que foi necessário realizar trabalho no transporte da cargaq0 para contrariar a força eléctrica aplicada. Desta forma, a carga deteste adquiriu energia potencial, que pode ser recuperada (sob a forma deenergia cinética, por exemplo) caso fosse largada. De notar que o trabalhorealizado não depende do caminho seguido; isto é característico de umaforça conservativa, como é o caso da força eléctrica.

• Potencial eléctricoEscolhendo o infinito como ponto de referência, o potencial eléctrico emP é dado por:

φP =

∫ ∞

P

~E · d~ℓ =

∫ ∞

x

Q

2πε0

x[

x2 +(d2

)2]3/2

dx

=Q

2πε0

− 1√

x2 +(d2

)2

x

=Q

2πε0

1√

x2 +(d2

)2

De notar que o potencial eléctrico em P se poderia ter calculado utilizandoo princípio da sobreposição; isto é, somando os potenciais criado em Pquer pela carga +Q1, quer pela carga +Q2:

F.Barão, L.F.Mendes Electromagnetismo e Óptica (MEEC-IST) 19

Page 26: Colecção de Problemas de Electromagnetismo e Ópticafcomp.ist.utl.pt/EO.web/Book.29_01_2014.pdf · Preâmbulo Esta colectânea de problemas resulta da experiência dos autores na

Capítulo 1

φ = φ1 + φ2 =

∫ ∞

P

~E1 · d~ℓ+∫ ∞

P

~E2 · d~ℓ

• Energia potencial da carga q0Dado que o trabalho calculado anteriormente corresponde ao ganho deenergia potencial da carga e que no infinito (ponto de referência) a energiapotencial é nula, vem para a energia potencial da carga q0:

UE = W = q0Q

2πε0

1√

x2 +(d2

)2

De notar que a energia potencial poderia ter sido calculada com recursodirecto ao potencial eléctrico calculado anteriormente: UE = q0 φ.

F.Barão, L.F.Mendes Electromagnetismo e Óptica (MEEC-IST) 20

Page 27: Colecção de Problemas de Electromagnetismo e Ópticafcomp.ist.utl.pt/EO.web/Book.29_01_2014.pdf · Preâmbulo Esta colectânea de problemas resulta da experiência dos autores na

Capítulo 1

1.2.2 Distribuição contínua de carga eléctrica num fio

Considere uma distribuição contínua de carga positiva existente ao longo de um fiocuja densidade de carga é λ (C/m). Num ponto P , a uma distância d, encontra-seuma carga de teste q0.

• Campo eléctrico produzido pelo fio de carga no ponto PA determinação do campo eléctrico produzido por uma distribui-ção contínua de carga num ponto P é feita tendo um conta oprincípio da sobreposição, isto é, determina-se em primeiro lugaro campo eléctrico infinitesimal (d~E) produzido por ume elementoinfinitesimal de carga (dq) e depois somam-se (integram-se!) ascontribuições dos diferentes elementos de carga ao longo do fio.

Os elementos infinitesimais de carga e campo eléctrico expressam-sepor:

dq = λdy

d~E =dq

4πε0

~r

r3=

λdy

4πε0

d~ux − y~uy

(d2 + y2)3/2

E portanto, o campo eléctrico obtém-se por integração ao longo do fio delimites −L1 e +L2:

~EP =

d~E =λ

4πε0

∫ +L2

−L1

d~ux − y~uy

(d2 + y2)3/2

dy

4πε0

[

d

∫ +L2

−L1

dy

(d2 + y2)3/2~ux −

∫ +L2

−L1

ydy

(d2 + y2)3/2~uy

]

4πε0

d

[

1

d2

y√

d2 + y2

]+L2

−L1

~ux −[

− 1√

d2 + y2

]+L2

−L1

~uy

4πε0

[1

d

(L2√

d2 + L22+

L1√d2 + L1

2

)

~ux+

(

1√

d2 + L22− 1√

d2 + L12

)

~uy

]

Como se vê, no caso de a carga se distribuir assimetricamente no fio emrelação ao ponto P , o campo eléctrico possui componentes segundo x (Ex)e segundo y (Ey). No entanto, caso a carga se distribua simetricamenteem relação ao ponto P , o que significa o fio ter limites ±L, o campoeléctrico possui somente componente segundo x e vem:

~E =λ2L

4πε0

1

d√d2 + L2

~ux =λL

2πε0

1

d

1√d2 + L2

~ux

Podemos agora analisar o que acontece nos casos extremos em que o fio émuito comprido (L >> d) e muito pequeno (L << d), quando compa-rado com a distância ao ponto P :

q0

++

+++

+

+

+

+

+

+

dq

P

r

dx

y

dE

F.Barão, L.F.Mendes Electromagnetismo e Óptica (MEEC-IST) 21

Page 28: Colecção de Problemas de Electromagnetismo e Ópticafcomp.ist.utl.pt/EO.web/Book.29_01_2014.pdf · Preâmbulo Esta colectânea de problemas resulta da experiência dos autores na

Capítulo 1

– Fio muito longo (L >> d)

~E =λ

2πε0

1

d

L√d2 + L2

~ux =λ

2πε0

1

d

L2

d2 + L2~ux

2πε0

1

d

√1

1 +(dL

)2 ~ux → λ

2πε0

1

d~ux

– Fio muito pequeno (L << d)

~E =λ

2πε0

1

d

L√d2 + L2

~ux

=λL

2πε0

1

d

1

d

1 +(Ld

)2~ux → λL

2πε0

1

d2~ux

Neste caso, o campo eléctrico obtido corresponde ao campo produ-zido por uma carga pontual de valor q = 2λL. Isto era de esperaruma vez que um fio muito pequeno é em boa aproximação uma cargapontual.

• Força eléctrica sobre a carga de teste q0A força que actua a carga de teste q0 é dada por: ~F = q0 ~E.

• Potencial eléctrico criado pelo fioO potencial eléctrico produzido pelo fio carregado pode ser determinado re-correndo à relação entre o campo e o potencial, ~∇φ = −~E; isto é, poder-se-ia obter o potencial por integração do campo eléctrico. No entanto,vamos fazer o cálculo do potencial, usando o princípio da sobreposição.

Comecemos por determinar o potencial produzido por um elemento infi-nitesimal de carga dq, utilizando o infinito como ponto de referência, eintegre-se ao longo de todo o fio:

dφ =1

4πε0

dq

r

φ =1

4πε0

∫ +L2

−L1

λdy√

x2 + y2=

λ

4πε0

[

ln(

y +√

x2 + y2)]+L2

−L1

4πε0

[

ln

(

L2 +√

x2 + L22

)

− ln

(

−L1 +√

x2 + L21

)]

4πε0ln

(

L2 +√

x2 + L22

−L1 +√

x2 + L21

)

4πε0ln

(

L2 +√

x2 + L22

) (

+L1 +√

x2 + L21

)

(

−L1 +√

x2 + L21

) (

+L1 +√

x2 + L21

)

4πε0ln

(

L2 +√

x2 + L22

) (

+L1 +√

x2 + L21

)

x2

No caso de termos um fio com uma distribuição de carga simétrica emrelação ao ponto, vem:

F.Barão, L.F.Mendes Electromagnetismo e Óptica (MEEC-IST) 22

Page 29: Colecção de Problemas de Electromagnetismo e Ópticafcomp.ist.utl.pt/EO.web/Book.29_01_2014.pdf · Preâmbulo Esta colectânea de problemas resulta da experiência dos autores na

Capítulo 1

φ =λ

4πε0ln

(

L +√x2 + L2

)2

x2

2πε0ln

(

L +√x2 + L2

x

)

• Energia potencial da carga de testeA energia potencial da carga de teste q0 num ponto P a uma distância x dofio, corresponde à energia gasta (ou recebida) para a lá colocar, partindo deum ponto de referência onde a energia potencial seja nula (habitualmenteo infinito, mas nem sempre!). No nosso caso, esta pode ser calculadaconhecendo o potencial eléctrico no ponto P :

U = q0 φP

• Campo eléctrico calculado pela Lei de GaussPara o caso do fio muito comprido (também dito vulgarmente infinito), adeterminação do campo eléctrico no ponto P pode ser feita recorrendo àlei de Gauss. Para tal, recorre-se a uma superfície cilíndrica que passe porP e tira-se partido do facto do campo eléctrico (que para o fio infinito, sódepende da distância radial) ser constante em toda a superfície da superfícielateral do cilindro. Calculemos primeiro o fluxo do campo eléctrico. Esteresulta da integração do campo eléctrico ao longo das diferentes superfícies(duas tampas mais superfície lateral) que compôem o cilindro.

S

~E · ~n dS =

tampas

~E · ~n dS +

lateral

~E · ~n dS

Uma vez que as linhas de campo eléctrico não atravessam as tampas docilindro, o fluxo do campo aí é nulo, vindo portanto:

S

~E · ~n dS =

lateral

~E · ~n dS

A lei de Gauss pode então escrever-se como:∫

lateral

~E · ~n dS =Qint

ε0

E

dS = E2πrL =λ L

ε0

E =λ

2πε0

1

r

~E =λ

2πε0

1

r~ur

F.Barão, L.F.Mendes Electromagnetismo e Óptica (MEEC-IST) 23

Page 30: Colecção de Problemas de Electromagnetismo e Ópticafcomp.ist.utl.pt/EO.web/Book.29_01_2014.pdf · Preâmbulo Esta colectânea de problemas resulta da experiência dos autores na

Capítulo 1

1.2.3 Distribuição contínua de carga eléctrica num plano infinito

Considere-se um plano muito grande (plano infinito) carregado com uma densidadede carga σ [C.m−2].

• Campo eléctrico pela lei de GaussO campo eléctrico pode ser facilmente calculado utilizando a lei deGauss (o plano infinito é uma das três geometrias em que a aplicação dalei de Gauss é o método mais simples para calcular o campo eléctrico).

Como primeiro passo é necessário verificar que as linhas de camposão perpendiculares ao plano. Para chegar a esta conclusão bastapensar que todos os pontos do plano infinito são o seu centro. Eno centro as componentes do campo eléctrico no plano do próprioplano infinito anulam-se devido à simetria do problema. Resta-nosassim a componente perpendicular ao plano (em todos os pontos!).

Conhecida a direcção do campo há que escolher a superfície apro-priada para aplicar a lei de Gauss. Neste caso pode utilizar-se umasuperfície cilíndrica, tal como se mostra na figura. Na superfície lateraldo cilindro (superfície S1) a normal é perpendicular ao campo enquantoque nas "tampas" (superfícies S2 e S3, de área A) a normal é paralela aocampo. Assim sendo, e constatando que esta superfície fechada contémno seu interior uma carga Qint = σ A, tem-se:

S

~E · ~n dS =Qint

ε0∫

S1

~E · ~n dS +

S2

~E · ~n dS +

S3

~E · ~n dS =σ A

ε0∫

S1

0 dS +

S2

E dS +

S3

E dS =σ A

ε0

E

S2

dS + E

S3

dS =σ A

ε0

2 E A =σ A

ε0

E

S2

dS + E

S3

dS =σ A

ε0

E =σ

2 ε0

Agora que calculámos o módulo do campo eléctrico podemos escrever asua expressão final:

~E =

+ σ

2 ε0~uz z > 0

− σ2 ε0

~uz z < 0

F.Barão, L.F.Mendes Electromagnetismo e Óptica (MEEC-IST) 24

Page 31: Colecção de Problemas de Electromagnetismo e Ópticafcomp.ist.utl.pt/EO.web/Book.29_01_2014.pdf · Preâmbulo Esta colectânea de problemas resulta da experiência dos autores na

Capítulo 1

1.2.4 Condutores em equilíbrio electrostático

Um condutor cilíndrico de raio R, comprimento ℓ (ℓ >> R) e que se encontraimerso no ar, está uniformemente carregado e possui uma carga total Q.

• Densidade de carga no condutorTratando-se de um condutor em regime electrostático, a sua carga estádistribuída na sua superfície, pois só deste modo o campo eléctrico seanula no seu interior. A densidade de carga será:

σ =Q

A=

Q

2πRℓ

ou, se preferirmos utilizar uma densidade de carga linear já que o condutortem a forma de um fio,

λ =Q

• Campo eléctricoDentro do condutor o campo é nulo, como já se referiu. Fora do condutor,tratando-se de uma distribuição de carga com geometria cilíndrica, as linhasde campo são radiais em coordenadas cilíndricas. Deste modo, utiliza-seuma superfície também cilíndrica para a aplicação da lei de Gauss. Nessasuperfície, a normal tal como o próprio campo, é radial, excepto nos toposda superfície cilíndrica onde a normal é perpendicular ao campo e o integralse anula (~E ⊥ ~n ⇒ ~E · ~n = 0). Além disso, como o problema temsimetria cilíndrica, o módulo do campo é constante para a superfície emque o integral não se anula.

r < R

~E = ~0

r > R∮

S

~E · ~n dS =Qint

ε0

E2πrL = Lλ

ε0

~E =1

2πε0

λ

r~ur

• Influência de um condutor externo de forma cilíndricaO condutor cilíndrico externo não se encontra carregado. No entanto,na presença do campo eléctrico do cilindro condutor interno, existe carganegativa na sua superfície interna e carga simétrica positiva na superfícieexterna. De notar, que a carga existente em ambas as superfícies (interna eexterna) é em módulo igual à carga existente no cilindro condutor interior+Q; isto resulta do facto do campo eléctrico ser nulo no interior doscondutores. Como o condutor exterior não destruiu a simetria cilíndrica doproblema e não está carregado, o campo permanece igual, quer no espaçoentre os condutores quer na região exterior aos condutores.

F.Barão, L.F.Mendes Electromagnetismo e Óptica (MEEC-IST) 25

Page 32: Colecção de Problemas de Electromagnetismo e Ópticafcomp.ist.utl.pt/EO.web/Book.29_01_2014.pdf · Preâmbulo Esta colectânea de problemas resulta da experiência dos autores na

Capítulo 1

• Influência de um condutor externo de forma paralelipipédicaNeste caso o condutor exterior destruiu a simetria cilíndrica do problema,apesar de não estar carregado. O campo altera-se de forma a garantir queas linhas de campo são perpendiculares à superfície do condutor exteriore anula-se no seu interior. Longe (com rigor matemático, no infinito) doscondutores o campo recupera a sua forma radial uma vez que o conjuntovolta a parecer um fio com densidade de carga λ.

F.Barão, L.F.Mendes Electromagnetismo e Óptica (MEEC-IST) 26

Page 33: Colecção de Problemas de Electromagnetismo e Ópticafcomp.ist.utl.pt/EO.web/Book.29_01_2014.pdf · Preâmbulo Esta colectânea de problemas resulta da experiência dos autores na

Capítulo 1

1.2.5 Campo eléctrico num dieléctrico

Uma esfera de material isolante, de constante dieléctrica ε e raio R, está unifor-memente carregada em volume, com uma densidade de carga ρ, encontrando-seimersa no vácuo.

• Campo eléctricoPara determinar o campo eléctrico no interior do material dieléctrico vamosusar a lei de Gauss generalizada. Uma vez que o campo eléctrico produzidopela esfera é radial e o seu módulo só depende da distância ao seu centro,pode-se utilizar uma superfície de Gauss esférica de raio r. Nesta superfícieo vector ~D será sempre paralelo à normal e terá módulo constante.

r < R∮

S

~D · ~n dS =

ρdV

D4πr2 = ρ4

3πr3

D =ρr

3

~E =ρr

3ε~ur

r > R∮

S

~D · ~n dS =

ρdV

D4πr2 = ρ4

3πR3

D =ρR3

3r2

~E =ρR3

3ε0r2~ur

• Potencial eléctricoConsideremos como ponto de referência o infinito já que neste caso o po-tencial se vai naturalmente anular naquele ponto.

para r>R:

φ =

∫ ∞

r

~E · d~ℓ =

∫ ∞

r

ρR3

3ε0r2dr =

ρR3

3ε0r

para r=R:

φ =ρR2

3ε0

para r<R:

φ =

∫ R

r

~E · d~ℓ +∫ ∞

R

~E · d~ℓ =

∫ R

r

ρr

3εdr +

∫ ∞

R

ρR3

3ε0

dr

r2

=ρr

(R2 − r2

2

)

+ρR2

3ε0

• Cargas de polarizaçãoem volume:

ρpol = −~∇ · ~P = −~∇ ·(

ε0χe~E)

F.Barão, L.F.Mendes Electromagnetismo e Óptica (MEEC-IST) 27

Page 34: Colecção de Problemas de Electromagnetismo e Ópticafcomp.ist.utl.pt/EO.web/Book.29_01_2014.pdf · Preâmbulo Esta colectânea de problemas resulta da experiência dos autores na

Capítulo 1

= −ε0χe

(1

r2

∂r

(

r2ρr

))

= −(ε − ε0

ε

)

ρ

Note que em alternativa poderia ter chegado a este resultado de uma formamais simples, tendo em conta que ~D = ε ~E:

ρpol = −~∇ · ~P = −~∇ ·(

ε0χe~E)

= −~∇ ·(

ε0χe

~D

ε

)

= −ε0

εχe

~∇ · ~D = −(ε − ε0

ε

)

ρ

em superfície:

σpol = ~P · ~next = ε0χe~E · ~next = ε0χe

ρR

3ε=

(

1 − ε0

ε

)ρR

3

Nota: 43πR3ρpol + 4πR2σpol = 0

F.Barão, L.F.Mendes Electromagnetismo e Óptica (MEEC-IST) 28

Page 35: Colecção de Problemas de Electromagnetismo e Ópticafcomp.ist.utl.pt/EO.web/Book.29_01_2014.pdf · Preâmbulo Esta colectânea de problemas resulta da experiência dos autores na

Capítulo 1

1.2.6 Condensador plano

Dois planos paralelos condutores com com uma área A muito maior do que adistância d que os separa, estão ligados a uma bateria de diferença de potencialV .

• O campo eléctrico entre as placasA ligação à bateria induz transferência de carga de um plano para outro,ficando os planos com uma distribuição uniforme de carga (σ). Na aproxi-mação do plano infinito, o campo eléctrico criado pelo plano não dependeda distância ao plano. Desta forma e tendo em conta que uma das placas secarrega com carga positiva (+Q) e a outra com carga negativa (−Q), naregião entre os planos o campo eléctrico será uniforme. Fora desta região,o campo eléctrico anula-se.

V =

∫ d

0

~E · d~ℓ =

∫ d

0

~Edℓ = E d

E =V

d

• A carga armazenada pelas placasO campo eléctrico existente entre as placas condutoras é função da cargaexistente. Uma vez que o campo eléctrico entre as placas é conhecido,pode-se utilizar a lei de Gauss para determinar a densidade de carga nasplacas. Para superfície de Gauss considera-se uma superfície cilíndrica comuma das “tampas” situada no interior da armadura (onde o campo eléctricoé nulo) e a outra na região entre as armaduras (ver figura). O fluxo docampo eléctrico que atravessa a superfície de Gauss é somente aquele queatravessa a “tampa” situada na região entre as armaduras.

S

~E · ~n dS =Qint

ε0

ES =σS

ε0

E =σ

ε0

Como Q = σA, tem-se:

E =Q

ε0 A= V d

Q = Aε0V

d

• A capacidade do condensadorA capacidade do condensador vem:

C =Q

V=

A ε0

V

V

d= ε0

A

d

• A energia armazenada pelo condensadorA energia armazenada pelo condensador corresponde ao trabalho realizadopara o carregar com uma carga Q. De acordo com a equação 1.34, tem-se:

+Q −Qd

E

V

n

n

F.Barão, L.F.Mendes Electromagnetismo e Óptica (MEEC-IST) 29

Page 36: Colecção de Problemas de Electromagnetismo e Ópticafcomp.ist.utl.pt/EO.web/Book.29_01_2014.pdf · Preâmbulo Esta colectânea de problemas resulta da experiência dos autores na

Capítulo 1

UE =1

2C V 2 =

1

2ε0

A

dV 2

F.Barão, L.F.Mendes Electromagnetismo e Óptica (MEEC-IST) 30

Page 37: Colecção de Problemas de Electromagnetismo e Ópticafcomp.ist.utl.pt/EO.web/Book.29_01_2014.pdf · Preâmbulo Esta colectânea de problemas resulta da experiência dos autores na

Capítulo 1

1.2.7 Associação de condensadores em série

Dois condensadores (1 e 2) de faces paralelas associados em série encomtram-se ligados a uma diferença de potencial V . Ambos os condensadores possuemarmaduras de área A, espessura a e distanciadas entre si de d. O espaço entre asarmaduras está preenchido por ar num dos condensadores e por material dieléctricode permitividade eléctrica ε, no outro.

• Campo eléctricoSubmetidas à diferença de potencial V , as placas extremas carregam-secom uma carga Q (+Q de um lado e −Q do outro). A placa intermédiapossui uma carga total nula, existindo no entanto uma carga negativa−Q de um lado e uma carga positiva +Q no outro. A carga existentena placa intermédia situa-se à superfície (condutor em equilíbrio!) epode ser determinada a partir da lei de Gauss. Para tal fazemos passaruma superfície cilíndrica com as “tampas” colocadas no interior deduas das armaduras. Dado que o campo eléctrico é nulo no interiordas armaduras condutoras e que o campo eléctrico existente entre asarmaduras é paralelo à superfície lateral do cilindro, tem-se um fluxodo campo eléctrico nulo. Ou seja, a lei de Gauss permite concluirque a carga total contida no cilindro é nula. Isto significa que cargasopostas (+Q e −Q) se encontram nas superfícies interiores das armaduras.

O determinação do campo eléctrico no interior dos condensadores(entre as armaduras) pode fazer-se com o auxílio da lei de Gaussgeneralizada, utilizando a metodologia empregue no exemplo 1.2.6:

S

~D · ~n dS = Qliv

D =Q

A

O vector Deslocamento eléctrico ( ~D) é igual em ambos os condensadores,pois só depende das cargas livres existentes nas armaduras:

D1 = D2

O campo eléctrico em cada um dos condensadores obtém-se como:

E1 =Q

ε0 A

E2 =Q

ε A=

ε0

εE1

A diferença de potencial pode ser escrita como:

V =

∫ d

0

~E1 · d~ℓ+∫ d+a

d

~0 · d~ℓ+∫ 2d+a

d+a

~E2 · d~ℓ

=

∫ d

0

E1 dℓ+

∫ 2d+a

d+a

ε0

εE1

= E1 d

(

1 +ε0

ε

)

ε0

2

1

− − − − − − − − − −−+ + + + + + + + +

ED P ε

d

d

a

+

−+

F.Barão, L.F.Mendes Electromagnetismo e Óptica (MEEC-IST) 31

Page 38: Colecção de Problemas de Electromagnetismo e Ópticafcomp.ist.utl.pt/EO.web/Book.29_01_2014.pdf · Preâmbulo Esta colectânea de problemas resulta da experiência dos autores na

Capítulo 1

Pelo que,

E1 =V

d(1 + ε0

ε

)

E2 =V

d(

1 + εε0

)

• Carga eléctrica armazenadaA carga eléctrica armazenada nas placas condutoras obtém-se como:

Q = A ε0 E1

Q =Ad

V1ε0

+ 1ε

• Capacidade do condensadorA capacidade do condensador calcula-se a partir da carga armazenada Q eda diferença de potencial a que está sujeito V , como sendo:

C =Q

V=

Ad

1ε0

+ 1ε

Note-se que poderíamos ter obtido a capacidade do condensador, a partirda associação em série de duas capacidades C1 e C2. Relembrando oresultado obtido no exemplo 1.2.6, tem-se:

1

C=

d

A ε0+

d

A ε=

1

C1

+1

C2

F.Barão, L.F.Mendes Electromagnetismo e Óptica (MEEC-IST) 32

Page 39: Colecção de Problemas de Electromagnetismo e Ópticafcomp.ist.utl.pt/EO.web/Book.29_01_2014.pdf · Preâmbulo Esta colectânea de problemas resulta da experiência dos autores na

Capítulo 1

1.2.8 Condensador cilíndrico

Um cabo coaxial é constituído por um condutor cilíndrico interior de raio R1, euma película exterior condutora de raio R2, existindo no espaço que separa oscondutores um material dieléctrico de permitividade ε. O cabo está ligado a umafonte de tensão V . Consideremos o comprimento do cabo, ℓ, muito maior queR2.

• Campo eléctrico no espaço entre os condutoresTal como se viu no exemplo 1.2.4 pode-se aplicar a este problema a lei deGauss para uma geometria cilíndrica. Neste caso iremos aplicar a lei deGauss generalizada já que existe um dieléctrico a separar os condutores.Como superfície de Gauss utilizamos uma superfície cilíndrica que envolveum pedaço de condutor interior de comprimento L. As linhas de campodo vector deslocamento eléctrico serão radiais em coordenadas cilíndricas,sendo o campo paralelo às normais na superfície lateral do cilindro e per-pendicular às normais nas “tampas”. Na superfície lateral do cilindro omódulo de ~D é constante.

Neste problema, como não se conhece a carga que está nos condutoresvamos assumir que existe uma densidade de carga por unidade de com-primento do condutor interior λ. A lei de Gauss generalizada escreve-seentão:

S

~D · ~n dS = Qint

S1

~D · ~n dS +

S2

~D · ~n dS +

S3

~D · ~n dS = λ L

S1

0 dS +

S2

0 dS +

S3

D dS = λ L

D

S3

dS = λ L

D2πrL = λ L

D =λ

2πr

pelo que,

E =λ

2πεr

Agora que se conhece a expressão do módulo do campo eléctrico pode-serelacioná-lo com a diferença de potencial entre os condutores e calcular adensidade de carga:

V =

∫ R2

R1

~E · ~dl =∫ R2

R1

Edr =

∫ R2

R1

λ

2πεrdr

2πε[ln(r)]R2

R1=

λ

2πεln

(R2

R1

)

λ =2πεV

ln(

R2

R1

) (1.41)

F.Barão, L.F.Mendes Electromagnetismo e Óptica (MEEC-IST) 33

Page 40: Colecção de Problemas de Electromagnetismo e Ópticafcomp.ist.utl.pt/EO.web/Book.29_01_2014.pdf · Preâmbulo Esta colectânea de problemas resulta da experiência dos autores na

Capítulo 1

Substituindo na expressão do campo eléctrico obtém-se:

E =V

r ln(R2

R1)

• Capacidade do cabo coaxialA capacidade é dada por:

C =Q

V

Multiplicando pelo comprimento do cabo a equação 1.41, temos uma rela-ção entre a carga do cabo coaxial e a tensão aplicada:

Q = λℓ =2πεV ℓ

ln(R2

R1)

C =2πεℓ

ln(R2

R1)

• Energia potencial electrostática armazenada no cabo coaxial

Conhecida a capacidade do cabo, a energia armazenada por este vem:

UE =1

2CV 2 =

πεℓ

ln(R2

R1)V 2

A energia potencial armazenada pelo cabo poderia ter sido calculada, de-terminando a densidade de energia potencial electrostática associada aocampo eléctrico que existe dentro do cabo coaxial e integrando em todo ovolume onde existe campo (volume do cabo):

uE =1

2εE2 =

1

(

V

r ln(R2

R1)

)2

UE =

v

uEdv =

∫ ℓ

0

∫ 2π

0

∫ R2

R1

1

(

V

r ln(R2

R1)

)2

r dr dθ dz

=1

2

εV 2

ln2(R2

R1)

∫ ℓ

0

∫ 2π

0

∫ R2

R1

1

rdrdθdz

=1

2

εV 2

ln2(R2

R1)2πℓ ln(

R2

R1

)

=πεℓV 2

ln(R2

R1)

F.Barão, L.F.Mendes Electromagnetismo e Óptica (MEEC-IST) 34

Page 41: Colecção de Problemas de Electromagnetismo e Ópticafcomp.ist.utl.pt/EO.web/Book.29_01_2014.pdf · Preâmbulo Esta colectânea de problemas resulta da experiência dos autores na

Capítulo 1

• Pressão sobre a película condutora exteriorA força atractiva entre o condutor interior e a película exterior é radial peloque a força total é nula (o centro de massa, em caso de colapso, não semoveria). No entanto existe uma pressão que pode ser calculada como omódulo da força por unidade de área de película:

P =Fpel ícula

2πR2ℓ

A força sobre a película pode ser calculada utilizando a expressãoda energia potencial electrostática e o método do deslocamentovirtual: a força radial será igual à variação da energia potencialelectrostática do sistema quando a película sofre um deslocamentovirtual, no nosso caso, quando R2 sofre um acréscimo infinitesimal dr .

Observando a expressão disponível para realizar o cálculo verifica-mos que a tensão aparece explicitamente pelo que será mais simples fazeras contas considerando o sistema ligado à fonte e, portanto, não isolado(V = cte). Nesse caso,

Fpel ícula ≡ Fr = +dUE

dR2

=d

dR2

(

πεℓ

ln(R2

R1)V 2

)

= − πεℓV 2

R2 ln2(R2

R1)

P = − πεℓV 2

(2πR2l)(R2 ln2(R2

R1))

= − εV 2

2R22 ln

2(R2

R1)

A pressão é negativa pois a força é "para dentro".

F.Barão, L.F.Mendes Electromagnetismo e Óptica (MEEC-IST) 35

Page 42: Colecção de Problemas de Electromagnetismo e Ópticafcomp.ist.utl.pt/EO.web/Book.29_01_2014.pdf · Preâmbulo Esta colectânea de problemas resulta da experiência dos autores na

Capítulo 1

1.3 Exercícios Propostos

Campo eléctrico no vácuo

Exercício 1.1 : Dois pêndulos de comprimento ℓ, massam e carga Q, encontram-se suspensos num mesmo ponto.Considere que os pêndulos se encontram na sua posiçãode equilíbrio e que o ângulo que os fios fazem com avertical do lugar, θ, é muito pequeno. Determine:

a) as forças que actuam as duas massas.

b) a distância entre as duas massas.

Exercício 1.2 : Uma carga Q1 de −3 µC é colocadanum ponto de posição ~r1 = (1, 3, 0) cm num dadoreferencial e uma segunda carga Q2 de 5 µC é colocadana origem desse referencial.

a) Qual a força exercida pela carga Q1 sobre a cargaQ2?

b) Qual o ponto do espaço em que o campo eléctricocausado pela duas cargas é nulo? Existe maisalgum ponto nessas condições?Sugestão: comece por mudar de sistema de eixos.

c) Que valor de massa colocada à superfície da Terrasofreria uma força gravítica de módulo igual à daforça sofrida pela carga Q2?

Exercício 1.3 : Determine o trabalho necessário paratransportar uma carga eléctrica q desde um ponto A(0, yA) a um ponto B (xB, 0), na presença de umcampo eléctrico uniforme ~E = E0~ux (ver figura).

x

y

A

B

Exercício 1.4 : Dois protões estão separados de umadistância d = 4 fm, tal como é mostrado na figura.

xd

y

a) Qual é a direcção do campo eléctrico em qualquerponto do eixo yy?

b) Determine o potencial eléctrico num ponto situ-ado entre os protões no eixo xx.

c) Esboçe as linhas do campo eléctrico.

d) Determine o ponto de equilíbrio de um outro pro-tão que se traz para a vizinhança dos dois pro-tões, considerando que as três cargas estão con-finadas ao eixo xx. Trata-se de um ponto deequilíbrio estável ou instável?

Exercício 1.5 : Um dipolo eléctrico é definido por umconjunto de duas cargas eléctricas simétricas (+q e −q)separadas de uma distância d.

r

x

y

d

+ P(x,y)

a) Determine a expressão do potencial criado pelascargas em qualquer ponto do espaço.

b) Determine a expressão do campo eléctrico criadopelas duas cargas em qualquer ponto do espaço.

c) Particularize a expressão obtida na alínea b) paraos pontos situados ao longo do eixo xx e doeixo yy e obtenha as expressões válidas parax, y >> d.

d) Sabendo que o momento dipolar eléctrico de umadistribuição de N cargas qi é definido por~p =

∑Ni=1 ~riqi, sendo ~ri o vector posição da

F.Barão, L.F.Mendes Electromagnetismo e Óptica (MEEC-IST) 36

Page 43: Colecção de Problemas de Electromagnetismo e Ópticafcomp.ist.utl.pt/EO.web/Book.29_01_2014.pdf · Preâmbulo Esta colectânea de problemas resulta da experiência dos autores na

Capítulo 1

carga qi, calcule o momento dipolar eléctrico eescreva as equações obtidas na alínea c) em fun-ção de ~p.

e) Esboce as linhas do campo eléctrico e as equipo-tenciais.

Exercício 1.6 : Um quadripolo eléctrico é constituídopor dois dipolos eléctricos de igual momento dipolar esentidos opostos.

−−

+

+

+q

+q

−q−q

y

d

d

P

a) Calcule o momento dipolar do quadripolo.

b) Determine o campo eléctrico no ponto P situ-ado a uma distância x do centro do quadripolo(origem dos eixos).

c) Determine o campo eléctrico para pontos no eixodos xx muito afastados da origem (x >> d) .

Exercício 1.7 : Um aro circular de raio R encontra-se linearmente carregado com uma densidade de cargaλ(C.m−1).

a) Determine, a partir da lei de Coulomb, a expres-são do campo eléctrico num qualquer ponto darecta perpendicular ao plano definido pelo aro eque passa no seu centro.

b) Determine a expressão do potencial eléctrico φnum qualquer ponto da mesma recta.

c) Determine a expressão do campo eléctrico a par-tir do potencial calculado na alínea anterior.

Exercício 1.8 : Um disco de raio a encontra-se unifor-memente electrizado em superfície, com uma densidadede carga σ(C.m−2).

a) Determine a expressão do potencial num pontoqualquer do eixo perpendicular ao disco que passapelo seu centro.

b) Determine a expressão do campo eléctrico numponto qualquer do eixo perpendicular ao discoque passa pelo seu centro.

c) Utilizando o resultado da alínea anterior, deter-mine a expressão do campo eléctrico criado porum plano infinito uniformemente electrizado emsuperfície, com uma densidade de carga σ. Co-mente se poderia utilizar a mesma estratégia parao cálculo do potencial criado pelo plano infinito.

Exercício 1.9 : Um fio de comprimento 2a, encontra-se carregado com uma densidade de carga λ (C.m−1).Determine:

a

XP

Y

Xa

a) a expressão do campo eléctrico num ponto P auma distância y do fio e situado no eixo que odivide ao meio.

b) a expressão do campo eléctrico no caso de o fioser infinito.

Exercício 1.10 : Uma barra de comprimento a cm edensidade linear de carga λ C/m é colocada alinhadacom o eixo xx. Determine:

Y

XX

−aO

Px

a) a expressão do potencial eléctrico no ponto P.

b) a expressão do campo eléctrico no ponto P.

c) a expressão aproximada do campo eléctrico parapontos do semi-eixo positivo xx muito afasta-dos da barra (x >> a). Comente a expressãoobtida.

F.Barão, L.F.Mendes Electromagnetismo e Óptica (MEEC-IST) 37

Page 44: Colecção de Problemas de Electromagnetismo e Ópticafcomp.ist.utl.pt/EO.web/Book.29_01_2014.pdf · Preâmbulo Esta colectânea de problemas resulta da experiência dos autores na

Capítulo 1

Exercício 1.11 : Uma barra carregada de comprimentoa=3 cm e densidade linear de carga λ = 2 C/m écolocada alinhada com o eixo xx. A uma distânciad=4 cm e ao longo do mesmo eixo, é colocada umabarra isolante de comprimento ℓ=2 cm com duas cargaspontuais Q1 e Q2 nas extremidades.

Q1 Q2

l

Y

XO−a

d

a) Sabendo que Q2 = 1 µC, determine o valorde Q1 que permite à barra isolante permanecerimóvel.

b) O movimento da barra isolante poderia ser estu-dado considerando todas as forças aplicadas noseu centro de massa e toda a massa do sistemaaí concentrada. Atendendo ao resultado da alí-nea anterior, indique justificando se faz sentidodefinir um centro de cargas.

Exercício 1.12 : O campo eléctrico numa vasta regiãoda atmosfera terrestre é vertical e dirigido para baixo,sendo o seu valor 60 V.m−1 a 300 m de altitude e 100V.m−1 a 200 m. Determine a carga total existente numcubo de 100 m de lado, localizado entre 200 m e 300m de altitude. Despreze a curvatura da Terra.

Exercício 1.13 : Considere um plano infinito carregadouniformemente com uma densidade de carga σ.

a) Determine, usando a lei de Gauss, o campo eléc-trico a uma distância r do plano.

b) Considere agora o plano infinito inicial com umburaco circular de raio R (ver figura). Calculeo campo eléctrico num ponto qualquer do eixodesse buraco (eixo zz).NOTA: recorra também ao resultado do exercício8.

Condutores em equilíbrio electrostático

Exercício 1.14 : Utilizando a lei de Gauss e a lei dasmalhas (

∮~E · ~dℓ = 0) verifique que junto à superfície

de um condutor se tem:

a) a componente tangencial do campo eléctrico (E‖)é nula.

b) a componente perpendicular do campo eléctricoé dada por: E⊥ = σ

ε0

Exercício 1.15 : Considere uma esfera condutora deraio a, carregada uniformemente em superfície com umadensidade de carga σ.

a) Obtenha a expressão do campo eléctrico nas di-ferentes regiões do espaço (r < a e r > a).

b) Calcule a energia necessária para trazer uma carga+q desde o infinito até ao centro da esfera.

Exercício 1.16 : Considere duas esferas condutoras deraios RA e RB e relativamente afastadas uma da ou-tra pelo que a influência recíproca dos campos pode serdesprezada. Cada uma das esferas tem uma carga Q.

a) Diga como está distribuída a carga nas esferascondutoras e calcule a sua densidade em funçãode Q e dos seus raios.

b) Calcule o campo eléctrico junto à superfície dasduas esferas em função de Q e dos seus raios.

c) Suponha que se ligavam as esferas através de umfio condutor. Calcule a carga que existiria emcada esfera após se atingir a situação de equi-líbrio, QA e QB, em função de Q e dos seusraios.

Exercício 1.17 : Um condutor esférico oco de raiosinterior e exterior respectivamente R2 e R3, tem noseu interior um outro condutor esférico maciço de raioR1. As duas esferas estão inicialmente ligadas por umfio condutor. Coloca-se uma carga positiva Q na esferaexterior e, passado algum tempo, retira-se o fio condutorque unia as duas esferas.

F.Barão, L.F.Mendes Electromagnetismo e Óptica (MEEC-IST) 38

Page 45: Colecção de Problemas de Electromagnetismo e Ópticafcomp.ist.utl.pt/EO.web/Book.29_01_2014.pdf · Preâmbulo Esta colectânea de problemas resulta da experiência dos autores na

Capítulo 1

x

R3

R2

R1

a) Qual a diferença de potencial entre as duas esfe-ras?

b) Qual a distribuição de carga nas duas esferas apósse ter retirado o fio? Justifique.

c) O resultado da alínea anterior modificava-se seinicialmente se tivesse carregado a esfera interiorem vez da exterior? Justifique.

Exercício 1.18 : O campo eléctrico máximo que o ar su-porta sem se ionizar e sem que haja disrupção é 3 × 106

V.m−1. Determine o raio mínimo de uma esfera metá-lica que possa estar ao potencial de 1 milhão de Voltssem que haja disrupção do ar.

Exercício 1.19 : Um cabo coaxial de comprimentomuito grande quando comparado com a sua espessura(infinito), é constituído por um condutor cilíndrico deraio R1 e por uma malha cilíndrica condutora, de raiosinterno e externo respectivamente R2 e R3 (R3 >R2 > R1). O cabo foi ligado a uma bateria quecarregou o cabo interior com uma densidade de cargaλ (C.m−1), sendo a malha ligada à terra.

a) Determine o campo eléctrico nas várias regiõesdo espaço. Esboce o gráfico de E(r).

b) Calcule a diferença de potencial entre os cabos edesenhe as linhas equipotenciais.

c) Calcule a diferença de potencial entre o condutorexterior do cabo e um ponto a uma distânciaradial R4 do centro do cabo (R4 > R3)

Exercício 1.20 : O gerador de Van der Graaf foi inven-tado para produzir um potencial eléctrico elevado e destaforma funcionar como acelerador de partículas (electros-tático). O gerador é formado por uma coroa esféricametálica que é carregada a partir do seu interior. A co-roa esférica possui raios interno e externo R1 = 0, 25m e R2 = 0, 30 m. Uma correia de borracha é accio-nada por um motor e transporta cargas até ao interior

da coroa esférica onde são recolhidas por um fio con-dutor que liga a correia à coroa. Considere que, apesarda abertura na parte inferior do gerador para passar acorreia, é uma boa aproximação considerar que no pro-blema há simetria esférica, desde que não se esteja juntoà abertura.

a) Determine o campo eléctrico num ponto A, auma distância r > R2 do centro da coroa es-férica condutora, em função da carga depositadana coroa, Q.

b) Sabendo que o campo eléctrico máximo que o arsuporta sem que haja disrupção é Ear = 3 × 106

V.m−1, calcule o potencial máximo a que podeficar a coroa metálica.

c) Calcule a diferença de potencial entre os pontosB e C e explique resumidamente porque razãoas cargas se dirigem da correia para o exterior dacoroa esférica.

Exercício 1.21 : Uma carga pontual q encontra-se auma distância d de um condutor semi-infinito, que seencontra ligado à Terra (φ = 0) e portanto todos ospontos do plano (x = 0, y, z) estão a um potencialnulo. A resolução deste problema pode ser feita com odenominado método das imagens que consiste em subs-tituir o condutor por uma carga imagem, q

, colocadasimetricamente em relação à superfície do condutor. Osdois problemas serão equivalentes do ponto de vista dopotencial criado no exterior do condutor desde que opotencial na superfície do condutor seja tambémnulo.

F.Barão, L.F.Mendes Electromagnetismo e Óptica (MEEC-IST) 39

Page 46: Colecção de Problemas de Electromagnetismo e Ópticafcomp.ist.utl.pt/EO.web/Book.29_01_2014.pdf · Preâmbulo Esta colectânea de problemas resulta da experiência dos autores na

Capítulo 1

a) Determine a carga imagem, q′

.

b) Determine o potencial criado pela carga q e pelocondutor num ponto P fora do condutor, na suaproximidade φP (x < d, y, 0).

c) Determine o campo eléctrico no mesmo ponto,~EP . Particularize para x = 0, ou seja, para asuperfície do condutor.

d) Determine a densidade de carga na superfície docondutor, σ(y), e esboce as linhas de campoeléctrico.

e) Calcule a força exercida pelo condutor sobre acarga q.

Exercício 1.22 : Uma nuvem num dia de tempestadepode ser representada por um dipolo eléctrico com umacarga de ±10 C. A parte inferior da nuvem está a umaaltura de h1 = 5 km acima do solo e a parte superiora h2 = 8 km acima do solo. O solo está molhadoe pode-se considerar um bom condutor. Utilizando ométodo descrito no problema 21, determine:

a) a expressão do potencial eléctricoφ(x, y, z), na região 0 < z < h1;

b) o campo eléctrico na vizinhança da Terra;

c) a densidade de carga induzida na Terra.

Exercício 1.23 : Utilizando a lei de Gauss generalizadae a lei das malhas (

Γ~E · d~ℓ = 0) verifique que junto

à superfície de separação entre dois materiais de cons-tantes dieléctricas ε1 e ε2:

a) a componente do campo paralela à superfície écontínua: E1‖ = E2‖

b) componente do campo perpendicular à superfí-cie não é contínua e, se existir uma densidade decarga σ, verifica-se a relação: ε1E1⊥−ε2E2⊥ =σ.

Campo eléctrico nos materiais dieléctricos

Exercício 1.24 : Uma molécula de água é compostapor dois grupos OH. Num grupo OH um átomo deHidrogénio (H) liga-se a um átomo de Oxigénio (O)comportando-se o conjunto como um dipolo eléctrico,com uma carga +q no hidrogénio e uma carga -q nooxigénio, em que q = 0, 316 e.

1040

H H

O

a) Calcule o momento dipolar do conjunto OH for-mado pelos átomos, sabendo que a distância en-tre os dois núcleos é d=0,97Å.

b) Uma molécula de água é constituída por dois gru-pos OH fazendo um ângulo de 104. Calcule omomento dipolar da molécula da água.

c) Determine o campo eléctrico criado pela molé-cula para distâncias (r) muito maiores que as dis-tâncias internucleares e na direcção do momentodipolar. Calcule o seu valor a 10 Å de distância.

Exercício 1.25 : Uma esfera condutora de raio R1 érevestida com material isolante de constante dieléctricarelativa εr = 5, de forma a obter-se uma esfera de raioR2. Durante o processo de fabrico a superfície interiordo isolante adquiriu uma carga electrostática Q.

a) Determine o campo ~D em função da distânciaao centro da esfera, r.

F.Barão, L.F.Mendes Electromagnetismo e Óptica (MEEC-IST) 40

Page 47: Colecção de Problemas de Electromagnetismo e Ópticafcomp.ist.utl.pt/EO.web/Book.29_01_2014.pdf · Preâmbulo Esta colectânea de problemas resulta da experiência dos autores na

Capítulo 1

b) Determine o campo ~E em função da distância aocentro da esfera, r.

c) Represente graficamente ~D e ~E.

d) Determine as cargas de polarização nas superfí-cies do isolante.

Exercício 1.26 : Uma esfera de material isolante deconstante dieléctrica ε e raio a está uniformemente car-regada em volume, com uma densidade de carga ρ eimersa no vácuo.

a) Determine, explicando detalhadamente todos oscálculos efectuados, o campo eléctrico ~E dentroe fora da esfera.

b) Determine a expressão do potencial eléctrico den-tro e fora da esfera.

c) Calcule as cargas de polarização (em volume esuperfície) existentes na esfera.

Exercício 1.27 : A permitividade eléctrica de um meioinfinito depende da distância radial (r) a um centro desimetria segundo a expressão ε = ε0(1 + a/r) coma > 0. Uma esfera condutora de raio R e carga Q écolocada naquele meio e centrada em r = 0. Deter-mine:

a) o campo eléctrico em função de r;

b) o potencial eléctrico em função de r;

c) o vector de polarização, ~P , em função de r;

d) a densidade volúmica de carga de polarizaçãoexistente no dieléctrico;

e) a densidade de carga superficial de polarizaçãono dieléctrico;

f) a carga total de polarização existente no dieléc-trico.

Condensadores

Exercício 1.28 : Um condensador plano tem armadu-ras quadradas de lado ℓ separadas de uma distância d,sendo ℓ >> d. No seu interior existe um dieléctricolinear e não homogéneo com uma constante dieléctricarelativa εr = 1+ay , sendo a uma constante positiva.Determine:

a) o campo eléctrico dentro do condensador, ~E, su-pondo que o condensador está ligado a uma bate-ria de tensão V (armadura positiva em y = 0);

b) a capacidade do condensador;

c) as densidades de carga de polarização em funçãoda tensão da bateria, V .

Exercício 1.29 : Considere um condensador esféricoconstituído por duas superfícies condutoras concêntri-cas. O condutor interior tem um raio R1 e condutorexterior com a forma de uma coroa esférica, tem raiosR2 e R3. Antes de se colocar o condutor exterior, quese encontra neutro, carregou-se o condutor interior comuma carga Q.

a) Determine a capacidade do condensador.

b) Determine o potencial do condutor exterior emrelação à terra.

c) Admita agora que se liga o condutor exterior àterra.

c.1) Determine a carga total existente no con-dutor externo e o campo eléctrico na regiãoexterior do dispositivo.

c.2) A capacidade do condensador altera-se? jus-tifique.

Exercício 1.30 : Os iões no interior e no exterior deum neurónio estão separados por uma membrana planade 10−8 m de espessura, que se comporta como umisolante com uma permitividade eléctrica ε = 8ε0.

a) Qual é a capacidade de 1 cm2 desse neurónio?

b) Qual a capacidade de 1 cm2 de neurónio no casode a membrana ter uma permitividade eléctricaigual à do ar.

c) Sabendo que o campo eléctrico devido aos iõesque se acumulam à superfície da membrana neu-ronal é da ordem 106 N/C, calcule a diferençade potencial a que está sujeito o neurónio.

F.Barão, L.F.Mendes Electromagnetismo e Óptica (MEEC-IST) 41

Page 48: Colecção de Problemas de Electromagnetismo e Ópticafcomp.ist.utl.pt/EO.web/Book.29_01_2014.pdf · Preâmbulo Esta colectânea de problemas resulta da experiência dos autores na

Capítulo 1

d) Determine a carga por unidade de superfície damembrana neuronal.

Exercício 1.31 : Um condensador de faces paralelasde área A e separadas de uma distância d (d << A)encontra-se carregado com uma carga eléctrica Q. Doismateriais dieléctricos de permitividades eléctricas ε1 eε2 são colocados entre as placas condutoras, de acordocom a figura.

ε1 ε2

+Q −Q

d/3 2/3d

a) Determine os vectores deslocamento eléctrico, ~D,e campo eléctrico, ~E, no interior dos dois dieléc-tricos do condensador.

b) Determine a capacidade do condensador.

c) Determine a densidade de carga de polarizaçãonas superfícies dos dieléctricos.

Exercício 1.32 : Um condensador plano é constituídopor duas armaduras paralelas de lado ℓ separadas de umadistância d. O espaço entre as placas está preenchidopor dois dieléctricos lineares e homogéneos de permitivi-dades ε1 e ε2, de acordo com a figura. O condensadorestá ligado a uma fonte de tensão cuja diferença de po-tencial aplicada é V .

ε2

ε1

l/2

d

V

a) Determine o campo eléctrico no espaço entre asarmaduras.

b) Determine a distribuição de carga na superfíciedas armaduras.

c) Determine a densidade de carga de polarizaçãona superfície dos dieléctricos.

d) Determine a capacidade do condensador.

Exercício 1.33 : Um condensador esférico é constituídopor um condutor de raio R1 e uma cavidade esféricacondutora de raio interno R2 e externo R3. O espaçoentre as armaduras metálicas está preenchido por doisdieléctricos de permitividade ε1 e ε2 (ε1 > ε2), cuja su-perfície esférica de separação possui raio Rd. Suponhaque a armadura interna do condensador foi carregadainicialmente com uma carga +Q.

ε1

ε2

R1

R3

R2Rd

a) Determine o campo eléctrico existente em todasas regiões do espaço e esboce num gráfico a suamagnitude em função de r.

b) Determine o potencial eléctrico existente nas vá-rias regiões e faça a sua representação gráfica.

c) Determine a capacidade do condensador.

d) Calcule o vector polarização nas várias regiões.

e) Identifique as regiões onde existe carga de po-larização e determine as densidades de carga depolarização.

Exercício 1.34 : Um cabo coaxial é constituído porum condutor cilíndrico interior de raio R1, e uma coroacilíndrica condutora de raios R2 e R3, existindo no es-paço que separa os condutores um material dieléctricode permitividade ε. Consideremos o comprimento docabo , ℓ, muito maior que R3.

a) Determine a capacidade do cabo por unidade decomprimento.

b) Determine as distribuições de carga de polariza-ção por unidade de comprimento no caso de ligaro cabo a uma fonte de tensão V.

F.Barão, L.F.Mendes Electromagnetismo e Óptica (MEEC-IST) 42

Page 49: Colecção de Problemas de Electromagnetismo e Ópticafcomp.ist.utl.pt/EO.web/Book.29_01_2014.pdf · Preâmbulo Esta colectânea de problemas resulta da experiência dos autores na

Capítulo 1

Energia electrostática e Forças

Exercício 1.35 : Considere uma gota de chuva de formaesférica, com um raio R = 2 mm e uma carga Q = 10−9

C uniformemente distribuída pela sua superfície.

a) Calcule o potencial eléctrico a que se encontra agota em relação ao infinito e a sua energia po-tencial electrostática.

b) Suponha que em determinado momento a gota sedivide em duas gotas iguais, igualmente esféricase que estas se afastam muito. Averigue se estanova situação corresponde a um ganho ou umaperda de energia electrostática.

Exercício 1.36 : Duas cargas q1 e q2 são colocadas res-pectivamente em dois pontos A e B que estão separadospor uma distância d.

a) Determine o potencial eléctrico nos pontos A eB, assumindo o potencial nulo no infinito.

b) Determine a energia potencial electrostática dosistema de duas cargas.

c) Utilizando o resultado da alínea b) determine aforça eléctrica que a carga q1 exerce sobre a cargaq2.

d) Diga, justificando a sua resposta, no caso de ter-mos q1 = −q2, como poderíamos adicionar umaterceira carga q3 ao sistema sem realizar traba-lho.

Exercício 1.37 : Os ossos humanos são piezoeléctri-cos, ou seja, quando sujeitos a uma pressão produzemuma diferença de potencial. Esta diferença de potencialé fundamental no processo de fixação do cálcio. Porexemplo, para não descalcificarem quando estão em ór-bita, os astronautas fazem exercício físico. Pessoas comossos partidos não os podem exercitar e uma terapia uti-lizada para promover a fixação do cálcio nestas situaçõesé a aplicação de uma diferença de potencial exterior.Considere a aplicação de uma diferença de potencial aum braço, de acordo com a figura. Embora a aproxima-ção só seja válida na zona central dos eléctrodos, paraefeitos deste problema vamos considerar a aproximaçãodo condensador de placas infinitas. Os eléctrodos estãoisolados mas a espessura do isolante pode ser despre-zada.

a) Sabendo que a tensão aplicada aos eléctrodos é45 V, calcule o campo eléctrico no interior doosso.

b) Sabendo que a área dos eléctrodos é 45cm2, cal-cule a capacidade do sistema.

c) A densidade de energia eléctrica é maior no ossoou nos tecidos? Justifique.

Exercício 1.38 : Uma placa condutora de lado ℓ é car-regada com uma carga Q.

a) Determine o campo eléctrico num ponto a umadistância d muito próximo da placa.Nota: Pode considerar a aproximação do planoinfinito.

b) Uma segunda placa condutora, também de ladoℓ e carregada com uma carga −Q, é colocada auma distância d da primeira, formando um con-densador de faces paralelas. Determine a forçaexercida sobre esta segunda placa.

Exercício 1.39 : Um condensador plano com armadu-ras quadradas de lado ℓ e distanciadas de d (d << ℓ),é ligado a uma fonte de tensão V . O espaço entreas armaduras do condensador encontra-se parcialmentepreenchido com uma material dieléctrico de permitivi-dade ε. O material dieléctrico pode mover-se segundoa direcção do eixo dos xx.

F.Barão, L.F.Mendes Electromagnetismo e Óptica (MEEC-IST) 43

Page 50: Colecção de Problemas de Electromagnetismo e Ópticafcomp.ist.utl.pt/EO.web/Book.29_01_2014.pdf · Preâmbulo Esta colectânea de problemas resulta da experiência dos autores na

Capítulo 1

a) Determine a capacidade do condensador em fun-ção da posição do dieléctrico.

b) Determine a energia armazenada pelo condensa-dor em função da posição do dieléctrico. Esbocea curva da energia em função da posição do die-léctrico.

c) Determine a força exercida sobre o dieléctrico.

Exercício 1.40 : Um condensador esférico é compostopor um condutor de raio R1 envolvido por uma películacondutora deforma esférica deraio R2. O espaço entreos condutores está preenchido por um material dieléc-trico de permitividade eléctrica ε. O condensador estáligado a uma fonte de tensão que carrega o condutorinterior com uma carga Q.

a) Calcule a energia armazenada no condensador.

b) Calcule a pressão exercida sobre a película exte-rior devido ao campo eléctrico.

F.Barão, L.F.Mendes Electromagnetismo e Óptica (MEEC-IST) 44

Page 51: Colecção de Problemas de Electromagnetismo e Ópticafcomp.ist.utl.pt/EO.web/Book.29_01_2014.pdf · Preâmbulo Esta colectânea de problemas resulta da experiência dos autores na

Capítulo 1

1.4 Soluções dos exercícios propostos

1.1 a) ~P = m~g

~Fe = 116πε0

Q2

ℓ2sen2θ~ux

~T + ~Fe + ~P = 0

b) sen3θcos θ

= 116πε0

1mg

(Qℓ)2

no caso de pequenos ângulos, x =(

Q2ℓ2πε0mg

)1/3

1.2 a) ~F = +42, 7(~ux + 3~uy) [N]

b) Ponto que dista 14 cm da origem segundo a di-recção ~r = ~ux + 3~uy

c) 13, 8 kg

1.3 W = −qE0xB

1.4 a) ~E = E~uy

b) φ(x) = q2πε0

d2/

(

(

d2

)2− x2

)

d) x = 0, equilíbrio estável

1.5 a) φP = q4πε0

[

x2 + (y − d/2)2]−1/2 −

[

x2 + (y + d/2)2]−1/2

b) ~E(x, y) = q4πε0

(

~r+

r3+

− ~r−

r3−

)

~r+ = x~ux + (y − d2)~uy

~r− = x~ux + (y + d2)~uy

c) ~E(x, 0) = − q4πε0

d(

x2+( d2 )

2)3/2 ~uy

~E(x >> d, 0) = − q4πε0

dx3 ~uy

~E(0, y) = q4πε0

(

1

(y−d2 )

2 − 1

(y+d2 )

2

)

~uy

~E(0, y >> d) = q2πε0

dy3 ~uy

d) ~p = q ~d~E(x >> d, 0) = − 1

4πε0

~px3

~E(0, y >> d) = 12πε0

~py3

1.6 a) ~p = ~0

b) ~E = q2πε0

1x2

[

1

[1+( dx)2]3/2 − 1

]

~ux

c) ~E − 34πε0

qd2

x4 ~ux

1.7 a) ~E = E~uz

E(z) = λ2ε0

R z (z2 + R2)−3/2

b) φ(z) = λ2ε0

R (z2 + R2)−1/2

c) ~E = −~∇ φ

1.8 a) φ(z) = σ2ε0

(

a2 + z2 − |z|)

b) Ez = −∂φ∂z

= σ2ε0

(

z|z| − z√

a2+z2

)

c) lima→∞

Ez = σ2ε0

z|z| ;

Não, porque o potencial da solução anteriormenteapresentada tem o ponto de referência no ∞.

1.9 a) ~E = λ2πε0

a

y√

a2+y2~uy

b) ~E = λ2πε0

1y~uy

1.10 a) φ = λ4πε0

ln x+ax

b) ~E = λa4πε0

1x2

(

11+a

x

)−→ex

c) ~E = λa4πε0

1x2

−→ex

1.11 a) Q1 = −Q2

(

dd+ℓ

) (

d+ad+a+li

)

Q1 ≃ −0.5 × 10−6 [C]

b) Não faz sentido.

1.12 Q = 3, 6 µC

1.13 a) ~E = σ2ε0

z|z|~uz

b) ~E = σ2ε0

z√a2+z2

~uz

1.15 a) r < a : ~E = ~0

r > a : ~E = σε0

(

ar

)2~ur

b) UE = qφ(r = a) = qσaε0

1.16 a) σA = Q

4πR2A

σB = Q

4πR2B

b) ~EA = Q4πε0

1R2

A

~urA

~EB = Q4πε0

1R2

B

~urB

c) QA = RAQRA+RB

QB = RBQRA+RB

1.17 a) ∆φ = 0

b) Na superfície exterior do condutor ôco.

c) Não.

1.18 R = 33 cm.

1.19 a) r < R1 : E = 0

R1 < r < R2 : ~E = λ2πε0r

~ur

R2 < r < R3 : E = 0r > R3 : E = 0

b) V = λ2πε0

ln R2R1

c) V = 0

1.20 a) ~EA = Q4πε0

1r2 ~ur

b) φ(R2) = 900 KV

c) φB − φC = 0; para r < R2 tem-se E = 0 (eportanto φ =constante) sendo o comportamentodo sistema é idêntico ao de um condutor esféricode raio R2.

1.21 a) q′

= −q

b) φ(x, y, z) = q4πε0

(

1√(d−x)2+y2

− 1√(d+x)2+y2

)

c) ~E = − q4πε0

[(

d−x

((d−x)2+y2)3/2 + d+x

((d+x)2+y2)3/2

)

~ux

−(

y

((d−x)2+y2)3/2 − y

((d+x)2+y2)3/2

)

~uy

]

Para x = 0, ~E(y) = qd2πε0

1(d2+y2)3/2 ~ux

d) σ(y) = ε0E(y)

e) ~F = − 14πε0

q2

(2d)2~ux

F.Barão, L.F.Mendes Electromagnetismo e Óptica (MEEC-IST) 45

Page 52: Colecção de Problemas de Electromagnetismo e Ópticafcomp.ist.utl.pt/EO.web/Book.29_01_2014.pdf · Preâmbulo Esta colectânea de problemas resulta da experiência dos autores na

Capítulo 1

1.22 a) φ(x, y, z) = Q4πε0

[

(

x2 + y2 + (h2 − z)2)−1/2 −

(

x2 + y2 + (h1 − z)2)−1/2

+(

x2 + y2 + (h1 + z)2)−1/2 −

(

x2 + y2 + (h2 + z)2)−1/2

]

b) ~E = Q2πε0

[

h1(

x2 + y2 + h21

)−3/2 −

h2(

x2 + y2 + h22

)−3/2]

~uz

c) σ = ε0E

1.24 a) p = 5 × 10−30 C.m

b) ~p = 6 × 10−30~uy C.m; vectorialmente, corres-ponde à soma dos momentos dipolares das duasligações OH.

c) ~E = 10−19

r3 V/m ; ~E (r = 10 A) = 108 V/m

1.25 a) D = 0 (r < R1)~D = Q

4πr2 ~ur (r > R1)

b) r < R1 : E = 0

R1 < r < R2 : ~E = Q4πεr2 ~ur

r > R2 : ~E = Q4πε0r2 ~ur

d) Qpol(R1) = −45Q

Qpol(R2) = +45Q

1.26 a) r < a : ~E = ρr3ε

~ur

r > a : ~E = ρa3

3ε0r2 ~ur

b) r > a : φ = ρa3

3ε0r

r = a : φ = ρa2

3ε0

r < a : φ = ρ3ε

(

a2−r2

2

)

+ ρa2

3ε0

c) σpol =(

1 − ε0ε

) ρa3

ρpol = −(

ε−ε0ε

)

ρ

1.27 a) r < R : E = 0

r > R : ~E = Q4πε0

1r(r+a)

~ur

b) r < R : φ = Q4πε0

1aln

(

R+aR

)

r > R : φ = Q4πε0

1aln

(

r+ar

)

c) ~P = Q4π

ar2(r+a)

~ur r > R

d) r > R : ρpol = Q4π

ar2(r+a)2

e) r = R : σpol = − Q4π

aR2(R+a)

f) Qpol = 0

1.28 a) ~E = V a(1+ay) ln (1+ad)

~uy

b) C = ℓ2aε0ln (1+ad)

c) ρpol = − a2ε0V(1+ay)2 ln (1+ad)

σpol(0) = 0

σpol(d) = a2ε0V d(1+ad) ln (1+ad)

1.29 a) C = 4πε0R1R2

R2−R1

b) φ = Q4πε0

1R3

c.1) Qext = −QE = 0

c.1) Não se altera.

1.30 a) C = 710 nF

b) C = 89 nF

c) V = 10−2 V

d) σ = 7 nC.cm−2

1.31 a) ~D = QA~ux

~E1 = QAε1

~ux

~E2 = QAε2

~ux

b) C = 3Ad

ε1ε22ε1+ε2

c) σpol(0) = −QA

ε1−ε0ε1

σpol(d) = +QA

ε2−ε0ε2

σpol(d/3) = +QA

(

ε1−ε0ε1

− ε2−ε0ε2

)

1.32 a) E1 = E2 = Vd

b) σ1 = ε1Vd

σ2 = ε2Vd

c)(

σ1pol

)−= −(ε1 − ε0)

Vd

(

σ2pol

)−= −(ε2 − ε0)

Vd

d) C = ℓ2

d

( ε12

+ ε22

)

C = C1 + C2

1.33 a) ~E = ~0, r < R1

~E = Q4πε1

1r2 ~ur, R1 < r < Rd

~E = Q4πε2

1r2 ~ur, Rd < r < R2

~E = ~0, R2 < r < R3~E = Q

4πε01r2 ~ur, r > R3

b) (r < R1) : φ = Q4π

[

1ε0

1R3

+ 1ε2

(

1Rd

− 1R2

)

+

1ε1

(

1R1

− 1Rd

)]

(R1 < r < Rd) : φ = Q4π

[

1ε0

1R3

+ 1ε2

(

1Rd

− 1R2

)

+

1ε1

(

1r− 1

Rd

)]

(Rd < r < R2) : φ = Q4π

[

1ε0

1R3

+ 1ε2

(

1r− 1

R2

)]

(R2 < r < R3) : φ = Q4π

1ε0

1R3

(r > R3) : φ = Q4π

1ε0

1r

c) C = 4π1ε1

(

1R1

− 1Rd

)

+ 1ε2

(

1Rd

− 1R2

)

d) ~P = (ε1 − ε0)~E, no dieléctrico 1;~P = (ε2 − ε0)~E, no dieléctrico 2.

e) σpol(r = R1) = −(

1 − ε0ε1

)

Q

4πR21

σpol(r = R−d ) = +

(

1 − ε0ε1

)

Q

4πR2d

σpol(r = R+d ) = −

(

1 − ε0ε2

)

Q

4πR2d

σpol(r = R2) = +(

1 − ε0ε2

)

Q

4πR22

1.34 a) C′

= 2πε

ln(

R2R1

)

b) λpol(R1) = − 2πV

ln(

R2R1

) (ε − ε0)

λpol(R2) = −λpol(R1)

1.35 a) φ = Q4πε0

1R

UE = 2, 2 µJ

F.Barão, L.F.Mendes Electromagnetismo e Óptica (MEEC-IST) 46

Page 53: Colecção de Problemas de Electromagnetismo e Ópticafcomp.ist.utl.pt/EO.web/Book.29_01_2014.pdf · Preâmbulo Esta colectânea de problemas resulta da experiência dos autores na

Capítulo 1

b) A uma perda.

1.36 a) φA = q24πε0

1d

φB = q14πε0

1d

b) UE = q1q24πε0

1d

c) ~F = q1q24πε0

1d2 ~ur

d) Trazendo a carga para um ponto da superfícieequipotencial φ = 0, constituída pelos pontosequidistantes das duas cargas.

1.37 a) E = 600 V/m.

b) C = 1 pF.

c) uE = σ2

4ε0, osso.

uE = σ2

8ε0, tecido.

1.38 a) E = Q2ℓ2ε0

b) F = Q2

2ℓ2ε0(atractiva)

1.39 a) C = ε0ℓd

(εrx + (ℓ − x))

C = ε0ℓd

(ℓ + x(εr − 1))C = C1 + C2

b) UE(x) = 12C(x)V 2

c) Fx = 12V 2 ℓ

dε0(εr − 1)

1.40 a) UE = 2πε R1R2R2−R1

V 2

b) P = − ε2V 2

(

R1R2

)21

(R2−R1)2(para dentro)

F.Barão, L.F.Mendes Electromagnetismo e Óptica (MEEC-IST) 47

Page 54: Colecção de Problemas de Electromagnetismo e Ópticafcomp.ist.utl.pt/EO.web/Book.29_01_2014.pdf · Preâmbulo Esta colectânea de problemas resulta da experiência dos autores na

Capítulo 1

F.Barão, L.F.Mendes Electromagnetismo e Óptica (MEEC-IST) 48

Page 55: Colecção de Problemas de Electromagnetismo e Ópticafcomp.ist.utl.pt/EO.web/Book.29_01_2014.pdf · Preâmbulo Esta colectânea de problemas resulta da experiência dos autores na

Capítulo 2

Capítulo 2

Corrente eléctrica

2.1 Introdução

Uma corrente eléctrica consiste no movimento ordenado de cargas eléctricas numdado meio. O número de cargas eléctricas que atravessam uma dada superfície porunidade de tempo, corresponde à intensidade de corrente eléctrica que atravessa essasuperfície:

I =dQ

dt[A] (2.1)

A quantidade de carga eléctrica que atravessa a superfície num intervalo de tempot, será dada por:

Q =

∫ t

0

I dt [C] (2.2)

Por convenção o sentido da corrente é definido pelo movimento das cargas positivas.Isto significa que num condutor, caso em que as cargas que se movem são negativas(electrões), o sentido da corrente é contrário ao do movimento das cargas.

Corrente eléctrica

A densidade de corrente eléctrica num dado ponto do meio material, depende dadensidade de carga portadora da corrente, ρ [C.m−3] e da velocidade associada àcarga, ~v:

~J = ρ ~v [A/m2] (2.3)

Se um material possuir uma densidade de portadores de carga N [m−3] e se cadaportador possuir uma carga q [C], a densidade de corrente eléctrica pode escrever-se:

~J = Nq ~v (2.4)

Densidade de correnteeléctrica

A corrente eléctrica que atravessa uma superfície S é o fluxo da densidade de correnteatravés dessa superfície:

I =

S

~J · ~n dS (2.5)

F.Barão, L.F.Mendes Electromagnetismo e Óptica (MEEC-IST) 49

Page 56: Colecção de Problemas de Electromagnetismo e Ópticafcomp.ist.utl.pt/EO.web/Book.29_01_2014.pdf · Preâmbulo Esta colectânea de problemas resulta da experiência dos autores na

Capítulo 2

Nos anos de 1825 - 1827 Georg Simon Ohm verificou experimentalmente que adensidade de corrente eléctrica que percorre um condutor é proporcional ao campoeléctrico aplicado, sendo a constante de proporcionalidade a condutividade do con-dutor, σc [Ω−1.m−1]:

~J = σc~E (2.6)

Esta lei é conhecida como lei de Ohm local, uma vez que é válida em cada ponto deum meio condutor, e pode também ser escrita em função do inverso da condutividade,a resistividade do condutor ρ = 1/σc [Ω.m]:

~E = ρ ~J (2.7)

A sua correspondente lei macroscópica, a conhecida lei de Ohm, relaciona as gran-dezas também macroscópicas V e I mas pressupõe o conhecimento da resistênciado condutor, R [Ω], cujo cálculo abordaremos mais à frente

V = R I (2.8)

Ainda que menos usual é também possível escrever esta lei com o inverso da resis-tência, a condutância, G [S]

I = G V (2.9)

Lei de Ohm

Para manter a corrente eléctrica estacionária o campo eléctrico tem de realizar tra-balho nos condutores reais (meios em que existe uma resistência à passagem dacorrente). Consideremos os portadores de carga existentes num elemento de volumedV . Sendo N a densidade destes portadores e q a sua carga individual, no elementodV existirá uma carga

dQ = Nq dV

e a força que o campo eléctrico exerce sobre essa carga será

d~F = dQ ~E = Nq ~E dV

relembrando que a potência de uma força pode ser obtida pelo seu produto internocom a velocidade do deslocamento, ~v, podemos escrever a potência dispendida pelaforça eléctrica sobre o volume dV :

dP = d~F · ~v = Nq ~v · ~E dV = ~J · ~E dV

ou, definindo a densidade de potência como a potência por unidade de volume,

p ≡ dP

dV= ~J · ~E = JE (2.10)

Esta é a densidade de potência despendida por efeito de Joule localmente e a potênciatotal despendida por efeito de Joule pode ser calculada em todo o volume de umcondutor somando a contribuição de todos os seus elementos de volume:

P =

V

p dV =

V

JE dV (2.11)

Lei de Joule

F.Barão, L.F.Mendes Electromagnetismo e Óptica (MEEC-IST) 50

Page 57: Colecção de Problemas de Electromagnetismo e Ópticafcomp.ist.utl.pt/EO.web/Book.29_01_2014.pdf · Preâmbulo Esta colectânea de problemas resulta da experiência dos autores na

Capítulo 2

O princípio físico de que a carga é uma grandeza que é conservada em todos osprocessos e transformações permite-nos encontrar um equação importante que ca-racteriza a densidade de corrente eléctrica. Considere-se um certo volume V quecontém no seu interior uma carga Q cuja densidade de carga é ρ. Se houver umfluxo de carga positivo que atravessa a superfície fronteira desse volume, S, issosignificará que a carga dentro do volume vai variar no tempo (diminuir):

S

~J · ~n dS = −dQ

dt∮

S

~J · ~n dS = − d

dt

V

ρ dV (2.12)

~∇ · ~J = −∂ρ

∂t(2.13)

As equações da conservação da carga no caso do regime estacionário, em que∂ρ/∂t = 0, apresentam uma forma mais simples:

S

~J · ~n dS = 0 , ~∇ · ~J = 0 (2.14)

Conservação da cargaeléctrica

A equação da conservação da carga em regime estacionário não é mais que a lei dosnós e descreve matematicamente o facto de a corrente que entra numa superfíciefechada (um nó) ser exactamente igual à corrente que de lá sai (fluxo total zero):

i

Ii = 0 (2.15)

A lei das malhas foi já encontrada na electrostática e mantém-se válida para acorrente estacionária (em rigor, é válida desde que não existam campos magnéticosvariáveis):

Γ

~E · d~ℓ = 0 , ~∇ × ~E = 0 (2.16)

Esta lei estabelece que num circuito fechado (uma malha) a soma de todas as tensõesserá igual a zero:

i

Vi = 0 (2.17)

Leis de kirchhoff

F.Barão, L.F.Mendes Electromagnetismo e Óptica (MEEC-IST) 51

Page 58: Colecção de Problemas de Electromagnetismo e Ópticafcomp.ist.utl.pt/EO.web/Book.29_01_2014.pdf · Preâmbulo Esta colectânea de problemas resulta da experiência dos autores na

Capítulo 2

Como no exterior de um condutor não há corrente elétrica, junto à sua superfície adensidade de corrente eléctrica tem de lhe ser paralela (a corrente não pode sair docondutor!):

~J = ~J‖ (2.18)

E, pela lei de Ohm, a densidade de corrente é paralela ao campo eléctrico pelo quetambém o campo eléctrico junto à superfície do condutor lhe é paralelo, ficando aslinhas de campo eléctrico encerradas no interior do condutor:

~E = ~E‖ (2.19)

Desta forma, os condutores comportam-se como caminhos para o campo eléctrico(para fornecer a tensão de uma bateria a uma lâmpada basta ligá-los com caboscondutores, não importa a sua geometria).

Campo eléctrico dentrodos condutores

O cálculo de uma resistência eléctrica, quando não se trata de um condutor de secçãoconstante, pode não ser simples. Como método geral pode afirmar-se o cálculo daresistência eléctrica de um condutor passa por calcular a corrente, I, que o percorreem função da tensão que lhe é aplicada, V . Depois de determinadas estas duasgrandezas, a resistência eléctrica é dada simplesmente por:

R =V

I(2.20)

Quando temos dois condutores de resistências R1 e R2 ligados em série a resistênciatotal do sistema é dada por

R = R1 + R2 (2.21)

Quando temos dois condutores de resistências R1 e R2 ligados em paralelo a resis-tência total do sistema é dada por:

1

R=

1

R1 + R2

(2.22)

Resistência eléctrica de umcondutor

Partindo da equação local da conservação da carga pode encontrar-se a equaçãodiferencial que rege o escoamento de carga num meio de condutividade não nula.Considere-se um meio de condutividade σc que tem no seu interior um elementode volume dV com uma quantidade infinitesimal de carga, de densidade ρ (dQ =ρ dV ). A equação local de conservação da carga relaciona o fluxo de carga atravésda superfície infinitesimal fechada que envolve o elemento de volume, S, com avariação no tempo da densidade de carga existente dentro desta:

~∇ · ~J +∂ρ

∂t= 0

Tendo em conta que a relação entre campo elétrico e densidade de corrente existenteno condutor obedece à lei de Ohm local, ~J = σc

~E, obtém-se:

σc~∇ · ~E +

∂ρ

∂t= 0

Escoamento de carga

F.Barão, L.F.Mendes Electromagnetismo e Óptica (MEEC-IST) 52

Page 59: Colecção de Problemas de Electromagnetismo e Ópticafcomp.ist.utl.pt/EO.web/Book.29_01_2014.pdf · Preâmbulo Esta colectânea de problemas resulta da experiência dos autores na

Capítulo 2

Uma vez que o fluxo do vector deslocamento eléctrico através da superfície infinite-simal S, se pode obter a partir da lei de Gauss generalizada, ~∇ · ~D = ρ, e que ocampo eléctrico se relaciona com o vector deslocamento eléctrico a partir da relaçãoconstitutiva do meio, ~D = ε ~E, pode-se escrever:

σc

ρ

ε+

∂ρ

∂t= 0

A solução desta equação diferencial é

ρ = ρ0e−σc

ε t (2.23)

A constante de tempo associada ao escoamento da carga dQ é então

τ =ε

σc

(2.24)

Esta constante de tempo é da ordem de 10−19s nos condutores (escoamento ins-tantâneo!), da ordem das décimas de segundo em papel, da ordem das dezenas deminutos em borrachas comuns e de milhares de anos em plásticos especiais como éo caso do PTFE (Teflon).

F.Barão, L.F.Mendes Electromagnetismo e Óptica (MEEC-IST) 53

Page 60: Colecção de Problemas de Electromagnetismo e Ópticafcomp.ist.utl.pt/EO.web/Book.29_01_2014.pdf · Preâmbulo Esta colectânea de problemas resulta da experiência dos autores na

Capítulo 2

2.2 Exercícios Resolvidos

2.2.1 Velocidade de deriva dos electrões

Num fio de cobre com uma secção S de 2 mm de diâmetro passa uma correnteeléctrica I = 16 A. Sabendo que a densidade do cobre é ρ = 8, 95 g.cm−3,que o seu número de massa é A = 63, 5 g.mole−1 e considerando que existe 1electrão livre por átomo, pretende-se calcular a velocidade de deriva dos electrões(a componente da velocidade dos electrões que, sendo ordenada, cria a correnteeléctrica)

• A densidade de corrente no condutorSendo o fio de cobre homogéneo, a corrente eléctrica está uniformementedistribuída e a densidade de corrente é constantes no seu interior. Podemosentão escrever:

I =

S

~J · ~n dS =

S

J dS = J S

J =I

S=

I

π R2=

16

π (1 × 10−3)2

= 5, 1 × 106 A.m−2

• A velocidade de deriva dos electrõesConhecida a densidade de corrente eléctrica, a velocidade dos electrõespode ser calculada se soubermos qual a densidade de portadores e a cargade cada portador da corrente eléctrica. A densidade de portadores de cargaé

N =ρ

ANA × 1 =

8, 95 × 103

63, 5 × 10−36, 022× 1023

= 8, 5 × 1028 electrões.m−3

A velocidade dos electrões será então,

J = Nq v

v =J

Nq=

5, 1 × 106

8, 5 × 1028 × 1, 6 × 10−19

= 0, 38 mm.s−1 (!)

F.Barão, L.F.Mendes Electromagnetismo e Óptica (MEEC-IST) 54

Page 61: Colecção de Problemas de Electromagnetismo e Ópticafcomp.ist.utl.pt/EO.web/Book.29_01_2014.pdf · Preâmbulo Esta colectânea de problemas resulta da experiência dos autores na

Capítulo 2

2.2.2 Resistência de um condutor cilíndrico

Consideremos o condutor cilíndrico de secção S, comprimento L e condu-tividade σc, representado na figura. O condutor está ligado a uma fonte detensão V que lhe impõe uma corrente estacionária I.

• Campo eléctrico no interior do condutorSe o material condutor for homogéneo, de condutividade constante, a cor-rente eléctrica está uniformemente distribuída e a densidade de corrente, talcomo o campo eléctrico (lei de Ohm), são constantes. As linhas de camposão então paralelas ao eixo longitudinal do condutor e têm a direcção dacorrente eléctrica (~uz) Podemos então escrever:

V =

∫ L

0

~E · ~dℓ =

∫ L

0

E dz = E

∫ L

0

dz = E L

• Corrente eléctricaA corrente eléctrica que percorre o condutor é o fluxo da densidade decorrente através de uma secção do condutor. Sendo a normal à secção docondutor paralela à densidade de corrente temos:

I =

S

~J · ~n dS =

S

J dS = J

S

dS = J S

• Resistência eléctricaA resistência eléctrica do condutor pode agora ser encontrada utilizando alei de Ohm:

R =V

I=

E L

J S=

1

σc

L

S

F.Barão, L.F.Mendes Electromagnetismo e Óptica (MEEC-IST) 55

Page 62: Colecção de Problemas de Electromagnetismo e Ópticafcomp.ist.utl.pt/EO.web/Book.29_01_2014.pdf · Preâmbulo Esta colectânea de problemas resulta da experiência dos autores na

Capítulo 2

2.2.3 Adição de resistências em série

Consideremos a associação de dois condutores cilíndricos de secção S, comprimen-tos L1 e L2 e condutividades σc1 e σc2, representada na figura. Os condutoresestão ligados a uma fonte de tensão V que lhes impõe uma corrente estacionáriaI.

• Relação entre as densidade de corrente nos condutoresSe os materiais condutores forem homogéneos, de condutividade constante,a corrente eléctrica está uniformemente distribuída. Por outro lado, a cor-rente que atravessa os condutores é igual. Deste modo, e como os condu-tores têm secção igual, as densidades de corrente são idênticas e constantesnos dois condutores. As linhas de campo são paralelas ao eixo longitudinaldo condutor e têm a direcção da corrente eléctrica (~uz). Podemos entãoescrever:

~J1 = ~J2 = ~J = J~uz

• Relação entre os campos eléctricos nos dois condutoresConhecida a densidade de corrente, a relação entre os campos eléctricospode ser obtida pela lei de Ohm:

~J = σc1~E1 = σc2

~E2 ⇔ E1

E2

=σc2

σc1

• Corrente eléctricaA corrente eléctrica que percorre o condutor é o fluxo da densidade decorrente através de uma secção do condutor. Sendo a normal à secção docondutor paralela à densidade de corrente temos:

I =

S

~J · ~n dS =

S

J dS = J

S

dS = J S

a corrente pode ser também expressa em função dos campo eléctricos:

I = σc1 E1 S

I = σc2 E2 S

• Relação entre os campos eléctricos e a tensão aplicadaA tensão aplicada ao sistema pode ser relacionada com os dois camposeléctricos existentes ou mesmo com cada um deles:

V =

∫ L1

0

~E1 · ~dℓ+

∫ L1+L2

L1

~E2 · ~dℓ

=

∫ L1

0

E1 dz +

∫ L1+L2

L1

E2 dz

= E1 L1 + E2 L2 = E1 L1 + E1

σc1

σc2

L2

= E1 (L1 +σc1

σc2

L2) = E2

σc2

σc1

L1 + E2 L2

= E2 (σc2

σc1

L1 + L2)

F.Barão, L.F.Mendes Electromagnetismo e Óptica (MEEC-IST) 56

Page 63: Colecção de Problemas de Electromagnetismo e Ópticafcomp.ist.utl.pt/EO.web/Book.29_01_2014.pdf · Preâmbulo Esta colectânea de problemas resulta da experiência dos autores na

Capítulo 2

• Resistência do sistemaCom as expressões obtidas para a corrente eléctrica e para a tensão podemosagora calcular a resistência eléctrica. Por exemplo, utilizando as expressõesque estão expressas em função de E1:

R =V

I=

E1 (L1 + σc1

σc2L2)

σc1 E1 S=

L1

σc1 S+

L2

σc2 S

comparando esta expressão com a obtida para a resistência de um únicocondutor verificamos que

R = R1 + R2

F.Barão, L.F.Mendes Electromagnetismo e Óptica (MEEC-IST) 57

Page 64: Colecção de Problemas de Electromagnetismo e Ópticafcomp.ist.utl.pt/EO.web/Book.29_01_2014.pdf · Preâmbulo Esta colectânea de problemas resulta da experiência dos autores na

Capítulo 2

2.2.4 Adição de resistências em paralelo

Consideremos a associação de dois condutores cilíndricos de secções S1 e S2,comprimento L e condutividades σc1 e σc2, representada na figura. Os condutoresestão ligados a uma fonte de tensão V .

• Relação entre os campos eléctricos nos dois condutores e a tensãoVSe os materiais condutores forem homogéneos, de condutividade constante,a corrente eléctrica está uniformemente distribuída em cada um deles. Aslinhas de campo são paralelas ao eixo longitudinal do condutor e têm adirecção da corrente eléctrica (~uz). Como ambos os condutores estãoligados directamente à fonte de tensão a diferença de potencial entre osseus extremos é a mesma. Desta forma,

V =

∫ L

0

~E1 · ~dℓ =

∫ L

0

~E2 · ~dℓ

V = E1 L = E2 L

E1 = E2 = E

• A densidade de corrente nos condutores e a corrente eléctrica totalUtilizando o resultado anterior e a lei de Ohm temos:

~J1 = σc1E~uz

~J2 = σc2E~uz

A corrente eléctrica total que percorre os condutores é o fluxo de cada umadas densidades de corrente através da respectiva secção. Sendo as normaisàs secções dos condutores paralelas às densidades de corrente temos:

I =

S1

~J1 · ~n dS +

S2

~J2 · ~n dS

= J1 S1 + J2 S2 = σc1 E S1 + σc2 E S2

= E (σc1 S1 + σc2 S2)

• Resistência do sistemaCom as expressões obtidas para a corrente eléctrica e para a tensão podemosagora calcular a resistência eléctrica:

R =V

I=

E L

E (σc1 S1 + σc2 S2)

=L

σc1 S1 + σc2 S2

comparando esta expressão com a obtida para a resistência de umúnico condutor verificamos que

1

R=

1

R1

+1

R2

F.Barão, L.F.Mendes Electromagnetismo e Óptica (MEEC-IST) 58

Page 65: Colecção de Problemas de Electromagnetismo e Ópticafcomp.ist.utl.pt/EO.web/Book.29_01_2014.pdf · Preâmbulo Esta colectânea de problemas resulta da experiência dos autores na

Capítulo 2

2.2.5 Corrente eléctrica numa coroa esférica

O espaço existente entre um condutor esférico de raio R1 e uma coroa esféricacondutora de raios R2 e R3, está preenchido com um material de condutividadeeléctrica σc. Os condutores estão ligados a uma fonte que assegura a passagemde uma corrente eléctrica estacionária de intensidade I, do condutor interior parao exterior.

• A densidade de corrente eléctricaTendo o sistema simetria esférica e sendo o material homogéneo (σc éconstante), a corrente eléctrica dirige-se radialmente do condutor interiorpara o condutor exterior, pelo ~J = J~ur. Para calcular o módulo da densi-dade de corrente podemos usar uma secção que é uma superfície esférica deraio r (R1 < r < R2). Nesta superfície ~J será senpre paralelo à normalà superfície e além disso, devido à simetria do problema, J será constante.Deste modo,

I =

~J · ~n dS =

J dS = J 4πr2

~J =I

4πr2~ur

• A resistência eléctrica do sistemaCalculando a diferença de potencial a partir da lei de Ohm temos:

V =

~E · ~dℓ =

∫ R2

R1

J

σc

dr

=I

σc 4π

R2 − R1

R1 R2

R =V

I=

1

σc 4π

R2 − R1

R1 R2

• Densidade de potência dissipada por efeito de Joule

p = J E =J2

σc

=1

σc

(I

4πr2

)2

• Potência dissipada por efeito de JouleComo a densidade de potência não é constante (depende de r) não pode-mos obter a potência total multiplicando a sua densidade pelo volume dacoroa esférica. Temos, isso sim, de integrar a densidade de potência novolume da coroa esférica (em coordenadas esféricas):

P =

V

p dV

=

∫ 2π

0

∫ π

0

∫ R2

R1

1

σc

(I

4πr2

)2

r2senθ drdθdφ

=I2

σc 4π

R2 − R1

R1 R2

= R I2 = V I

F.Barão, L.F.Mendes Electromagnetismo e Óptica (MEEC-IST) 59

Page 66: Colecção de Problemas de Electromagnetismo e Ópticafcomp.ist.utl.pt/EO.web/Book.29_01_2014.pdf · Preâmbulo Esta colectânea de problemas resulta da experiência dos autores na

Capítulo 2

2.3 Exercícios Propostos

Exercício 2.1 : Um feixe de partículas alpha (cons-tituídas por dois protões e dois neutrões), com cargaeléctrica q = 2e, massa m = 4mp, energia cinética20 MeV/partícula e de secção S = 1 mm2, transportauma corrente eléctrica de 0,25 µA.

a) Determine a densidade de corrente eléctrica, ~J .

b) Determine a densidade de partículas existente nofeixe, n.

c) Determine o número de partículas alpha, N , queatravessam uma superfície plana perpendicularao feixe, em cada segundo.

d) Que diferença de potencial foi aplicada às partí-culas para as levar do estado de repouso ao es-tado cinético do feixe?

Exercício 2.2 : Um cabo de cobre com um comprimentoℓ = 1 km e uma secção de S = 10−4 m2 tem aplicadoum campo eléctrico de módulo E = 0, 05 V.m−1. Acondutividade eléctrica do cobre é σc = 5, 8 × 107

Ω−1.m−1. Determine:

a) a densidade de corrente no cabo.

b) a diferença de potencial entre as extremidades docabo.

c) a corrente transportada pelo cabo.

d) a potência dissipada pelo cabo.

Exercício 2.3 : Um disco de espessura ℓ = 0, 5 cme raio R = 50 cm está carregado uniformemente comuma densidade de carga ρ = 10−6 C.m−3. O discoroda com uma frequência angular ω = 50 r.p.m. De-termine:

a) a densidade de corrente eléctrica existente nodisco.

b) a corrente total que atravessa uma secção dodisco.

Exercício 2.4 : Um pára-raios termina num condutoresférico meio enterrado no solo. Uma pessoa dirige-se nasua direcção quando este recebe uma descarga de 2000A. Sabendo que quando se dá a descarga a pessoa estáa dar um (grande!) passo, estando o seu pé da frente a50 metros do pára-raios e o seu pé de trás a 51 metrosdo pára-raios, calcule a diferença de potencial entre os

seus pés.(Nota: σsolo = 10−2 Ω−1.m−1)

Exercício 2.5 : Para colocar o toner no papel uma fo-tocopiadora deposita primeiro uma certa quantidade decarga sobre uma superfície de selénio. Quando a su-perfície de selénio é iluminada com a imagem da có-pia as zonas claras (com luz) tornam-se condutoras,escoando-se a carga que lá estava. As zonas negras(sem luz) mantém-se isolantes, não escoando imediata-mente a carga depositada. São essas as zonas escurasque vão atrair o toner e depois transferi-lo para o pa-pel. A velocidade do processo está limitada pelo tempode residência da carga no local em que foi colocada.Sabendo que a resistividade do selénio utilizado numadeterminada fotocopiadora é ρ = 1011 Ω.m e que asua permitividade eléctrica é ε = 6 ε0, calcule o tempodurante o qual a carga depositada se reduz para metade.

Exercício 2.6 : Um condensador cilíndrico de compri-mento L e raios R1 e R2 (L >> R2), possui entreos condutores um material de permitividade eléctrica εe de condutividade eléctrica σc. O condensador foi car-regado electricamente por uma fonte de tensão V , como terminal positivo ligado ao condutor interior do con-densador, encontrando-se agora desligado.

a) Determine a capacidade do condensador.

b) Determine a corrente de fuga (I) do condensa-dor através do material que separa os conduto-res, imediatamente após se ter desligado a fontede tensão (quando o condensador possui a cargamáxima).

c) Verifique que a constante de tempo associada àdescarga do condensador através da corrente defuga, τfuga = RfugaC, é independente dasdimensões do condensador e relacione o resultadoobtido com a equação diferencial que descreve oescoamento de carga num meio condutor.

F.Barão, L.F.Mendes Electromagnetismo e Óptica (MEEC-IST) 60

Page 67: Colecção de Problemas de Electromagnetismo e Ópticafcomp.ist.utl.pt/EO.web/Book.29_01_2014.pdf · Preâmbulo Esta colectânea de problemas resulta da experiência dos autores na

Capítulo 2

Exercício 2.7 : Uma resistência eléctrica, constituídapor um material de condutividade eléctrica σc, possuiuma forma cónica de altura L em que o topo supe-rior possui um raio a e o topo inferior um raio b. Ainclinação do cone é pequena (b∼a), pelo que se podeconsiderar a densidade de corrente uniforme ao longo docone. Sabendo que a resistência é percorrida por umacorrente I, determine:

a

L

I

b

a) o campo eléctrico ao longo do condutor.

b) a diferença de potencial na resistência.

c) a resistência do dispositivo; particularize o resul-tado para um dispositivo cilindrico (b = a).

Exercício 2.8 : Considere o sistema da figura em queum condutor esférico de raio a está separado por umlíquido de condutividade eléctrica σc e permitividadeeléctrica ε, de um outro condutor cuja superfície in-terna de forma esférica, possui raio b. Os condutoresestão ligados a uma bateria que mantém uma diferençade potencial V . Determine:

a) o campo eléctrico, ~E, no espaço entre os condu-tores (em função de V ).

b) a corrente eléctrica, I, que percorre o circuito.

c) a resistência do sistema; esboce o gráfico da re-sistência do sistema em função do raio b.

d) a resistência do sistema quando b >> a; quala relação entre a e b para a qual a resistênciado sistema é 90% da máxima possível (quandob → ∞)?

Exercício 2.9 : A medição da condutividade da águado mar pode fazer-se recorrendo a duas esferas metálicasde raio a, imersas na água e suficientemente afastadas,que se encontram ligas a uma fonte de tensão V .

V

a) Determine a resistência eléctrica do circuito.Nota: despreze a resistência dos condutores eutilize o resultado do problema anterior.

b) Admitindo que a leitura da corrente é I0, deter-mine a condutividade eléctrica do água do mar.

F.Barão, L.F.Mendes Electromagnetismo e Óptica (MEEC-IST) 61

Page 68: Colecção de Problemas de Electromagnetismo e Ópticafcomp.ist.utl.pt/EO.web/Book.29_01_2014.pdf · Preâmbulo Esta colectânea de problemas resulta da experiência dos autores na

Capítulo 2

2.4 Soluções

2.1 a) J = 0, 25 A.m−2

b) n = 2, 5 1010 partículas.m−3

c) N = 7, 8 1011 partículas

d) V = 10 MV

2.2 a) J = 29 × 105 A.m−2

b) V = 50 V

c) I = 290 A

d) P = 14, 5 kW

2.3 a) J = 5, 2 10−6 r (A.m−2)

b) I = 3, 25 10−9 A

2.4 12, 5 V

2.5 3, 7 s

2.6 a) C = 2πεℓ

ln(

R2R1

)

b) I = V 2πℓσc

ln(

R2R1

)

c) τfuga = εσc

, isto é, não depende da geometriado condensador e é idêntica à constante de tempode escoamento de carga, como seria de esperar.

2.7 a) E = Iσc

1

π(

a+b−aL

y)2

b) V = Iσc

Lπab

c) Rcone = 1σc

Lπab

; Rcil = 1σc

LπR2 ;

2.8 a) ~E = V abb−a

1r2 ~ur

b) I = V σc4πabb−a

c) R = 1σc

b−a4πab

d) Rmax(b >> a) = 1σc

14πa

;R = 0, 9Rmax ⇒ b = 10a

2.9 a) R = 12πaσc

b) σc = I02πaV

F.Barão, L.F.Mendes Electromagnetismo e Óptica (MEEC-IST) 62

Page 69: Colecção de Problemas de Electromagnetismo e Ópticafcomp.ist.utl.pt/EO.web/Book.29_01_2014.pdf · Preâmbulo Esta colectânea de problemas resulta da experiência dos autores na

Capítulo 3

Capítulo 3

Magnetostatica

3.1 Introdução

Em 1784 Coulomb estudou não só a força entre cargas eléctricas mas também aforça entre ímanes. As suas experiências levaram-no a concluir que existe umacerta semelhança entre os fenómenos eléctricos e magnéticos: o fluido magnéticoparece ter, pela sua natureza ou pelas suas propriedades, uma analogia com o fluidoeléctrico. No início de 1820, ano mágico do magnetismo, Oersted leva mais longea intuição de Coulomb ao verificar que as bússolas são desviadas na presença deuma corrente eléctrica; um fio com uma corrente é uma espécie de íman. Estaconstatação pode ser considerada como o início da unificação dos fenómenoseléctricos e dos fenómenos magnéticos, ou seja, o início do electromagnetismo. Hojesabe-se que de facto os dois fenómenos são diferentes manifestações de uma mesmainteracção física e que tudo depende do referencial. No entanto, por simplicidade,na maior parte das aplicações continua a ser mais conveniente estudar os fenómenosmagnéticos enquanto tal.

Magnetismo

Foi também no ano de 1820, após as experiências de Oersted, que Ampère realizouexperiências que o levam a concluir que a força existente entre dois fios rectilíneosde comprimentos L1 e L2 percorridos por correntes eléctricas I1 e I2 e distanciadosde d é descrita por

F = Km

I1L1 I2L2

d2(3.1)

Ampère concluiu ainda que dentro dos ímanes devem haver muitos circuitos muitopequenos percorridos por correntes eléctricas.Definindo um elemento de corrente I ~dℓ como o produto da corrente eléctrica quepercorre um circuito por um elemento infinitesimal desse circuito, sabemos hoje quea força exercida por um elemento de corrente de um circuito 1 sobre um elementode corrente de um circuito 2, distanciado de r, é dada por

d2 ~F12 =µ0

I2 ~dℓ2 × I1 ~dℓ1 × ~ur

r2(3.2)

Força magnética

F.Barão, L.F.Mendes Electromagnetismo e Óptica (MEEC-IST) 63

Page 70: Colecção de Problemas de Electromagnetismo e Ópticafcomp.ist.utl.pt/EO.web/Book.29_01_2014.pdf · Preâmbulo Esta colectânea de problemas resulta da experiência dos autores na

Capítulo 3

em que

µ0

4π= 10−7 N.A−2 (3.3)

sendo µ0 é a permitividade magnética do vácuo (e do ar!).

A força entre dois circuitos, da qual o resultado obtido por Ampère é umcaso particular, pode ser obtida somando a contribuição de todos os elementos decorrente de cada um dos circuitos,

~F12 =

2

1

d2 ~F12 (3.4)

Um facto curioso é o de em geral d2 ~F12 6= −d2 ~F21, parecendo à primeiravista que a terceira lei de Newton não se aplica à força magnética. No entanto, épreciso lembrar que elementos de corrente não existem sozinhos mas sim integradosem circuitos fechados. A terceira lei de Newton aplica-se à força magnética mas naforma ~F12 = −~F21.

Uma abordagem mais eficaz ao cálculo das forças magnéticas, e que se revelaria maistarde imprescindível para a descrição dos fenómenos electromagnéticos, foi propostaainda no mesmo ano de 1820 por Biot e por Savart. Estes, a partir dos resultadosde Ampère, propuseram a existência de um campo magnético, ~B, e de uma lei queé conhecida pelos seus nomes: lei de Biot-Savart. À semelhança da definição decampo eléctrico, o campo magnético é obtido como a força por unidade de elementode corrente sobre o qual é exercida, ou seja, isolando o efeito que o elemento decorrente gerador da força exerce sobre o espaço. Assim, um elemento de corrente docircuito 1 cria um campo magnético sobre um ponto P a uma distância r dado por

d ~B1 =µ0

I1 ~dℓ1 × ~ur

r2(3.5)

O campo magnético criado por todo o circuito 1 no ponto P é obtido somando(integrando) a contribuição de todos os seus elementos de corrente:

~B1 =

1

d ~B1 (3.6)

A força exercida por este campo magnético sobre um elemento de corrente do circuito2 é dada por

d~F12 = I2 ~dℓ2 × ~B1 (3.7)

E, finalmente, a força que o campo magnético do circuito 1 exerce sobre todo ocircuito 2 é dada por

~F12 =

2

I2 ~dℓ2 × ~B1 (3.8)

Campo magnético - Lei deBiot-Savart

F.Barão, L.F.Mendes Electromagnetismo e Óptica (MEEC-IST) 64

Page 71: Colecção de Problemas de Electromagnetismo e Ópticafcomp.ist.utl.pt/EO.web/Book.29_01_2014.pdf · Preâmbulo Esta colectânea de problemas resulta da experiência dos autores na

Capítulo 3

A lei de Ampère é uma consequência da lei de Biot-Savart e resulta do facto deo campo magnético criado por um elemento de corrente finito diminuir com 1/r,enquanto que o arco que corresponde a uma variação angular dθ aumenta com r.Desta forma, para caminhos caminhos fechados em torno de correntes, o produtodo campo pelo comprimento do caminho é independente da distância à que correnteque deu origem ao campo.A lei de Ampère relaciona o integral de linha do campo magnético num caminhofechado com a corrente eléctrica que atravessa qualquer superfície que tenha comofronteira esse caminho fechado. Em geral, e por conveniência de cálculo, considera-sea superfície plana que se "apoia"no caminho fechado:

Γ

~B · ~dℓ = µ0

~J · ~n dS (3.9)

Esta lei pode ser escrita na forma local, tendo em conta o teorema de Stokes, como:

~∇ × ~B = µ0~J (3.10)

Note que na aplicação da lei de Ampère, a escolha do sentido de circulação no cálculodo integral de linha do campo magnético define o sentido da normal à superfície planadelimitada pelo caminho fechado; uma circulação percorrida no sentido anti-horárioresulta, aplicando a regra da mão direita, numa normal a apontar para “cima”.A lei de Ampère é útil para determinar:

• o módulo do campo magnético para certas distribuições de corrente,utilizando-se para tal um caminho fechado conveniente;

• a densidade de corrente eléctrica se se conhecer a expressão que descreve ocampo magnético numa certa região do espaço

No cálculo do módulo do campo magnético, a aplicabilidade da lei de Ampéreencontra-se reduzida a quatro geometrias tipo de distribuição de corrente:

• simetria cilíndrica

• bobina infinita

• corrente em torno de um toróide

• plano infinito de corrente

Campo magnético - Lei deAmpère

Um dipolo magnético consiste num circuito fechado percorrido por uma correnteeléctrica, sendo o anel de corrente um bom exemplo. O campo magnético produzidopor um anel de corrente de raio a e corrente eléctrica I, em pontos existentes aolongo do seu eixo (assume-se o eixo dos zz coincidente com o eixo do anel), foiobjecto de resolução na aplicação resolvida ??, tendo-se obtido:

Bz =µ0

2I a2 1

(a2 + z2)3/2(3.11)

Definindo o momento dipolar magnético como,

~m = I ~A (3.12)

em que A é a área do dipolo, cuja direccção e sentido, são definidos respectivamentepela normal ao plano do anel de corrente e pelo sentido de circulação da corrente(recordar a regra da mão direita!), pode-se reescrever o campo magnético como:

Dipolo magnético

F.Barão, L.F.Mendes Electromagnetismo e Óptica (MEEC-IST) 65

Page 72: Colecção de Problemas de Electromagnetismo e Ópticafcomp.ist.utl.pt/EO.web/Book.29_01_2014.pdf · Preâmbulo Esta colectânea de problemas resulta da experiência dos autores na

Capítulo 3

Bz =µ0

m

(a2 + z2)3/2

(3.13)

Os planetas do sistema solar possuem campos magnéticos que variam de planetapara planeta mas em primeira aproximação, a configuração do campo magnéticocorresponde ao produzido por um dipolo magnético. O momento dipolar magnéticoda Terra é de m = 8 × 1022 A.m2. O campo magnético à superfície da Terra,calculado com o auxílio da expressão anterior, é da ordem de 0.1 µT .

A aplicação de um campo magnético ~B sobre um circuito eléctrico de comprimentoL e percorrido por uma corrente eléctrica I, tem como consequência a existência deuma força:

~F =

L

d~ℓ× ~B (3.14)

Caso a corrente eléctrica esteja distribuída em volume e tendo em conta que

I d~ℓ = ~J dS dℓ = ~J dvol (3.15)

a força pode-se re-escrever como sendo:

~F =

vol

( ~J × ~B) dvol (3.16)

Interacção do dipolo mag-nético com campo ~B

Para se compreender o funcionamento do motor de corrente contínua, importa es-tudar o comportamento de de uma espira quadrada de lado ℓ percorrida por umacorrente I e submetida a um campo B~ex. Os quatro lados da espira possuem forçasaplicadas. As forças existentes sobre os troços da espira perpendiculares ao papelcolocam a espira em rotação e as forças existente sobre os outros dois troços exer-cem forças deformadoras da espira e anulam-se globalmente. A força sobre cada umdos dois troços perpendiculares ao papel é F = IℓB. Sendo o momento dipolarmagnético da espira dado por ~m = iℓ2, pode-se calcular o momento da força sobrea espira como sendo:

~N =∑

~r × ~F = ℓ/2F sinα + ℓ/2F sinα = Iℓ2B sinα

= mB sinα = ~m × ~B (3.17)

Portanto, o momento da força aplicado à espira fá-la rodar de forma a alignar o seumomento dipolar magnético com o campo magnético externo.Em termos gerais, a aplicação de forças sobre um corpo pode ter como consequênciaum movimento de translação ou de rotação, no caso da existência de um momentoda força. Desta forma, o trabalho infinitesimal realizado é dado por:

dW = ~F · d~ℓ = ~F · ~vdt = ~F · (~ω × ~r)dt = (~r × ~F ) · ~ωdt= ~N · d~α (3.18)

A energia da espira no campo magnético corresponderá assim ao trabalho que umaforça exterior terá que realizar para a fazer rodar (contra o momento da força devidoao campo).

Energia potencial do dipolomagnético

F

F

α

α

mBα

F.Barão, L.F.Mendes Electromagnetismo e Óptica (MEEC-IST) 66

Page 73: Colecção de Problemas de Electromagnetismo e Ópticafcomp.ist.utl.pt/EO.web/Book.29_01_2014.pdf · Preâmbulo Esta colectânea de problemas resulta da experiência dos autores na

Capítulo 3

Admitindo, por exemplo, que a espira esteja na posição de equilíbrio em que o seumomento dipolar magnético se encontra alinhado com o campo ~B (α = 0) e quea força exterior a pretende rodar de um ângulo θ no sentido anti-horário, o trabalhorealizado vem:

W =

∫ θ

0

(− ~N) · d~α =

∫ θ

0

(−N)(−dα) = mB

∫ θ

0

sinαdα

= −mB(cos θ − 1) (3.19)

Este trabalho, corresponde à energia potencial do dipolo magnético na presença deum campo magnético uniforme. Por razões de simplicidade, uma vez que o zero dopotencial ou da energia potencial é completamente arbitrário, a parte constante daexpressão (+mB) pode ser eliminada, ficando então a energia potencial do dipolocomo:

U = −~m · ~B (3.20)

De notar que a posição de energia mínima U = −mB se obtém quando θ = 0,isto é, quando o momento dipolar magnético se encontra alinhado com o campo ~B.

A presença de um meio material (que não o ar!) produz efeitos no campo magnético,tal como acontecia com o campo eléctrico. Do ponto de vista microscópico, a matériaé formada por átomos e estes são constituídos por um núcleo feito de protões eneutrões (carga total positiva) e electrões que numa visão clássica orbitam em tornodo núcleo. Ao movimento orbital de raio r dos electrões com velocidade angular ωe carga eléctrica e, corresponde uma corrente eléctrica:

I =dq

dt=

dq

ds

ds

dt=

e

2πrωr︸︷︷︸

v

=e

2πω (3.21)

e portanto um momento dipolar magnético, ~m = 12er2ω. Tendo em conta que

o momento angular associado ao electrão é L = ~r × ~p = mer2ω, vem para o

momento dipolar magnético:

~m =e

2me

~L (3.22)

Além do mais, existe o número quântico spin associado à partícula cuja traduçãoclássica pode ser vista como uma rotação da partícula em torno de si própria, a quecorresponde também um momento dipolar magnético. Donde, o momento dipolarmagnético atómico resultará da soma dos diferentes momentos dipolares magnéticosassociados ao átomo. Por exemplo, o emparelhamento dos electrões nos átomos comspins opostos ↑↓ resulta na soma de dois momentos dipolares magnéticos simétricos,cujo resultado é zero. Uma nota adicional ainda para o facto do momento angularser quantificado (mecânica quântica!), isto é, só pode assumir determinados valoresmúltiplos da unidade h

2π, onde h é a constante de Planck. O momento dipolar do

electrão de 1a ordem é 9, 274−24 Am2.Do ponto de vista magnético, os materiais dividem-se entre os que possuiem átomosou moléculas com momento dipolar magnético - paramagnéticos e ferromagnéti-cos - e aqueles que não possuem - diamagnéticos. Nos materiais ferromagnéticos,a interaccão entre átomos resulta na existência de um alinhamento colectivo dosdipolos magnéticos, em domínios magnéticos.

Campo magnéticona matéria

F.Barão, L.F.Mendes Electromagnetismo e Óptica (MEEC-IST) 67

Page 74: Colecção de Problemas de Electromagnetismo e Ópticafcomp.ist.utl.pt/EO.web/Book.29_01_2014.pdf · Preâmbulo Esta colectânea de problemas resulta da experiência dos autores na

Capítulo 3

O comportamento de um material do ponto de vista magnético resulta do facto deeste ser formado por átomos e estes possuirem dipolos magnéticos. O momentodipolar dos átomos ou moléculas do material pode ser nulo ou não. Admitamos noentanto estar na presença de um material paramagnético ou ferromagnético ao qualé aplicado um campo magnético B0. Tal como vimos anteriormente, os momentosdipolar magnéticos da substância tendem-se a alinhar com a direcção do campomagnético. Olhando para o material como um contínuo de dipolos, em cada volumemicroscópico do material pode-se definir um momento magnético dipolar,

d~m = ~M dV ol (3.23)

onde M é o vector magnetização cuja unidade é A/m. Desta forma, a contri-buição dos dipolos para o campo magnético total poderia ser obtida integrando ocampo magnético produzido por cada elemento dipolar d~m sobre todo o volumemagnetizado.Os elementos dipolares da substância podem ser vistos como anéis de corrente eléc-trica que caso a magnetização seja uniforme, se anulam no interior do material. Doponto de vista do campo magnético, o efeito magnetização pode ser calculado apartir destas correntes eléctricas equivalentes - correntes de magnetização - quese formam no material seja à sua superfície, seja no seu interior. A densidade decorrente de magnetização é dada por:à superfície do material, ~Jm = ~M × ~n [A/m]onde ~n é o vector unitário normal à superfície do material e que aponta para foradeste.no interior do material, ~Jm = ~∇× ~M [A/m2]

Magnetização

mJ

M

Se colocarmos um enrolamento de espiras percorrido por corrente eléctrica I em voltade um cilindro feito de material magnetizável de permeabilidade µ, o campo mag-nético ~B no interior do material resultará das contribuições das correntes eléctricaslivre existente no enrolamento e de magnetização existente no material.

~B · d~ℓ = µ0Itot = µ0

(

Ilivre +

S

~Jm · d~S)

(3.24)

Tendo em conta que ~Jm = ~∇ × ~M e usando o teorema de Stokes obtemos aseguinte igualdade:

S~(~∇ × ~M) · d~S =

Γ~M · d~ℓ. Obtém-se assim a lei de

Ampère generalizada,

∮(

~B

µ0

− ~M

)

︸ ︷︷ ︸

~H

·d~ℓ = Ilivre (3.25)

onde ~H é o vector intensidade de campo magnético. Em meios lineares, a magne-tização do material é proporcional à corrente eléctrica que circula no enrolamento:~M = χm

~H. Em materiais do tipo paramagnético, χm > 0 e materiais diagné-ticos, χm < 0. A relação entre o campo magnético ~B e a intensidade de campomagnético ~H , obtém-se então:

~B

µ0

− ~M = ~H ⇒ ~B = µ0(~H + ~M) = µ0( +χm)~H = µ~H

(3.26)

Lei de Ampèregeneralizada

m

m

m

m

I

Circuitos magnéticos

Energia magnética

F.Barão, L.F.Mendes Electromagnetismo e Óptica (MEEC-IST) 68

Page 75: Colecção de Problemas de Electromagnetismo e Ópticafcomp.ist.utl.pt/EO.web/Book.29_01_2014.pdf · Preâmbulo Esta colectânea de problemas resulta da experiência dos autores na

Capítulo 3

Força magnética

1.0 Condições fronteira

E//

En

meio 1

meio 2

E//

E

meio1Cmeio2

h

Continuidade da componente normal do campo ~B Suponhamos dois meios, 1 e 2, e a sua superfíciede separação. Pela lei de conservação do fluxo, o fluxo do campo magnético ~B através de uma superfície cilíndrica,cujo eixo é paralelo à normal à superfície de separação dos meios é nulo. Designemos para tal, S1 a superfície dotopo do cilindro no meio 1, S2 a superfície do topo do cilindro no meio 2 e Sl a superfície lateral do cilindro.

ΦB =

S1

B1 · ~n1dS +

S2

B2 · ~n2dS +

Sl

B · ~ndS = 0

Se fizermos tender a altura do cilindro para zero (h → 0), o terceiro termo no cálculo do fluxo anula-se uma vezque a superfície lateral do cilindro tende para zero. Ficamos então com:

ΦB =

S1

B1 · ~n1dS +

S2

B2 · ~n2dS = 0

=

S

(B1 − B2) · ~ndS = 0

Obtém-se então a continuidade da componente normal do campo ~B na transição de meios, mesmo se a superfíciede separação possuir correntes eléctricas superficiais.

(

~B1 − ~B2

)

· ~n = 0 ⇒ B⊥1 = B⊥

2 (3.27)

Descontinuidade da componente tangencial do campo ~B Consideremos agora um plano per-pendicular à superfície de separação e neste desenhemos um caminho fechado C de altura h → 0. A aplicação doteorema de Ampère generalizado ao caminho fechado resulta em:

C

~H · d~ℓ = Iliv∫

C1

~H1 · d~ℓ+∫

C2

~H2 · d~ℓ =

∫ (

~H1 − ~H2

)

· d~ℓ =

~Js · (~n × d~ℓ)

onde Js é a corrente eléctrica superficial que passa na superfície de separação e atravessa o caminho C definido.Resulta então:

H1 − H2 = Js (3.28)

F.Barão, L.F.Mendes Electromagnetismo e Óptica (MEEC-IST) 69

Page 76: Colecção de Problemas de Electromagnetismo e Ópticafcomp.ist.utl.pt/EO.web/Book.29_01_2014.pdf · Preâmbulo Esta colectânea de problemas resulta da experiência dos autores na

Capítulo 3

3.2 Exercícios Propostos

Exercício 3.1 : Uma espira circular de raio R situada noplano XY e centrada na origem é percorrida por umacorrente eléctrica estacionária de intensidade I. Deter-mine o campo magnético ~B num ponto do eixo zz, àdistância genérica z do plano da espira.

Exercício 3.2 : Uma forma de gerar um campo magné-tico relativamente uniforme numa dada zona do espaço éatravés de um sistema composto por duas espiras circu-lares de raio R, separadas por uma distância R tambéme percorridas por uma corrente I com igual sentido, talcomo se mostra na figura (bobinas de Helmholtz).

a) Verifique que o campo magnético criado por umaespira circular de raio R e percorrida por umacorrente I, num ponto do eixo que passa peloseu centro a uma distância z da espira, é dadopela expressão:

~B =µ0

2IR2

(z2 + R2

)−3/2~uz

b) Verifique que o campo magnético criado pelo sis-tema de duas espiras representado na figura édado por:

~B =µ0

2IR2

((z2 + R2

)−3/2+

((z − R)2 + R2

)−3/2)

~uz

c) Verifique que para z = R2

, se tem dBdz

= 0.

Nota: Derivando mais uma vez obter-se-ia,d2Bdz2 = 0, o que é indicador de que o campomagnético é aproximadamente constante (até àsegunda ordem) na região entre as espiras.

Exercício 3.3 : Determine, utilizando a lei de Biot-Savart, o campo magnético criado por um fio infinitopercorrido por uma corrente estacionária I, a uma dis-tância r do fio.

Exercício 3.4 : Uma espira quadrada de lado ℓ, colo-cada no plano xy é percorrida por uma corrente eléc-trica I. Determine o campo magnético existente no seucentro geométrico.Exercício 3.5 : Dois fios paralelos muito compridostransportam correntes de I = 10 A no mesmo sentido eencontram-se separados por uma distância de ℓ=1 mm.

I

I

Determine a força que actua em 2 metros de cadaum dos fios.

Exercício 3.6 : Um fio rectilíneo muito longo transportauma corrente I1 e na sua proximidade e à distância dé colocada uma espira rectangular, que é percorrida poruma corrente eléctrica I2. Determine:

a) a força que o fio rectilíneo exerce sobre a espira.

b) a força que a espira exerce sobre o fio.

c) o momento das forças que actuam os lados daespira em relação ao seu centro geométrico.

Exercício 3.7 : Um motor de corrente contínua é cons-tituído por um circuito quadrado de lado ℓ, percorridopor uma corrente I, na presença de um campo magné-tico uniforme, B. A normal ao circuito forma um ângulode 90 com a direcção do campo magnético.

F.Barão, L.F.Mendes Electromagnetismo e Óptica (MEEC-IST) 70

Page 77: Colecção de Problemas de Electromagnetismo e Ópticafcomp.ist.utl.pt/EO.web/Book.29_01_2014.pdf · Preâmbulo Esta colectânea de problemas resulta da experiência dos autores na

Capítulo 3

a) Determine a força exercida em cada um dos ladosdo circuito.

b) Determine a resultante das forças que actuam nocircuito.

c) Determine o momento das forças ( ~N) que ac-tuam o circuito relativamente ao seu centro.

d) Define-se o momento do dipolo magnético como~m = IA~n, sendo A a área do circuito. Mos-tre que poderia escrever o momento das forçasaplicadas ao circuito como ~N = ~m × ~B.

Exercício 3.8 : Um corta-circuitos de correntes indus-triais é formado por dois fios muito longos de com-primento a, ligados por uma barra metálica de com-primento 2b << a, como se mostra na figura. Abarra contém a meio uma parta móvel de comprimentod << 2b, que está ligada a um ponto fixo através deuma mola. Para deslocar a parte móvel e interromper ocircuito é necessário exercer uma força ~F = −F~ux.

a) Determine a expressão do campo magnético, ~B,no ponto médio da parte móvel da barra, em fun-ção da intensidade de corrente que circula nocorta-circuitos, I. Despreze a contribuição dotroço inferior do circuito.

b) Admitindo que o campo calculado em a) é apro-ximadamente constante na parte móvel da barra,obtenha a expressão para o valor da intensidadede corrente acima do qual o circuito é interrom-pido.

c) Utilizando qualitativamente os resultados das alí-neas anteriores diga, justificando, qual a formageométrica que tomaria um circuito constituídopor um fio extremamente flexível, percorrido poruma corrente, na ausência de qualquer força ex-terior.

Exercício 3.9 : Um disco isolante de raio R, que estáuniformemente carregado com uma densidade de cargasuperficial σ, encontra-se a rodar com uma velocidadeangular ω. Determine o campo magnético no centro dodisco.

Exercício 3.10 : Numa fábrica de plásticos, devido àfricção do plástico nos rolos cilíndricos ao longo dosquais é arrastado, gerou-se no plástico uma carga su-perficial +σ. Sabendo que o plástico se desloca comuma velocidade v, determine o campo magnético juntoao plástico.

Exercício 3.11 : Determine, utilizando a lei de Ampère,o campo magnético criado por um fio infinito percorridopor uma corrente estacionária I, a uma distância r dofio.

Exercício 3.12 : Um solenóide bastante longo (compri-mento L >> R) possui uma densidade de espiras n e épercorrido por uma corrente estacionária I. Determineo campo magnético no interior do solenóide.

Exercício 3.13 : O tokamak, acrónimo russo para câ-mara magnética toroidal, é usado no confinamento doplasma quente ionizado na fusão nuclear. O enrola-mento eléctrico tem uma forma toroidal de raio médioR, possui N espiras e é percorrido por uma correnteeléctrica I.

I

R

a) Determine o campo magnético na circunferênciade raio R, que passa pelo centro das espiras.

F.Barão, L.F.Mendes Electromagnetismo e Óptica (MEEC-IST) 71

Page 78: Colecção de Problemas de Electromagnetismo e Ópticafcomp.ist.utl.pt/EO.web/Book.29_01_2014.pdf · Preâmbulo Esta colectânea de problemas resulta da experiência dos autores na

Capítulo 3

b) Verifique que se utilizar a densidade de espiras,n, referida ao comprimento da circunferência deraio R, a expressão do campo não depende deR. Diga qual será o campo magnético criadopor uma bobina infinita.

Exercício 3.14 : Um cabo coaxial tem um condutorcentral de raio a separado por um material isolante deum tubo condutor concêntrico de raios interno e externob e c, respectivamente. Os dois condutores transpor-tam correntes eléctricas com sentidos opostos, unifor-memente distribuídas e paralelas aos respectivos eixos.A intensidade da corrente em cada um dos condutores éI. Determine o campo magnético nas seguintes regiões:

a) Interior do condutor central (r < a).

b) Espaço entre os dois condutores (a < r < b).

c) Interior do condutor exterior (b < r < c).

d) Exterior do cabo coaxial (r > c).

Exercício 3.15 : Um condutor cilíndrico muito com-prido, de raio a e preenchido por um material de perme-abilidade magnética µ0, é percorrido por uma correnteeléctrica estacionária não uniforme cuja densidade decorrente é descrita por ~J = J0r~uz.

a) Determine a intensidade de corrente que atra-vessa a secção transversal do condutor.

b) Desenhe as linhas de campo magnético e obtenhaa sua expressão para todo o espaço (r < a er > a). Faça um gráfico de B(r).

c) Sabendo que a corrente é mantida por uma fonteque aplica ao condutor uma diferença de poten-cial por unidade de comprimento V

, determinea condutividade do cilindro, σ.

d) Imagine que se abria um orifício cilíndrico de raiob no interior do condutor central, a uma distânciaR do centro do condutor, tal como indicado na

figura. Determine, explicando detalhadamente oseu raciocínio, o campo magnético B no centrodo orifício.

Exercício 3.16 : Um cilindro com um comprimentoℓ = 20 cm, muito estreito e feito de um material comuma susceptibilidade magnética χm = 2, constitui onúcleo de um enrolamento com 150 espiras que sãopercorridas por uma corrente I = 2 A. Determine:

a) a permeabilidade magnética µ do material.

b) a intensidade do campo magnético, ~H , a mag-netização produzida no material ~M e o campomagnético ~B, no interior do cilindro.

c) as correntes de magnetização no material.

Exercício 3.17 : Um material condutor cilíndrico muitocomprido de raio R e permeabilidade magnética µ, épercorrido por uma corrente eléctrica estacionária, uni-formemente distribuída, de intensidade I.

a) Determine o campo magnético, ~B, criado pelocondutor no seu exterior a uma distância r doseu eixo (não muito afastado do condutor).

b) Determine o campo magnético, ~B, criado pelocondutor no seu interior.

c) Determine a densidade de corrente de magneti-zação no condutor, ~JM .

Exercício 3.18 : Um condutor de cobre, de secção cir-cular, comprido e rectilíneo, de raio a, está cobertocom uma camada de ferro de raio exterior b (b =a+espessura). Este condutor compósito é percorridopor uma intensidade de corrente I. Sendo a permeabi-lidade magnética do cobre µ0 e a do ferro µ e sendoas suas condutividades eléctricas respectivamente σCu

e σFe, determine:

F.Barão, L.F.Mendes Electromagnetismo e Óptica (MEEC-IST) 72

Page 79: Colecção de Problemas de Electromagnetismo e Ópticafcomp.ist.utl.pt/EO.web/Book.29_01_2014.pdf · Preâmbulo Esta colectânea de problemas resulta da experiência dos autores na

Capítulo 3

a) a densidade de corrente existente no Cobre (JCu)e no Ferro (JFe).

b) a intensidade do campo magnético, ~H e o campomagnético ~B, em todas as regiões do espaço, emfunção de JCu e JFe.

Exercício 3.19 : Utilize as condições fronteira do campomagnético na ausência de correntes para verificar que osmateriais ferromagnéticos se comçportam como condu-tores das linhas de campo.Sugestão: utilize a aproximação µ >> µ0.

Exercício 3.20 : Nas cabeças de gravação magnéticasos campos são criados por correntes pequenas e para seobterem campos intensos, utilizam-se entreferros (aber-turas em núcleos de materiais ferromagnéticos). Umcaso simples de um entreferro está representado na fi-gura abaixo, em que um núcleo de material ferromag-nético com a forma de um anel cilíndrico de raio médioR = 1 cm, possui um enrolamento de N = 20 espi-ras, percorridas por uma corrente de I = 1 mA. Nessenúcleo foi aberto um espaço de largura d = 10 µm.

I

N R

d

a) Utilizando as equações que descrevem o campomagnético, verifique que na fronteira de separa-ção entre o ar e o ferro, se verifica a conduiçãoB⊥ ar = B⊥ Fe.

b) Calcule o campo magnético no entreferro, Bar,na linha de campo média (r = R), assumindoque o material ferromagnético apresenta para es-tas condições uma permeabilidade magnéticaµ =105 µ0.

Exercício 3.21 : Um cilindro de espessura a = 1 mm eraio R = 1 cm encontra-se magnetizado uniformementeao longo do eixo zz, sendo a sua magnetização M =105 A.m−1.

a) Calcule as correntes equivalentes de magnetiza-ção no cilindro, ~Jm. Esboce as correntes no ci-lindro.

b) Fazendo as aproximações que considerar conve-nientes, calcule o campo magnético ~B no eixodo cilindro, para z = 0.

c) Esboce as linhas de campo magnético no interiordo disco e no espaço à sua volta.

F.Barão, L.F.Mendes Electromagnetismo e Óptica (MEEC-IST) 73

Page 80: Colecção de Problemas de Electromagnetismo e Ópticafcomp.ist.utl.pt/EO.web/Book.29_01_2014.pdf · Preâmbulo Esta colectânea de problemas resulta da experiência dos autores na

Capítulo 3

3.3 Resoluções

Exercício 3.8

a) Para calcular o campo magnético no ponto pretendido pode utilizar-se a lei de Biot-Savart; começando porver a direcção do campo criado por cada um dos lados do circuito, verifica-se que a própria barra não criacampo e que cada um dos lados de comprimento a, cria um campo segundo ~uz. Como os dois condutorescriadores do campo estão simetricamente colocados em relação ao ponto, pode-se simplesmente calcular ocampo de um dois lados e multiplicar por 2.

~B =

∫µ0

Id~ℓ × ~ur

r2=⇒ B =

∫ a

0

µ0

I‖d~x × ~ur‖r2

=

∫ a

0

µ0

4πIsen(dx, ur)

r2dx

=

∫ a

0

µ0

I

x2 + b2b

√x2 + b2

dx =µ0

4πIb

∫ a

0

1

(x2 + b2)3/2dx

~B = 2 × µ0

4πIb

a

b2√x2 + b2

~uz

=µ0

2πI

a

b√x2 + b2

~uz

Nota: lima→∞ B(a) = µ0

2πIb~uz

b) Já se conhece o campo magnético existente na parte móvel pelo que agora há somente que calcular a forçaque esse campo exerce sobre a corrente eléctrica que percorre a parte móvel,

~F =

Id~ℓ × ~B = IB

∫ d

0

dℓ(−~ux) = −IBd~ux = −µ0

2πI2 ad

b√x2 + b2

~ux

A corrente a partir da qual se interrompe o circuito é então,

I =

F2πb√a2 + b2

µ0ad

c) Verifica-se neste problema que o circuito cria sobre uma parte de si próprio uma força para fora. Se tivermosentão um circuito flexível, sem outras forças aplicads, e todos os seus pontos forem puxados para fora, estetomará uma forma circular.

Exercício 3.17

a) Como se trata de um problema de geometria cilíndrica com um condutor muito comprido (L >> R),vamos aplicar a lei de Ampère utilizando como caminho de integração uma linha de campo, ou seja, umacircunferência de raio r > R genérica (L >> r também). Neste caso a corrente que atravessa a superfíciedefinida pelo caminho fechado é toda a corrente I.

~H · d~ℓ =

~J · ~ndS = I =⇒ H2πr = I =⇒ H =I

2πr~uθ =⇒ ~B =

µ0

I

r~uθ

b) Neste aplicaremos a lei de Ampère como anteriormente mas a corrente que atravessa a superfície definida pelocaminho fechado é apenas a fracção de I que corresponde à fracção de área do condutor que nos interessa.

~H · d~ℓ =

~J · ~ndS = Iπr2

πR2=⇒ H2πr = I

r2

R2=⇒ H =

Ir

2πR2~uθ =⇒ ~B =

µ0

Ir

R2~uθ

c)

~B = µ0( ~H + ~M) ⇐⇒ ~M =~B

µ0

− ~H =

µ0

− 1

)

~H

~M =

µ0

− 1

)Ir

2πR2~uθ

F.Barão, L.F.Mendes Electromagnetismo e Óptica (MEEC-IST) 74

Page 81: Colecção de Problemas de Electromagnetismo e Ópticafcomp.ist.utl.pt/EO.web/Book.29_01_2014.pdf · Preâmbulo Esta colectânea de problemas resulta da experiência dos autores na

Capítulo 3

~JM = ~∇× ~M =

(1

r

∂Mz

∂θ− ∂Mθ

∂z

)

~ur +

(∂Mr

∂z− ∂Mz

∂r

)

~uθ +1

r

(∂(rMθ)

∂r− ∂Mr

∂θ

)

~uz

=∂Mθ

∂z~ur +

1

r

∂(rMθ)

∂r~uz =

1

r

∂(rMθ)

∂r~uz

=1

r

∂r

(

r

µ0

− 1

)Ir

2πR2

)

~uz =

µ0

− 1

)I

2πR2

1

r

∂r2

∂r~uz =

µ0

− 1

)I

πR2~uz

=

µ0

− 1

)

~J

A densidade de corrente de magnetização no interior do material existe ao longo do condutor, como a própriacorrente que o percorre. Vamos agora calcular a densidade de corrente que existe na sua superfície, e que éuma corrente por unidade comprimento, existente no perímetro da secção do condutor.

~J′

M = ~M(R) × ~next =

µ0

− 1

)IR

2πR2~uθ × ~ur

=

µ0

− 1

)I

2πR~−uz

Será ainda interessante verificar que a soma da corrente de magnetização no interior do condutor com acorrente de magnetização na sua superfície é nula. Isto conduz-nos ao resultado óbvio de que o campomagnético no exterior do condutor é independente das suas propriedades magnéticas.

Exercício 3.18

a) ~JCu = σcCu~E ; ~JFe = σcFe

~EO campo eléctrico é igual nos dois materiais pois a diferença de potencial entre os extremos do cabo é amesma, “vendo” pelo lado do Ferro ou pelo Cobre.

I = JCuπa2 + JFeπ(b

2 − a2) = σcCuEπa2 + σcFeEπ(b2 − a2)

=⇒ E =I

π (σcCua2 + σcFe(b

2 − a2))

~JCu =σcCuI

π (σcCua2 + σcFe(b

2 − a2))(−~uz)

~JFe =σcFeI

π (σcCua2 + σcFe(b

2 − a2))(−~uz)

b) Como se trata de um problema de geometria cilíndrica com um condutor em que L >> R, vamos aplicar alei de Ampère utilizando como caminho de integração uma linha de campo, ou seja, uma circunferêcia de raior genérico. Como temos dois materiais diferentes aplicaremos a lei de Ampère generalizada,

r < a∮

~H1 · d~ℓ =

~J · ~ndS ⇐⇒ H12πr = JCuπr2 ⇐⇒ ~H1 =

1

2JCur(−~uθ)

a < r < b

~H2 · d~ℓ =

~J · ~ndS ⇐⇒ H22πr = JCuπa2 + JFeπ(r

2 − a2)

=⇒ ~H2 =1

2r

(JCua

2 + JFe(r2 − a2)

)(−~uθ)

r > b∮

~H3 · d~ℓ =

~J · ~ndS ⇐⇒ H32πr = I ⇐⇒ ~H3 =I

2πr(−~uθ)

Como, ~B = µ ~H, vem:

r < a ~B1 = µ0~H1

a < r < b ~B2 = µ ~H2

r > b ~B3 = µ0~H3

F.Barão, L.F.Mendes Electromagnetismo e Óptica (MEEC-IST) 75

Page 82: Colecção de Problemas de Electromagnetismo e Ópticafcomp.ist.utl.pt/EO.web/Book.29_01_2014.pdf · Preâmbulo Esta colectânea de problemas resulta da experiência dos autores na

Capítulo 3

Exercício 3.20

a) Se aplicarmos a equação fundamental∮

~B · ~ndS = 0 a uma superfície cilíndrica, em que uma das tampasestá do lado do ar e a outra do lado do ferro, e fizermos a altura do cilindro tender para zero, verificamos quea diferença entre o fluxo através de cada uma das tampas é zero. Mas o produto interno do campo com anormal a cada tampa é precisamente a componente do campo perpendicular à superfície de separação entreo ar e o ferro.

b) Na linha de campo média,∮

~H · d~ℓ = I ⇐⇒∫

Fe

HFedℓ+

ar

Hardℓ = NI

De notar que se B é igual nos dois materias, H não é; na linha de campo B, só existe B ≡ B⊥ à superfíciede separação.

HFe(2πR − d) + Hard = NI ⇐⇒ BFe

µ(2πR− d) +

Bar

µ0

d = NI

=⇒ B =NI

2πR−dµ

+ dµ0

=µNI

2πR+ d(

µµ0

− 1) =

105µ0 20 10−3

2π10−2 + 200π10−6(105 − 1)

=2 × 103µ0

2π10−2 + 20π= 4 × 10−5 T = 40 µT

F.Barão, L.F.Mendes Electromagnetismo e Óptica (MEEC-IST) 76

Page 83: Colecção de Problemas de Electromagnetismo e Ópticafcomp.ist.utl.pt/EO.web/Book.29_01_2014.pdf · Preâmbulo Esta colectânea de problemas resulta da experiência dos autores na

Capítulo 3

3.4 Soluções

3.1 ~B = µ02

I R2

(R2+z2)3/2 ~uz

3.3 ~B = µ02π

Ir~uθ

3.4 ~B = 2√

2µ0π

IL~uz

3.5 F = 4 10−2 N (atractiva)

3.6 a) ~F12 = −µ02π

I1I2b(

1d− 1

d+a

)

~ux

b) ~F21 = −~F12

c) ~M = 0

3.7 a) ~F = 0, nos lados paralelos a ~B~F = ±IBℓ~n, nos dois outros lados

b)∑

i~Fi = 0

c) N = Iℓ2B

3.9 B = µ02

σωR

3.10 B = µ02σV , paralelo à superfície do plástico e per-

pendicular à sua velocidade.

3.11 ~B = µ02π

Ir~uθ

3.12 ~B = µ0nI~uz

3.13 a) ~B = µ02π

NIR

~uθ

b) ~B = µ0nI~uθ

3.14 a) ~B = µ02π

Ira2 ~uθ

b) ~B = µ02π

Ir~uθ

c) ~B = µ02π

Ir

(

1 − r2−b2

c2−b2

)

~uθ

d) B = 0

3.15 a) I = J02π3

a3

b) r < a, ~B = µ03J0r2~uθ

r > a, ~B = µ03

J0ra3~uθ

c) σ(r) = J0

V′ r

d) Idêntico ao calculado em b) para r = R. Pode-se chegar a esta conclusão utilizando o princípioda sobreposição e verificando que um condutor deraio b teria B = 0 no seu centro.

3.16 a) µ = 3, 8 µH.m−1; µr = 3

b) H = 1500 A.m−1

M = 3000 A.m−1

B = 5, 7 mT

c) JM = 3000 A.m−1

3.21 a) ~JM = 105~uθ A.m−1

c) ~B = 6, 3~uz mT

F.Barão, L.F.Mendes Electromagnetismo e Óptica (MEEC-IST) 77

Page 84: Colecção de Problemas de Electromagnetismo e Ópticafcomp.ist.utl.pt/EO.web/Book.29_01_2014.pdf · Preâmbulo Esta colectânea de problemas resulta da experiência dos autores na

Capítulo 3

F.Barão, L.F.Mendes Electromagnetismo e Óptica (MEEC-IST) 78

Page 85: Colecção de Problemas de Electromagnetismo e Ópticafcomp.ist.utl.pt/EO.web/Book.29_01_2014.pdf · Preâmbulo Esta colectânea de problemas resulta da experiência dos autores na

Capítulo 4

Capítulo 4

Partículas e campos

4.1 Introdução

As propriedades da força eléctrica já foram estudadas no capítulo da electrostática.Recapitulemos os aspectos relevantes da sua acção sobre uma partícula carregada.Na presença de um campo eléctrico ~E uma partícula de carga q sofre uma forçadescrita por

~F = q ~E (4.1)

O trabalho que esta força realiza sobre a carga ao longo de um percurso entre doispontos a e b é

We =

∫ b

a

~F · ~dℓ = q

∫ b

a

~E · ~dℓ = qVab (4.2)

Ou seja, o campo elécrico realiza um trabalho sobre as cargas que depende apenasda diferença de potencial aplicada e que vai acelerar as cargas.

Força eléctrica sobre umapartícula carregada

No capítulo da magnetostática verificou-se que a força que um campo magnético ~Bexercia sobre um elemento de corrente I ~dℓ é dada pela expressão de Laplace:

~dF = I ~dℓ × ~B (4.3)

Esta é a força exercida sobre um conjunto de cargas em movimento existentes noelemento dℓ. A questão que agora se põe é saber qual a força exercida sobre umaúnica carga.A um elemento dℓ corresponde um elemento de volume dV = Sdℓ, em que S éa secção onde passa a corrente eléctrica. O número de partículas carregadas numelemento de volume dV será então Np = NdV em que N é a densidade deportadores de carga, ou seja, o número de cargas existentes por unidade de volume.A força que o campo magnético exerce obre uma partícula carregada será então aforça que o campo magnético exerce sobre um elemento de corrente a dividir pelonúmero de particulas carregadas existentes nesse elemento de corrente:

Força magnética sobre umapartícula carregada

F.Barão, L.F.Mendes Electromagnetismo e Óptica (MEEC-IST) 79

Page 86: Colecção de Problemas de Electromagnetismo e Ópticafcomp.ist.utl.pt/EO.web/Book.29_01_2014.pdf · Preâmbulo Esta colectânea de problemas resulta da experiência dos autores na

Capítulo 4

~F =I ~dℓ

Np

× ~B =~JSdℓ

NSdℓ× ~B =

Nq~v

N× ~B = q~v × ~B (4.4)

Uma primeira propriedade desta força magnética pode ser encontrada decompondo avelocidade numa componente paralela ao campo magnético, ~v||, e numa componenteperpendicular ao campo magnético, ~v⊥. A equação da força magnética pode entãoser reescrita na forma

~F = q(~v|| + ~v⊥

)× ~B = q

(

0 + ~v⊥ × ~B)

= q~v⊥ × ~B (4.5)

Isto significa que a força magnética depende apenas da componente da velocidadeperpendicular ao campo.

Vamos agora calcular o trabalho que esta força realiza sobre a carga ao longo deum percurso entre dois pontos a e b:

Wm =

∫ b

a

~F · ~dℓ = q

∫ tb

ta

(

~v × ~B)

~vdt = 0 (4.6)

Podemos então concluir que a força magnética não realiza trabalho sobre as par-tículas carregadas. Isto significa que, em todos os instantes, a força magnética éperpendicular ao deslocamento da partícula, alterando a direcção da sua velocidademas não o seu módulo (a energia cinética mantém-se constante). Por outras pala-vras, a força magnética é uma força centrípeta que obriga as partículas a curvaremas suas trajectórias.

O raio de curvatura da trajectória de uma partícula carregada sujeita a um campomagnético pode ser encontrado igualando a expressão da força magnética à expressãode uma força centrípeta (segundo a direcção radial de um sistema de coordenadas):

qvB⊥ = mv2

R

R =mv

qB⊥

(4.7)

Uma partícula carregada que se mova com energia cinética constante na presençade uma campo magnético terá, em geral, uma trajectória que é uma hélice. Estatrajectória helicoidal resulta da sobreposição de uma trajectória que é uma circunfe-rência, devido à sua velocidade perpendicular ao campo magnético, com um trajec-tória linear, devido à sua velocidade paralela ao campo magnético que se mantéminalterada.

Trajectória de umapartícula carregada esujeita a um campo

magnético

F.Barão, L.F.Mendes Electromagnetismo e Óptica (MEEC-IST) 80

Page 87: Colecção de Problemas de Electromagnetismo e Ópticafcomp.ist.utl.pt/EO.web/Book.29_01_2014.pdf · Preâmbulo Esta colectânea de problemas resulta da experiência dos autores na

Capítulo 4

A força de Lorentz não é mais que o princípio da sobreposição aplicado a umapartícula que se move num campo eléctrico e num campo magnético:

~F = q(

~E + ~v × ~B)

(4.8)

A equação foi desenvolvida por Lorentz em 1895 e, apesar da sua aparência simples,foi um avanço científico significativo para a época. Havia já 10 anos que se co-nheciam as ondas electromagnéticas, corolário do trabalho de "finalização"da teoriado electromagnetismo de Maxwell, mas ainda não existiam partículas em física. Aforça de Lorentz foi confirmada pelos trabalhos de Thomson dois anos mais tarde,em 1897.

A força de Lorentz

F.Barão, L.F.Mendes Electromagnetismo e Óptica (MEEC-IST) 81

Page 88: Colecção de Problemas de Electromagnetismo e Ópticafcomp.ist.utl.pt/EO.web/Book.29_01_2014.pdf · Preâmbulo Esta colectânea de problemas resulta da experiência dos autores na

Capítulo 4

4.2 Exercícios Resolvidos

4.2.1 Partículas aceleradas

Num cinescópio de uma televisão são acelerados electrões, a partir do es-tado de repouso, por uma diferença de potencial de 20 kV.Para além do campo eléctrico, os electrões estão também sujeitos ao campomagnético terrestre. Vamos admitir que a componente do campo magnéticoterrestre que é perpendicular ao movimento dos electrões é B⊥ = 30 µT1.

• A velocidade atingida pelos electrõesO trabalho realizado pelo campo eléctrico é integralmente convertido emenergia cinética dos electrões, que partem de um estado de energia cinéticanula. Deste modo

W =

~F · ~dℓ = q

~E · ~dℓ = q V

= 1, 6 × 10−19 × 20 × 103 = 3, 2 × 10−15 J

W =1

2mv2 ⇔ v =

2W

m

v =

2 × 3, 2 × 10−15

9, 1 × 10−31= 0, 28 c

• Raio da trajectória dos electrões devido ao campo magnético terres-treO campo magnético terrestre não realizará trabalho mas exercerá uma forçacentripta que fará com que os electrões tenham uma trajectória curva. Va-mos calcular o raio da trajectória quando os electrões atingem a sua ve-locidade máxima. A relação entre o raio da trajetória e a velocidade doselectrões pode encontrar-se igualando o módulo da força magnética à forçacentripta:

qvB⊥ = mv2

R⇔ R =

mv

qB⊥

R =9, 1 × 10−31 × 0, 28 × 3 × 108

1, 6 × 10−19 × 30 × 10−6= 16 m

F.Barão, L.F.Mendes Electromagnetismo e Óptica (MEEC-IST) 82

Page 89: Colecção de Problemas de Electromagnetismo e Ópticafcomp.ist.utl.pt/EO.web/Book.29_01_2014.pdf · Preâmbulo Esta colectânea de problemas resulta da experiência dos autores na

Capítulo 4

4.2.2 Condutor em movimento

Um avião, com uma envergadura de asas de 40 m, desloca-se horizontal-mente com uma velocidade de cruzeiro de ~v = 900 ~uy km.h−1. No local em quese encontra, o campo magnético terrestre tem uma componente perpendicular àsua velocidade de B⊥ = −35 ~uz µT.

• A força magnética que actua sobre os electrõesOs electrões livres do avião deslocam-se com a sua velocidade e estãosujeitos a uma força magnética dada por

~Fm = q~v × ~B⊥ = −qvB⊥ ~ux

~Fm = evB⊥ ~ux

Onde e representa a carga unitária (positiva). Conclui-se então que oselectrões são "puxados"para uma das pontas da asa, ficando naturalmenteem falta na outra ponta. Esta distribuição de carga cria por sua vez umcampo eléctrico que actua sobre os electrões através de uma força que tema mesma direcção mas sentido contrário ao da força magnética.

• O campo eléctrico que actua sobre os electrõesEnquanto a força magnética que actua os electrões livre do metal for maiorque a força eléctrica devida à distribuição de carga os electrões continuarãoa ser "puxados"para a ponta da asa, aumentando a distrbuição de carga.Deste modo, o campo eléctrico irá aumentando até que a força eléctricaiguale em módulo a força magnética. A partir desse momento a força totalsobre os electrões é nula e atinge-se um equilíbrio electrostático. A partirda condição de equilíbrio obtemos:

~Fe + ~Fm = 0 ⇔ ~Fe = −~Fm

~Fe = −evB⊥ ~ux = qvB⊥ ~ux

O campo eléctrico é a força por unidade de carga:

~E =~Fe

q= vB⊥ ~ux

• A diferença de potencial entre as suas extremidadesA diferença de potencial entre as pontas das asas do avião pode ser calcu-lada a partir do campo eléctrico:

V =

∫ L

0

~E · ~dℓ =

∫ 40

0

vB⊥ dx = 40× vB⊥

V = 40900× 103

360035 × 10−6 = 0, 35 V

F.Barão, L.F.Mendes Electromagnetismo e Óptica (MEEC-IST) 83

Page 90: Colecção de Problemas de Electromagnetismo e Ópticafcomp.ist.utl.pt/EO.web/Book.29_01_2014.pdf · Preâmbulo Esta colectânea de problemas resulta da experiência dos autores na

Capítulo 4

4.2.3 Efeito de Hall

Uma sonda de efeito de Hall é utilizada para medir um campo magnéticocriado por uma bobine. A sonda tem uma secção quadrada de lado ℓ = 1 mm e éconstituída por um material semi-condutor cujos portadores de carga são lacunas,com uma densidade N = 1026 lacunas.m−3. Quando é injectada uma correntede 10 A na sonda, o valor da diferença de potencial máxima medida entre as duasfaces da sonda é V = 5 µV. Pretende-se determinar o campo magnético criadopela bobine.

• A direcção do campo magnéticoA tensão máxima que é possível medir com a sonda corresponde ao casoem que o campo é perpendicular à sonda: as cargas que se movem devidoà corrente eléctrica sofrem uma força que é proporcional à componente docampo magnético que é perpendicular ao seu movimento. Deste modo,movendo a sonda até se atingir a leitura máxima no voltímetro podemosficar a saber qual a direcção do campo.

• O módulo do campo magnéticoTal como no exemplo anterior, também neste caso a situação de equilíbrioserá alcançada quando os módulos das forças eléctrica e magnética apli-cadas às lacunas forem iguais. Note-se que neste caso, ao contrário doexemplo do avião, a sonda está parada e é a corrente eléctrica que moveas cargas:

Fe = Fm ⇔ E =Fe

q=

Fm

q= vB

V =

∫ ℓ

0

Edℓ = ℓvB

B =V

vℓ=

V

Nq

J=

V

Nqℓ2

I=

NqV ℓ

I

B =1026 × 1, 6 × 10−19 × 5 × 10−6 × 10−3

10=

= 80 mT

F.Barão, L.F.Mendes Electromagnetismo e Óptica (MEEC-IST) 84

Page 91: Colecção de Problemas de Electromagnetismo e Ópticafcomp.ist.utl.pt/EO.web/Book.29_01_2014.pdf · Preâmbulo Esta colectânea de problemas resulta da experiência dos autores na

Capítulo 4

4.3 Exercícios Propostos

Exercício 4.1 : Num ciclotrão (acelerador de partí-culas), partículas carregadas são sujeitas a um campomagnético B perpendicular à sua velocidade, sendo asua trajectória circular. Para além do campo magné-tico, ao fim de cada semi-volta aplica-se às partículasuma tensão sinusoidal dada por V (t) = V0sen(ωt).O campo eléctrico aplicado às partículas aumenta a suavelocidade, deixando a trajectória de ser circular e pas-sando a consistir em troços semicirculares de raio cadavez maior. Neste exercício pretende-se dimensionar umciclotrão que acelera partículas alfa (núcleos de átomosde hélio que possuem 2 protões e 2 neutrões).

a) Para que haja sincronia entre o efeito da acelera-ção do campo eléctrico e o movimento de rotaçãodas partículas, qual deve ser a frequência da ten-são sinusoidal aplicada (em função do campo B,da carga q e da massa m das partículas)?

b) Suponha que a frequência da tensão sinusoidalV (t) = V0sen(ωt) é de 10 kHz. Qual deveser então, de acordo com a alínea a), o valor docampo magnético aplicado?

c) Qual é a energia ganha por uma partícula emcada volta completa se lhe for aplicada a ampli-tude máxima da tensão, V0?

d) Suponha que o raio da órbita de extracção (raioda última volta) é R = 1m. Qual a energiacinética com que saem do ciclotrão as partículasalfa?

e) Suponha que a partícula que menos tempo per-manece no ciclotrão percorre 12 voltas no seuinterior. A partir das alíneas c) e d) calcule adiferença de potencial máxima V0 que é aplicadaàs partículas alfa.

Exercício 4.2 : Um feixe pulsado (bunch) de 1010 elec-trões inicialmente em repouso é acelerado numa zona 1por aplicação de uma diferença de potencial de 20 kV.

Seguidamente, após entrar numa zona 2, o feixe é sub-metido à acção de um campo magnético perpendicularà sua velocidade.

a) Calcule a velocidade do feixe de electrões quandosai da zona 1.

b) Obtenha a intensidade do campo magnético apli-cado na zona 2, sabendo que o feixe passa a teruma trajectória circular de raio R = 12 cm.

c) Calcule a intensidade da corrente eléctrica I cri-ada pelo movimento circular do feixe electrónicona zona 2.

d) Calcule a intensidade do campo magnético cri-ado pela corrente I no centro da trajectória dofeixe electrónico e compare-a com a intensidadedo campo magnético aplicado.

Exercício 4.3 : Uma corrente marinha tem uma velo-cidade de v = 1 m.s−1 numa zona em que a compo-nente vertical do campo magnético terrestre tem umaamplitude B⊥ = 3, 5 × 10−5 T. Sabendo que a con-dutividade eléctrica da água do mar é σc = 0, 04Ω−1.cm−1, determine a densidade de corrente eléctricaperpendicular à direcção da corrente marítima.

Exercício 4.4 : Uma barra metálica de comprimentoℓ = 1 m move-se com uma velocidade ~v = 10 ~ux

m.s−1 numa zona onde existe, perpendicularmente àsua velocidade, um campo magnético ~B = −~uz mT.

lv

. X

y

B

a) Determine o módulo e o sentido da força mag-nética que actua nos electrões de condução dabarra.

F.Barão, L.F.Mendes Electromagnetismo e Óptica (MEEC-IST) 85

Page 92: Colecção de Problemas de Electromagnetismo e Ópticafcomp.ist.utl.pt/EO.web/Book.29_01_2014.pdf · Preâmbulo Esta colectânea de problemas resulta da experiência dos autores na

Capítulo 4

b) Calcule a diferença de potencial entre as extremi-dades da barra após se ter atingido o equilíbrio.

c) Se os extremos da barra fossem ligados com umcondutor de resistência 100 Ω, solidário com abarra, qual seria a corrente no circuito?

Exercício 4.5 : Numa experiência de efeito de Hall,uma corrente de intensidade I = 10 A percorre umcondutor de secção quadrada com ℓ = 0, 5 cm de lado.Um campo magnético transversal ao condutor de ampli-tude B = 2 T induz uma tensão V = 2, 5 × 10−4

V. Supondo que os portadores de carga são electrões,calcule a densidade destes no condutor.

Exercício 4.6 : O primeiro gerador de corrente foi inven-tado por Faraday em 1831 e consiste num disco metálicoque é posto a rodar na presença de um campo magnéticoperpendicular à sua superfície. Após se fechar um inter-ruptor, o centro do disco e a sua periferia ficam ligadospor um circuito imóvel que é percorrido por corrente. Odisco,que possui raio a e espessura b, é posto a rodarcom uma velocidade angular ω e encontra-se sujeito aum campo magnético ~B = B0~uz.

a) Determine a expressão da força a que ficam sujei-tos os electrões livres do metal devido ao campomagnético,~Fm.

b) Determine a expressão do campo electrostáticodentro do disco, devido à distribuição de cargado disco, após se ter atingido o equilíbrio elec-trostático, ~E.

c) Determine a densidade de carga no interior dodisco e na sua superfície exterior, em equilíbrioelectrostático.

d) Determine a diferença de potencial (V ) a quevai ficar sujeita a resistência R após se fechar ointerruptor.

e) Determine a potência mecânica fornecida ao discopara o manter a rodar com velocidade angularconstante, após se fechar o interruptor e a re-sistência R passar a ser percorrida por uma cor-rente.

Exercício 4.7 : Considere um condutor de cobre muitocomprido, cilíndrico e de raio R = 1 mm, que trans-porta uma corrente estacionária e uniformemente distri-buída I = 10 A (segundo −~uz). Vamos verificar atéque ponto a lei de Ohm é válida (ignorando-se as forçasmagnéticas) no período transitório em que se estabelecea corrente eléctrica.

a) Calcule o campo magnético (B) criado pela cor-rente eléctrica dentro do condutor.

b) Calcule a força por unidade de carga a que ficamsujeitos os electrões devido ao campo magnético.Descreva sucintamente, de acordo com o resul-tado obtido, o que se passa no condutor no pe-ríodo transitório em que se estabelece a correntee como se atinge o regime estacionário.Sugestão: escreva primeiro a velocidade dos elec-trões (v) em função de ~J , de Ncobre e de q.

c) Calcule a força por unidade de carga a que ficamsujeitos os electrões devido ao campo electros-tático que cria a corrente. Compare o resultadoobtido com o da alínea b) e conclua da impor-tância do período transitório.

Exercício 4.8 : O caudalímetro electromagnético repre-sentado na figura está ser utilizado para medir o caudalvolumétrico (volume por unidade de tempo) de um lí-quido que contém iões NaCl+ (q = 1, 6 × 10−19 C).

F.Barão, L.F.Mendes Electromagnetismo e Óptica (MEEC-IST) 86

Page 93: Colecção de Problemas de Electromagnetismo e Ópticafcomp.ist.utl.pt/EO.web/Book.29_01_2014.pdf · Preâmbulo Esta colectânea de problemas resulta da experiência dos autores na

Capítulo 4

O líquido passa numa zona do tubo de secção quadran-gular em que existe um campo magnético uniforme cri-ado por um íman (de Norte para Sul). A diferença depotencial que surge entre as paredes do tubo é medidacom um voltímetro. No presente caso o líquido tem umcaudal volumétrico V = 2 L.s−1, o campo magnéticotem uma intensidade B = 2 mT, o tubo tem de ladod = 2 cm e a densidade de iões é N = 2 × 1010

iões.m−3.

a) Calcule a velocidade dos iões, v, e a intensidadede corrente eléctrica no tubo, I.

b) Calcule a força magnética que actua sobre cadaião, Fm.

c) Calcule a diferença de potencial medida pelo vol-tímetro, V , e indique a sua polaridade.

d) Diga, justificando a sua resposta, como se alte-rava a medida do voltímetro se se tratasse deum outro líquido com o mesmo caudal mas comoutro tipo e/ou densidade de iões.

F.Barão, L.F.Mendes Electromagnetismo e Óptica (MEEC-IST) 87

Page 94: Colecção de Problemas de Electromagnetismo e Ópticafcomp.ist.utl.pt/EO.web/Book.29_01_2014.pdf · Preâmbulo Esta colectânea de problemas resulta da experiência dos autores na

Capítulo 4

4.4 Soluções

4.1 a) f = qB2πm

b) B = 1, 3 mT

c) ∆T = 2qV0 =

d) T = 1, 3 × 10−17 J

e) V0 = 1, 7 V

4.2 a) v = 0, 28 c

b) B = 4 mT

c) I = 178 mA

d) B = 0, 9 µT

4.3 J = 1, 4 × 10−4 A.m−1

4.4 a) ~F = −1, 6 × 10−21 ~uy [N]

b) V = 10 mV

b) I = 0

4.5 N = 1026 electrões.m−3

4.6 a) ~Fm = −eωrB0~ur

b) ~E = −ωrB0~ur

c) ρ = −2ε0ωB0

σ = +ε0ωB0a

d) V = 12ωB0a2

e) P =ω2B2

0a4

4R

4.7 a) ~B = −2 r ~uθ

b) ~Fm/q = 0, 4 × 10−3r ~ur N.C−1

c) ~Fe/q = 5 × 10−2 ~uz V.m−1

Fmaxm /q = 4 × 10−7 V.m−1 << Fe/q

4.8 a) v = 5 m.s−1 ; I = 6, 4 × 10−12 A

b) Fm = 16 × 10−22 N

c) V = 0, 2 mV, da face posterior para a face ante-rior do tubo.

d) Não se alterava. Este é um dos interesses destescaudalímetros (até a água da torneira tem iões!).

F.Barão, L.F.Mendes Electromagnetismo e Óptica (MEEC-IST) 88

Page 95: Colecção de Problemas de Electromagnetismo e Ópticafcomp.ist.utl.pt/EO.web/Book.29_01_2014.pdf · Preâmbulo Esta colectânea de problemas resulta da experiência dos autores na

Capítulo 5

Capítulo 5

Campo Magnético Variável

F.Barão, L.F.Mendes Electromagnetismo e Óptica (MEEC-IST) 89

Page 96: Colecção de Problemas de Electromagnetismo e Ópticafcomp.ist.utl.pt/EO.web/Book.29_01_2014.pdf · Preâmbulo Esta colectânea de problemas resulta da experiência dos autores na

Capítulo 5

5.1 Exercícios Propostos

Exercício 5.1 : Um circuito de área A e resistênciaeléctrica R encontra-se numa zona do espaço em queexiste um campo magnético que lhe é perpendicular eque é uniforme. O campo tem uma variação temporaldescrita por B(t) = B0e

−αt. Determine a intensidadeda corrente eléctrica que percorre o circuito.

Exercício 5.2 : Para medir um campo magnético queexiste apenas numa zona confinada do espaço cuja sec-ção é um quadrado de lado ℓ, colocou-se em seu redoruma espira condutora quadrada também de lado ℓ = 1cm, cujos quatro lados têm a mesma resistência. Ocampo magnético teve uma variação temporal expressapor B = B0t Tesla e a diferença de potencial entredois vértices consecutivos da espira foi medida por doisvoltímetros, VA e VB , de acordo com a figura.

a) Determine a expressão da força electromotriz in-duzida na espira quadrada e indique o sentido dacorrente induzida.

b) Sabendo que a tensão medida por VA é 5 µV,calcule o valor de B0.

c) Qual é o valor medido pelo voltímetro VB?

Exercício 5.3 : Um circuito quadrado de resistênciaR = 20 Ω e de lado ℓ = 0, 2 m roda 100 vezes porsegundo em torno de um eixo horizontal que o divide aomeio. Existe no local em que se encontra o circuito umcampo magnético uniforme, de intensidade B = 1 Te perpendicular à posição ocupada pelo circuito quandot = 0 s. Determine:

a) o fluxo do campo magnético através da espira,em função do tempo.

b) a corrente induzida.

c) a energia dissipada na espira, por efeito de Joule,ao fim de 2 minutos.

Exercício 5.4 : Dois carris condutores paralelos entresi, que se encontram a uma distância d, estão unidosnuma das extremidade por um condutor. A resistênciaequivalente deste sistema é R. Uma barra condutora deresistência desprezável desliza apoiada nos carris comuma velocidade constante ~v = v0~ux, sem atrito, sobacção de uma força exterior. Existe um campo ~B =B~uz uniforme em toda a região ocupada pelo sistema.No instante inicial a barra encontra-se na posição X0.Determine:

Rv

d

a) a intensidade e sentido da corrente induzida nocircuito.

b) a potência dissipada por efeito de Joule.

c) a força que o campo ~B exerce sobre a barra mó-vel.

d) a potência correspondente ao trabalho da forçaaplicada sobre a barra para a movimentar.

Exercício 5.5 : Considere o circuito da figura, consti-tuído por uma fonte de tensão variável V , uma resis-tência R e uma barra móvel de comprimento ℓ e demassa m. O circuito está colocado na vertical, estandoa barra sujeita à aceleração da gravidade g. Perpendi-cularmente ao circuito existe um campo magnético uni-forme de intensidade ~B = B~uz. No instante inicial abarra encontra-se na posição y0.

F.Barão, L.F.Mendes Electromagnetismo e Óptica (MEEC-IST) 90

Page 97: Colecção de Problemas de Electromagnetismo e Ópticafcomp.ist.utl.pt/EO.web/Book.29_01_2014.pdf · Preâmbulo Esta colectânea de problemas resulta da experiência dos autores na

Capítulo 5

a) Admita que a barra tem uma velocidade segundoy (para baixo) ~v = v0~uy. Determine a inten-sidade e sentido da corrente que percorre o cir-cuito.

b) Admita que a barra é deixada cair sem velocidadeinicial. Determine a tensão que terá de ser im-posta pela fonte em cada instante, V (t), paraque não exista corrente no circuito (indique tam-bém a polaridade da fonte).

c) Admita que a barra é largada sem velocidade ini-cial. Determine a tensão que terá de ser impostapela fonte, V, para que a barra fique suspensasem cair (indique também a polaridade da fonte).

d) Admita agora que a barra é largada com umavelocidade inicial v0. Se a tensão imposta aocircuito for semelhante à da alínea c) que tipo demovimento terá a barra? Justifique qualitativa-mente.

Exercício 5.6 : Uma bobina muito comprida, com umdiâmetro D = 20 cm e uma densidade de espirasn = 1000 espiras.m−1, é percorrida por uma correnteeléctrica de intensidade I. Em torno do seu eixo ver-tical existe um anel de um material condutor com umdiâmetro D

= 40 cm.

a) Calcule a força electromotriz induzida no anelquando a corrente na bobina varia de I1 = 10A para I2 = 1 A numa décima de segundo.

b) Se o anel tiver uma secção S = 1 cm2 e umacondutividade σc = 6 × 108 Ω−1.m−1, qual acorrente que o percorre?

c) Qual a resposta à alínea b) se o anel tiver 1 mde diâmetro? É importante que os eixos estejamcoincidentes?

Exercício 5.7 : Um amperímetro clip-on é um disposi-tivo para medir correntes alternadas em cabos sem terque cortar o cabo. Consiste num enrolamento em tornode um núcleo metálico de permeabilidade magnética µcom a forma de um anel cilíndrico, de secção quadrada.O dispositivo é colocado de modo a que a normal quepassa pelo centro do anel esteja alinhada com o fio e pos-sui um voltímetro que mede a diferença de potencial aosterminais do enrolamento. Considere um destes dispo-sitivos com um anel de raios R1 = 10 cm e R2 = 11cm, uma espessura d = 2 cm e um enrolamento deN = 5000 espiras.

a) Determine a expressão do campo magnético pro-duzido pelo fio no núcleo metálico, em função dacorrente I que o percorre.

b) Determine a expressão do fluxo do campo mag-nético criado pelo fio no enrolamento.

c) Calcule a diferença de potencial medida pelo vol-tímetro, sabendo que a corrente que passa pelofio é I = 16cos(100πt) A e que o núcleo metá-lico tem uma permeabilidade magnética relativaµr = 1000.

Exercício 5.8 : Determine o coeficiente de auto-induçãoexterno por unidade de comprimento de dois fios con-dutores infinitos de raio R = 1 mm cujos centros seencontram a uma distância d = 1 cm um do outro.

Exercício 5.9 : Considere uma bobina de comprimentoℓ e raio R em que ℓ >> R, com n espiras por unidadede comprimento. Esta é percorrida por uma correntevariável I = I0cos(ωt). Determine:

a) o coeficiente de auto-indução da bobina.

b) a força electromotriz induzida num anel condutorconcêntrico com a bobina, de raio r < R.

c) o campo eléctrico existente num ponto P a umadistância r < R do eixo da bobina.

F.Barão, L.F.Mendes Electromagnetismo e Óptica (MEEC-IST) 91

Page 98: Colecção de Problemas de Electromagnetismo e Ópticafcomp.ist.utl.pt/EO.web/Book.29_01_2014.pdf · Preâmbulo Esta colectânea de problemas resulta da experiência dos autores na

Capítulo 5

Exercício 5.10 : Na figura que se segue está represen-tado um transformador de núcleo circular e de secçãoquadrada. Este núcleo é constituído por um material fer-romagnético linear de permeabilidade µ. Os enrolamen-tos primário e secundário são atravessados por correntesI1 e I2 e possuem N1 e N2 espiras, respectivamente.Determine:

a) o campo magnético ( ~B) no interior do núcleo;

b) o coeficiente de indução mútua entre os enrola-mentos;

c) as forças electromotrizes induzidas em cada umdos enrolamentos, ε1 e ε2, em função do fluxo docampo magnético (Φ) que atravessa uma secçãodo núcleo e do número de espiras. Calcule a razãoentre as tensões no circuito primário e secundáriodo transformador.

Exercício 5.11 : Considere uma bobina de comprimentoℓ e diâmetro D em que ℓ >> D, com n espiras porunidade de comprimento e um núcleo de ar, possui umaresistência R. Em torno da bobina existe uma espiraquadrada de lado a que é percorrida por uma correnteI = I0cos(ωt). Determine:

a) o coeficiente de indução mútua entre a espira eo solenóide;

b) a expressão da equação diferencial que permitedeterminar a corrente induzida no solenóide.

Exercício 5.12 : Considere uma bobina de comprimentoℓ, raio R1, N1 espiras e núcleo de ar, percorrida por

uma corrente I = I0e−at. Esta bobina está colo-

cada dentro de uma segunda bobina de comprimentoℓ, de raio R2, N2 espiras e de resistência R. Os eixosdas duas bobinas estão coincidentes e considera-se queℓ >> R2.

a) Determine o coeficiente de indução mútua do sis-tema das duas bobinas.

b) Determine a corrente induzida na bobina exterior.

Exercício 5.13 : Considere um condutor rectilíneo decomprimento infinito e secção circular de raio a, comuma permeabilidade magnética µ0 e estando a ser per-corrido por uma corrente eléctrica estacionária de inten-sidade I1. Determine:

a) a densidade de energia magnética no interior docondutor;

b) a energia magnética do condutor por unidade decomprimento;

c) o coeficiente de auto-indução interno do condu-tor por unidadede comprimento. Compare com oresultado do exercício 86;

Exercício 5.14 : Considere um cabo coaxial condutorrectilíneo de comprimento infinito e raios a e b, em queo espaço entre os condutores está preenchido com ar.No caso de o cabo ser percorrido por uma corrente I,determine:

a) a densidade de energia magnética no espaço entreos condutores;

b) a energia magnética por unidade de comprimento,no espaço entre os condutores;

c) O coeficiente de auto-indução do cabo, por uni-dade de comprimento.

Exercício 5.15 : As bobinas projectadas para camposmagnéticos fortes têm problemas mecânicos de constru-ção devido às pressões a que ficam sujeitas. Considereuma bobina de comprimento ℓ e raio R (ℓ >> R),com n espiras por unidade de comprimento, núcleo dear e percorrida por uma corrente I. Determine:

a) a densidade de energia magnética armazenada nointerior da bobina;

b) a energia magnética armazenada na bobina;

c) o coeficiente de auto-indução da bobine, a partirda energia magnética;

d) qualitativamente se a bobine fica sujeita a umaforça de implosão ou de explosão;

F.Barão, L.F.Mendes Electromagnetismo e Óptica (MEEC-IST) 92

Page 99: Colecção de Problemas de Electromagnetismo e Ópticafcomp.ist.utl.pt/EO.web/Book.29_01_2014.pdf · Preâmbulo Esta colectânea de problemas resulta da experiência dos autores na

Capítulo 5

e) a pressão sobre os enrolamentos em função docampo magnético, B.

Exercício 5.16 : Uma bobina de N espiras colocada navertical, tem um comprimento ℓ e um raio a (ℓ >> a)e é percorrida por uma corrente estacionária I1. No seuinterior é colocada uma espira de raio b < a cujo eixofaz um ângulo θ com o eixo da bobina.

a) Determine o coeficiente de auto-indução (L) dabobina.

b) Determine o coeficiente de indução mútua (M)do sistema.

c) Desprezando a auto-indução da espira, determinea expressão da energia magnética do sistema quandoa espira também é percorrida por uma correnteestacionária I2 com o mesmo sentido de I1.

d) Determine o momento da força que actua a es-pira. Identifique o ponto de equilíbrio estável.Nota: Recorde que a derivada de uma energiaem ordem a um ângulo não é uma força mas simo momento de uma força.

Exercício 5.17 : Considere duas espiras circulares pa-ralelas e alinhadas coaxialmente com raios a e b cujosplanos estão distanciados de z, a serem percorridas porcorrentes eléctricas Ia e Ib. Admitindo que uma dasespiras é muito mais pequena que a outra (a << b) eque a distância a que se encontram seja grande quandocomparada com os seus raios (z >> a, b), determine:

a) o coeficiente de indução mútua do sistema emfunção dos sentidos das correntes;

b) designando os coeficientes de auto-indução dasespiras por La e Lb, a energia magnética do sis-tema;

c) a força existente entre as espiras em função dossentidos das correntes; utilize o resultado paradiscutir qualitativamente as forças entre ímanes.

Exercício 5.18 : Considere um enrolamento de compri-mento ℓ, raio R2 (ℓ >> R2) e densidade de espirasn, percorrido por uma corrente I2. No seu interior, co-locado coaxialmente existe um segundo enrolamento decomprimento ℓ, raio R1, a mesma densidade de espirasn e percorrido por uma corrente I1 que tem o mesmosentido de I2. Este segundo enrolamento possui um nú-cleo de material ferromagnético, que nas condições defuncionalidade descritas, tem uma permeabilidade mag-nética µ. Determine:

a) o campo magnético existente no várias regiõesinteriores aos enrolamentos: r > R2, R1 <r < R2 e r < R1;

b) os coeficientes de auto-indução dos dois enro-lamentos e o coeficiente de indução mútua dosistema composto pelos dois enrolamentos;

c) a energia magnética do sistema;

d) a pressão aplicada sobre cada um dos enrolamen-tos;

Exercício 5.19 : Um electroíman é constituído por umenrolamento de espiras em torno de um núcleo ferro-magnético com a forma indicada na figura e cuja secçãoS = 400 cm2. Admitindo que a distância entre o ob-jecto a elevar e o núcleo ferromagnético seja pequena,pode-se considerar uniforme o campo magnético exis-tente nessa região e de valor B = 1 T. Dtermine:

x

I

d

a) a energia magnética existente no espaço entreo núcleo ferromagnético e o objecto que se pre-tende elevar, em função da distância (x) que se-para o electroíman do objecto;

F.Barão, L.F.Mendes Electromagnetismo e Óptica (MEEC-IST) 93

Page 100: Colecção de Problemas de Electromagnetismo e Ópticafcomp.ist.utl.pt/EO.web/Book.29_01_2014.pdf · Preâmbulo Esta colectânea de problemas resulta da experiência dos autores na

Capítulo 5

b) a força exercida pelo electroíman sobre o objectoa elevar.

Exercício 5.20 : Considere uma bobina de raio R, com-primento ℓ >> R e densidade de espiras n, preenchidacom ar.

a) Determine o campo magnético (B) no interior dabobina se esta for percorrida por uma corrente I.

Suponha agora que a bobina está parcialmente preen-chida com um núcleo de material ferromagnético de raioR e permeabilidade magnética µ (como um relé). De-termine:

b) o coeficiente de auto-indução da bobina;Nota: para efeitos de cálculo, considere comoválida a aproximação da bobina infinita quer dolado do material ferromagnético quer do lado doar.

c) a potência média fornecida pela fonte ao sistemapara estabelecer a corrente no circuito (partindoda situação I = 0) num intervalo de tempo ∆t;despreze as resistências dos condutores;

d) a força a que fica sujeito o núcleo ferromagnéticoapós se ter estabelecido a corrente I.

F.Barão, L.F.Mendes Electromagnetismo e Óptica (MEEC-IST) 94

Page 101: Colecção de Problemas de Electromagnetismo e Ópticafcomp.ist.utl.pt/EO.web/Book.29_01_2014.pdf · Preâmbulo Esta colectânea de problemas resulta da experiência dos autores na

Capítulo 5

5.2 Resoluções

Exercício 5.2

a) A força electromotriz:

ε = − d

dt

~B · ~ndS = − d

dt(BA) = −A

dB

dt= −ℓ2

d

dt(B0t) = −ℓ2B0

Note-se que esta força electromotriz só existe durante 1s. Como utilizámos uma normal com o sentido de ~Be obtivemos uma força electromotriz com um valor negativo, a corrente tem o sentido contrário ao que nosdaria a regra da mão direita em relação à normal, ou seja, tem o sentido anti-horário .Em alternativa, a Lei de Lenz indica-nos o sentido da corrente induzida. O sentido da corrente eléctrica é talque o campo magnético por si criado deve ser no sentido de se opôr à variação do fluxo de campo magnéticoque atravessa o circuito. Como o fluxo aumenta no tempo (porque o campo B aumenta) a corrente tem quepossuir o sentido anti-horário de forma a criar um campo B que possui um sentido contrário no interior docircuito, ao campo externo.

b) Da Lei de Faraday tem-se que:∮

~E · d~ℓ = −dφB

dt

A circulação do campo eléctrico no circuito que possui o voltímetro VA, admitindo que os condutores queligam o voltímetro à secção da espira quadrada são perfeitos (R = 0, E = 0), é dado por:∮

~E · d~ℓ =∫

VA

~E · d~ℓ+∫

ℓ~E · d~ℓ = −dφB

dt

O integral anterior expressa a queda de tensão no voltímetro VA e no ℓ troço da espira quadrada. Neste casoo circuito nao possui nehum componente com indutância relevante (note-se que a indutância do circuito édesprezável) nem campo magnético exterior a atravessá-lo por isso dφB

dt= 0. Resulta então:

VA + VDA = 0 ⇒ VA = −VDA = RℓI = Rℓε

4Rℓ= ℓ2B0

4

Como os quatro lados da espira têm igual resistência eléctrica, cada lado terá uma queda de tensão corres-pondente a um quarto da força electromotriz induzida total. O voltímetro VA está a medir a diferença depotencial num dos lados por isso:

VA =ε

4= 5 µV ⇐⇒ B0ℓ

2

4= 5 × 10−6 ⇐⇒ B0 =

4 × 5 × 10−6

10−4= 0, 2 T

c) Há duas maneiras de analisar o circuito do voltímetro VB . Num caso, consideremos o circuito constituído porVB − A − D − VB e no outro vamos considerar o circuito que passa em VB − A − B − C − D − VB.O primeiro circuito é mais difícil de analisar pois, para além da força electromotriz da espira, temos a forçaelectromotriz induzida no próprio circuito do voltímetro, devido à sua indutância (Lint):∮

~E · d~ℓ = −dφB

dt⇒ VDA + VB = −dφB

dtφB = Bℓ2 + LintIonde I, é a corrente que percorre o circuito (desconhecida).

A segunda maneira é bastante mais simples pois ficamos com um circuito sem indução. Nesse circuitoverificamos facilmente que a queda de tensão medida pelo voltímetro é a queda de tensão em três lados daespira, com um sentido contrário à do lado AD:

∮~E · d~ℓ = −dφB

dt⇒ VDC + VCB + VBA + VB =

0 ⇒ 3Vℓ + VB = 0Donde:VB = −3

4ε = −3 × 5 µV = −15 µV

F.Barão, L.F.Mendes Electromagnetismo e Óptica (MEEC-IST) 95

Page 102: Colecção de Problemas de Electromagnetismo e Ópticafcomp.ist.utl.pt/EO.web/Book.29_01_2014.pdf · Preâmbulo Esta colectânea de problemas resulta da experiência dos autores na

Capítulo 5

Exercício 5.5

a) Se a barra cai com velocidade constante (assume-se que existe uma força externa que permite verificar estacondição) o fluxo do campo magnético no circuito é dado por (assumindo uma normal com a mesma direcçãodo campo):

Φ =

~B · ~ndS =

BdS = B

dS = Bℓ(y0 + v0t)

Como a única diferença de potencial no circuito é a força electromotriz induzida (V = 0),

I =ε

R= − 1

R

dt= −Bℓv0

R

O sinal negativo indica-nos que, tendo-se escolhido uma normal de acordo com uma corrente de sentido horário(), a corrente no circuito tem um sentido anti-horário ().

b) Se não existir corrente no circuito, não existirá também qualquer força magnética sobre a barra pelo que estacairá em queda livre. Mas como existirá força electromotriz induzida devida ao aumento da área do circuito,a fonte de tensão terá que criar uma diferença de potencial que anule a tensão total. A posição da barra emfunção do tempo será,

y = y0 +1

2gt2

o fluxo através do circuito será (escolhendo novamente uma normal paralela ao campo magnético)

Φ = Bℓ

(

y0 +1

2gt2)

E a força electromotriz induzida será,

ε = − d

dt

[

Bℓ

(

y0 +1

2gt2)]

= −Bℓgt

Isto significa então que a fonte de tensão variável terá que variar linearmente no tempo, de acordo com aexpressão anterior mas com uma polaridade que criaria uma corrente com sentido horário ().

c) Neste caso a corrente criada pela fonte tem que originar uma força magnética na barra que contrarie a forçagravítica. A força criada pelo campo sobre um elemento de corrente da barra se esta for percorrida por umacorrente I será,

d~F = Id~ℓ × ~B

F.Barão, L.F.Mendes Electromagnetismo e Óptica (MEEC-IST) 96

Page 103: Colecção de Problemas de Electromagnetismo e Ópticafcomp.ist.utl.pt/EO.web/Book.29_01_2014.pdf · Preâmbulo Esta colectânea de problemas resulta da experiência dos autores na

Capítulo 5

É fácil verificar que para termos uma força vertical para cima que equilibre o peso necessitamos de umacorrente com sentido anti-horário e neste caso,

~F =

Id~ℓ × ~B =

∫ ℓ

o

IBdℓ(−~uy) = −IBℓ~uy = −mg~uy

I =V

R=

mg

Bℓ⇐⇒ V = R

mg

Bℓ

Ou seja, a fonte terá um valor constante e criará uma corrente com sentido anti-horário ().

d) Movimento retardado até parar. A corrente na barra será a soma de duas correntes anti-horárias: a da alíneac), que contrariará sempre a força gravítica; uma corrente antihorária devido à indução, tal como na alínea a)que criará uma força adicional para cima. Esta força adicional fará a barra parar, uma vez que, ao contrárioda alínea a), não existe a força exterior que garante a velocidade constante da barra. Quando a barra pararesta segunda corrente desaparece, desaparecendo com ela a força exterior e a barra ficará suspensa.

Exercício 5.10

a) Os dois enrolamentos estão ligados por um núcleo de material ferromagnético que, como sabemos, conduz aslinhas de campo. Deste modo, e tendo o núcleo uma forma circular, as linhas de campo vão ser circunferências.Da observação da figura verifica-se que ambos os enrolamentos criam um campo com a mesma direcção: linhasde campo com sentido horário. Vamos então aplicar a lei de Ampère a uma linha de campo genérica de raio r,circulando em sentido horário e verificando que todas as espiras dos dois enrolamentos atravessam a superfícieque se apoia em qualquer linha de campo:

~H · d~ℓ =∑

I ⇐⇒ B2πr = µ(N1I1 + N2I2) =⇒ ~B = µN1I1 + N2I2

2πr(−~uθ)

b)

M =Φ12

I1

Φ12 =

S2

~B · ~ndS =

S2

µN1I1

2πr(−~uθ) (−~uθ)dS =

(∫

S2 (espira)

µN1I1

2πrdS

)

N2

= µN1N2I1

∫ b

a

dr

r

∫ h

0

dz = µN1N2I1

2πhln

(b

a

)

c)

ε = −dΦ

dt= −Nespiras

dΦespira

dt

ε1 = −N1

dt

ε2 = −N2

dtε1

ε2=

N1

N2

Em que Φ é o fluxo que atravessa a secção do núcleo, ou seja, o fluxo que atravessa cada espira dos doisenrolamentos.

Exercício 5.20

F.Barão, L.F.Mendes Electromagnetismo e Óptica (MEEC-IST) 97

Page 104: Colecção de Problemas de Electromagnetismo e Ópticafcomp.ist.utl.pt/EO.web/Book.29_01_2014.pdf · Preâmbulo Esta colectânea de problemas resulta da experiência dos autores na

Capítulo 5

a) Na aproximação da bobina infinita, as linhas de campo magnético são paralelas ao eixo da bobina e o campoé nulo no seu exterior. Nesse caso aplica-se a lei de Ampère com um caminho fechado que coincide com umalinha de campo ao longo de um troço L, que tem dois lado perpendiculares às linhas de campo que saem parafora da bobina e que se fecha do lado de fora da bobina. A corrente que atravessa a superfície que se apoiano caminho assim definido depende do número de espiras que atravessa essa superfície (nL) e da correnteque cada espira transporta, I:

~B · d~ℓ = µ0

I ⇐⇒ BL = µ0nLI =⇒ B = µ0nI

~B = µ0nI~ux

b) Utlizemos a lei de Ampère generalizada. Se podermos considerar os dois lados da bobina como uma bobinainfinita, o campo tem a expressão da alínea anterior com a diferença de que no material ferromagnético emvez de µ0 tem-se µ.

L =Φ

I, Φ = Φ1 + Φ2

Φ1 =

~B1 · ~ndS =

B1dS =

µnIds = µnIπR2 × (n de espiras) = µn2IπR2x

Φ2 =

~B2 · ~ndS =

B2dS =

µ0nIds = µ0nIπR2 × (n de espiras) = µ0n

2IπR2(ℓ − x)

L = n2πR2 [µ0(ℓ− x) + µx]

c) A posição do núcleo de ferro só pode variar segundo xx:

Fx = +dWm

dx

∣∣∣∣I=cte

=1

2n2πR2I2(µ − µ0)

Nota: quando µ → µ0, Fx → 0

F.Barão, L.F.Mendes Electromagnetismo e Óptica (MEEC-IST) 98

Page 105: Colecção de Problemas de Electromagnetismo e Ópticafcomp.ist.utl.pt/EO.web/Book.29_01_2014.pdf · Preâmbulo Esta colectânea de problemas resulta da experiência dos autores na

Capítulo 5

5.3 Soluções

5.1 I = B0R

Aαe−αt

5.3 a) Φ = Bℓ2cos(ωt), com ω = 200π

b) I = 1, 3sen(ωt)

c) E = 3, 8 κJ

5.4 a) I = Bdv0R

, sentido .

b) P =(Bdv0)

2

R

c) ~F = B2d2v0R

(−~ux)

d) P =(Bdv0)

2

R

5.6 a) ε = 3, 6 mV

b) I = 171 A

c) I = 72 A; não.

5.7 a) B = µ2π

Ir

b) Φ = N µ2π

I d ln(R2R1

)

c) ε = 9, 6sen(100πt) V

5.8 L′

= µ0π

ln(

d−RR

)

= 0, 9 µH.m−1

5.9 a) L = µ0n2ℓπR2

b) ε = µ0nωI0sen(ωt)πr2

c) E = ε2πr

5.11 a) M = µnπ(D2)2

b) didt

+ RLi = M

LI0ωsen(ωt)

5.12 a) M = µ0N1N2

ℓπR2

1

b) i2(t) = MaI0R−aL

(

e−at − e−R/L)

5.13 a) um = µ08

(

Irπa2

)2

b) Wm = µ016π

I2

c) L′

= µ08π

5.14 a) um = µ08

(

Iπr

)2

b) Wm = µ04π

I2ln( ba)

c) L′

= µ02π

ln( ba)

5.15 a) um = 12µ0n2I2

b) Wm = 12µ0n2I2πR2ℓ

c) L = µ0n2ℓπR2

d) explosão.

e) P = 12µ0n2I2

5.16 a) L = µ0N2

ℓπa2

b) L = µ0Nℓπb2cos(θ)

c) Wm = 12µ0

N2

ℓπa2I2

1+ µ0Nℓπb2cos(θ)I1I2

d) N = −µ0Nℓπb2sen(θ)I1I2;

o sinal negativo significa que q diminui, ou seja,a espira roda em sentido horário; Os pontos deequilíbrio são θ = 0 e θ = π mas apenas θ = 0é estável.

5.17 a) M = µ0

2z3 πa2b2

b) Wm = 12LaI2

a + 12LbI

2b + MIaIb

c) Fz = 32IaIb

µ0

z4 π(ab)2

atractiva se as correntes tiverem o mesmo sentidoe repulsiva se as correntes tiverem sentidos dife-rentes.

5.18 a) B = µn(I1 + I2) (r < R1)B = µ0nI2 (R1 < r < R2)B = 0 (r > R2)

b) L2 = µπR21n

2ℓ + µ0π(R22 − R2

1)n2ℓ

M = L1 = µπR21n

2ℓ

c) Wm = 12µπR2

1n2ℓ(I1 + I2)2+

12µ0π(R2

2 − R21)n

2ℓI22

d) P1 = 12

[

µn2(I1 + I2)2 − µ0n2I22

]

P2 = 12µ0n2I2

2

5.19 a) Wm = 3, 2 × 104 J

b) Fx = −3, 2 × 104 N3,2 toneladas para cima.

F.Barão, L.F.Mendes Electromagnetismo e Óptica (MEEC-IST) 99

Page 106: Colecção de Problemas de Electromagnetismo e Ópticafcomp.ist.utl.pt/EO.web/Book.29_01_2014.pdf · Preâmbulo Esta colectânea de problemas resulta da experiência dos autores na

Capítulo 5

F.Barão, L.F.Mendes Electromagnetismo e Óptica (MEEC-IST) 100

Page 107: Colecção de Problemas de Electromagnetismo e Ópticafcomp.ist.utl.pt/EO.web/Book.29_01_2014.pdf · Preâmbulo Esta colectânea de problemas resulta da experiência dos autores na

Capítulo 6

Capítulo 6

Circuitos eléctricos

F.Barão, L.F.Mendes Electromagnetismo e Óptica (MEEC-IST) 101

Page 108: Colecção de Problemas de Electromagnetismo e Ópticafcomp.ist.utl.pt/EO.web/Book.29_01_2014.pdf · Preâmbulo Esta colectânea de problemas resulta da experiência dos autores na

Capítulo 6

6.1 Exercícios Propostos

Exercício 6.1 : Considere um circuito RC com umafonte de tensão constante Vf . No instante inicial ocondensador encontra-se descarregado.

a) Escreva as equações diferenciais que descrevema carga no condensador, q(t), a corrente no cir-cuito, i(t), a tensão no condensador, vC(t), e atensão na resistência, vR(t).

b) Encontre as equações que descrevem q(t), i(t),vC(t) e vR(t).Sugestão: experimente soluções do tipo a+beαt.

c) Determine a expressão da energia fornecida pelafonte ao circuito durante a carga do condensador.

d) Determine as expressões da energia dissipada porefeito de Joule na resistência durante a carga docondensador e da energia armazenada no conden-sador após este se encontrar carregado.Nota: o resultado não depende do valor da re-sistência utilizada e que a energia fornecida pelafonte se reparte em partes iguais pelo condensa-dor e pela resistência!

e) Sabendo que a tensão da fonte é Vf = 9 V, quea capacidade do condensador é C = 1 µF e quea resistência é R = 100 Ω, esboce os gráficosde q(t), i(t), vC(t) e cR(t).

f) Quanto tempo demora o condensador a atingiruma fracção q/q0 = 0, 9 da sua carga final?

Exercício 6.2 : Considere um circuito RC em que ocondensador se descarrega através da resistência. Noinstante inicial o condensador tem uma carga q(0) =q0.

a) Escreva as equações diferenciais que descrevema carga no condensador, q(t), a corrente no cir-cuito, i(t), a tensão no condensador, vC(t) e atensão na resistência, vR(t)

b) Encontre as equações que descrevem q(t), i(t),vC(t) e vR(t). Sugestão: experimente soluçõesdo tipo a+ beαt.

c) Determine a expressão da energia dissipada porefeito de Joule na resistência durante a descargado condensador.

d) Sabendo que a capacidade do condensador é C =10 µF e que a resistência é R = 1 KΩ, esboceos gráficos de q(t), i(t), vC(t) e vR(t).

Exercício 6.3 : Considere um circuito RL com umafonte de tensão constante Vf . No instante inicial acorrente no circuito é i(0) = 0.

a) Escreva as equações diferenciais que descrevem acorrente no circuito, i(t), a tensão na indutância,vL(t) e a tensão na resistência, vR(t).

b) Encontre as equações que descrevem i(t), vL(t)e vR(t). Sugestão: experimente soluções do tipoa + beαt.

c) Determine a expressão da energia fornecida pelafonte ao circuito durante o período de estabele-cimento da corrente no circuito (“carga” da indu-tância).

c) Determine as expressões da energia dissipada porefeito de Joule na resistência durante o períodode estabelecimento da corrente no circuito e daenergia armazenada na indutância.

e) Sabendo que a tensão da fonte é Vf = 9 V, quea indutância é L = 1 mH e que a resistência éR = 100 Ω, esboce os gráficos de i(t), vL(t)e vR(t).

f) Quanto tempo demora o circuito a atingir umafracção i/i0 = 0, 9 da sua corrente final?

Exercício 6.4 : Considere um circuito RL onde umafonte estabelece uma corrente i0 sendo de seguida reti-rada a fonte.

a) Escreva as equações diferenciais que descrevem acorrente no circuito, i(t), a tensão na indutância,vL(t) e a tensão na resistência, vR(t).

b) Encontre as equações que descrevem i(t), vL(t)e vR(t). Sugestão: experimente soluções do tipoa + beαt.

c) Determine a expressão da energia dissipada porefeito de Joule na resistência até a corrente cessarno circuito.

d) Sabendo que a indutância é L = 10 mH e quea resistência é R = 100 Ω, esboce os gráficosde i(t), vL(t) e vR(t)

Exercício 6.5 : Considere um circuito LC em que noinstante inicial o condensador está carregado com umacarga q(0) = q0 e a corrente no circuito é i(0) = 0.

a) Escreva as equações diferenciais que descrevema carga do condensador q(t), a corrente no cir-cuito, i(t), a tensão na indutância, vL(t) e atensão no condensador, vC(t).

b) Encontre as equações que descrevem q(t), i(t),vL(t) e vC(t). Sugestão: experimente soluçõesdo tipo αsen(ωt + β).

F.Barão, L.F.Mendes Electromagnetismo e Óptica (MEEC-IST) 102

Page 109: Colecção de Problemas de Electromagnetismo e Ópticafcomp.ist.utl.pt/EO.web/Book.29_01_2014.pdf · Preâmbulo Esta colectânea de problemas resulta da experiência dos autores na

Capítulo 6

c) Se o valor da capacidade for C = 10 µF e o valorda indutância fôr L = 1 mH, qual a frequênciade oscilação do circuito?

d) Determine as expressões da energia armazenadano condensador, UC , da energia armazenada naindutância, UL, e da energia total do sistema,U = UC + UL. Esboce os seus gráficos emfunção do tempo.

Exercício 6.6 : Um circuito RLC em que R = 1 kΩ,L = 1 mH e C = 1 µF, encontra-se ligado a umafonte de tensão sinusoidal V (t) = V0sen(ωt), sendoV0 = 5 V.

a) Escreva a equação diferencial que descreve a cor-rente no circuito, i(t).

Sabendo que a solução da equação diferencial encon-trada na alínea a) para o regime forçado (após ter desa-parecido o regime livre) é

I(t) = I0sen(ωt+ φ) =

V0√

R2 + (ωL− 1/ωC)2sen(ωt + φ)

com, tg(φ) = 1ωRC

− ωLR

,

b) Determine as expressões que descrevem vR(t),vL(t) e vC(t).

c) Calcule a frequência angular da fonte, ω, e a cor-respondente frequência, f , que tornam máximaamplitude da corrente no circuito, e calcule o seuvalor. Escreva neste caso a expressão da correnteno circuito.

d) Nas condições da alínea anterior, determine asexpressões da energia armazenada no condensa-dor, UC e da energia armazenada na indutância,UL. Verifique que a energia total armazenada nosistema é constante. Esboce os gráficos de UC ,UL e da energia total.

Exercício 6.7 : Um circuito que permite sintonizar umreceptor de rádio é constituído por uma indutância de10 mH e por uma capacidade variável. Qual o valorda capacidade quando o circuito está sintonizado parareceber uma estação que emite em 1, 4 MHz?

Exercício 6.8 : Considere o circuito da figura em queC1 = C2 = 1 µF e R = 100 Ω. Inicialmente o inter-ruptor encontra-se aberto, o condensador C1 tem umatensão de 10 V e o condensador C2 está descarregado.

a) Calcule a carga e a energia inicial do condensadorC1.

b) Após se ter fechado o interruptor e se ter atingidoo regime estacionário:

b1) Calcule a tensão de cada condensador.

b2) Calcule a energia dissipada na resistência.

c) Considere agora o mesmo circuito mas sem resis-tência (utilizaram-se materiais supercondutores,não existem resistências de contacto, etc). Tendoem conta os resultados da alínea b) e o facto denão haver energia dissipada (não existe resistên-cia), discuta qualitativamente o comportamentodo circuito quando se fecha o interruptor.

Exercício 6.9 : Um transformador (dois enrolamen-tos com um núcleo de ferro comum, como mostradona figura) encontra-se ligado à rede eléctrica mas noseu circuito secundário não está ligado qualquer equi-pamento. O circuito primário do transformador consti-tui um circuito RL (resistência interna R e indutânciaL do enrolamento) ligado a uma fonte de tensão al-terna (v(t) = V0sen(ωt)) com V0 =

√2× 230 V e

frequência f = 50 Hz.

F.Barão, L.F.Mendes Electromagnetismo e Óptica (MEEC-IST) 103

Page 110: Colecção de Problemas de Electromagnetismo e Ópticafcomp.ist.utl.pt/EO.web/Book.29_01_2014.pdf · Preâmbulo Esta colectânea de problemas resulta da experiência dos autores na

Capítulo 6

a) Escreva a equação diferencial que descreve a cor-rente no circuito primário do transformador.

b) Verifique que a corrente no circuito primário, noregime forçado, é descrita pela equação

i(t) = i0sen(ωt − φ)

com, i0 = V0√ω2L2+R2

e φ = atan(ωLR

).

c) Sabendo que o núcleo de ferro tem uma secçãoS = 1 cm2 e um raio médio a = 4 cm, queo ferro tem µ = 105µ0 e que circuito primáriotem N = 200 espiras, calcule um valor aproxi-mado para o coeficiente de autoindução do enro-lamento do circuito primário, L, baseado no raiomédio do núcleo.

d) Sabendo que a resistência interna do enrolamentodo circuito primário é R = 0, 1 Ω, determine apotência média dissipada na resistência. Com-pare com a potência que seria dissipada na resis-tência no caso de não haver indução magnéticano enrolamento do primário.

e) Existe força electromotriz aos terminais do enro-lamento do circuito secundário?

Exercício 6.10 : Considere o circuito da figura em quea fonte tem uma tensão de 10 V, a resistência R1 = 10kΩ, a resistência R2 = 1 Ω e a indutância L = 10mH. O interruptor encontra-se fechado há muitotempo.

a) Determine a força electromotriz induzida na bo-bine e as correntes eléctricas I1 e I2. Justifiquea sua resposta.

b) Num dado instante abre-se o interruptor.

b1) Escreva a equação diferencial que descrevea tensão na bobine. Marque no circuito asdirecções da tensão na bobine e da forçaelectromotriz induzida na bobine.

b2) Qual será a energia dissipada por efeito deJoule nas resistências até a corrente quepercorre o circuito se anular?

F.Barão, L.F.Mendes Electromagnetismo e Óptica (MEEC-IST) 104

Page 111: Colecção de Problemas de Electromagnetismo e Ópticafcomp.ist.utl.pt/EO.web/Book.29_01_2014.pdf · Preâmbulo Esta colectânea de problemas resulta da experiência dos autores na

Capítulo 6

6.2 Resoluções

Exercício 6.9

a) O circuito equivalente do primário do transformador é um circuito RL com fonte pelo que a lei das malhasnos dá:

V = VR + VL = Ri + Ldi

dt⇐⇒ di

dt+

R

Li =

V0

Lsen(ωt)

b)

di

dt= i0ωcos(ωt− φ)

i0ωcos(ωt− φ) +R

Li0sen(ωt− φ) =

V0

Lsen(ωt − φ)

Desenvolvendo,

sen(ωt) = sen [(ωt − φ) + φ)] = sen(ωt − φ)cos(φ) + cos(ωt− φ)sen(φ)

e igualando os termos em sen(ωt − φ) e cos(ωt− φ),

R

Li0sen(ωt − φ) =

V0

Lsen(ωt − φ)cos(φ) ⇐⇒ R

Li0 =

V0

Lcos(φ) (6.1)

i0ωcos(ωt− φ) =V0

Lcos(ωt− φ)sen(φ) ⇐⇒ i0ω =

V0

Lsen(φ) (6.2)

Dividindo (6.2) por (6.1),ωL

R= tan(φ) =⇒

(ωL

R

)

Tendo em conta que,

cos(φ) =R

√R2 + ω2L2

e substituindo por exemplo em (6.1), vem:

R

Li0 =

V0

L

R√R2 + ω2L2

=⇒ i0 =V0√

R2 + ω2L2

c) Utilizando a lei de Ampère com um caminho fechado que é a linha de campo de raio médio criada peloenrolamento do circuito primário temos,

~H · d~ℓ =∑

I = NI =⇒ H =NI

2πa

sendo I a corrente estacionária no enrolamento utilizada para este cálculo. O fluxo do campo magnético seráo fluxo através de uma espira vezes o número de espiras do enrolamento,

Φ =

~B · ~ndS = BNS = µN2I

2πaS

E o coeficiente de autoindução é,

L =Φ

I= µ

N2I

2πaS = 2

[H]

F.Barão, L.F.Mendes Electromagnetismo e Óptica (MEEC-IST) 105

Page 112: Colecção de Problemas de Electromagnetismo e Ópticafcomp.ist.utl.pt/EO.web/Book.29_01_2014.pdf · Preâmbulo Esta colectânea de problemas resulta da experiência dos autores na

Capítulo 6

d)

P = Ri2 = R

(V0√

R2 + ω2L2sen(ωt − φ)

)2

Como o valor médio da função sen2(ωt− φ) no tempo é 1/2,

P =R

2

V 20

R2 + ω2L2= 13

mW Se não houvesse indução o circuito apresentaria uma resistência de 0, 1 Ω ligada à fonte e,

P =V 20

Rsen2(ωt)

P =1

2

V 20

R= 530

KW e o circuito não resistiria. Nota: Para além da dissipação na resistência, mesmo com o secundáriodesligado, existem ainda perdas devidas às correntes de EddyFocault (ver, por exemplo, Popovic pág. 247) edevidas à histerese (ver, por exemplo, Popovic pág. 282

e) Sim. Apesar de não haver corrente e, portanto, não existir no núcleo de ferro um campo magnético criadopelo enrolamento secundário, as linhas do campo magnético criado pelo primário atravessam o circuito dosecundário induzindolhe uma força electromotriz, ou seja, uma tensão aos seus termi nais.

F.Barão, L.F.Mendes Electromagnetismo e Óptica (MEEC-IST) 106

Page 113: Colecção de Problemas de Electromagnetismo e Ópticafcomp.ist.utl.pt/EO.web/Book.29_01_2014.pdf · Preâmbulo Esta colectânea de problemas resulta da experiência dos autores na

Capítulo 6

6.3 Soluções

6.1 a) dqdt

+ qRC

=Vf

Rdidt

+ iRC

= 0dvCdt

+ vCRC

=Vf

RCdvRdt

+ vRRC

= 0

b) q(t) = q0(

1 − e−t/RC)

i(t) = i0e−t/RC

vC(t) = Vf

(

1 − e−t/RC)

vR(t) = Vfe−t/RC

c) Uf = CV 2f

d) UR = 12CV 2

f

UC = 12CV 2

f

e) t = 230 µs

6.2 a) dqdt

+ qRC

= 0didt

+ iRC

= 0dvCdt

+ vCRC

= 0dvRdt

+ vRRC

= 0

b) q(t) = q0e−t/RC

i(t) = i0e−t/RC

vC(t) = VC0e−t/RC

vR(t) = VC0e−t/RC

d) UR = 12CV 2

C0

6.3 a) didt

+ RLi =

Vf

LdvLdt

+ RLvL = 0

dvRdt

+ RLvR = R

LVf

b) i(t) = i0(

1 − e−R/L t)

vL(t) = Vfe−R/L t

vR(t) = Vf

(

1 − e−R/L t)

c) Uf =∫∞0

Vf i0dt − Li20

d) UL = 12Li20

UR =∫∞0 Vf i0dt − 3

2Li20

f) t = 23 µs

6.4 a) didt

+ RLi = 0

dVLdt

+ RLVL = 0

dVRdt

+ RLVR = 0

b) i(t) = i0e−R/L t

VL(t) = Ri0e−R/L t

VR(t) = Ri0e−R/L t

d) UR = 12Li20

6.5 a) d2qdt2

+ qLC

= 0

d2idt2

+ iLC

= 0

d2vCdt2

+ vCLC

= 0

d2vLdt2

+ vLLC

= 0

b) q(t) = q0cos(ωt)i(t) = i0sen(ωt)vC(t) = VC0cos(ωt)vL(t) = VC0cos(ωt)

c) f = 1, 6 KHz

d) UC = 12

q20C

cos2(ωt)

UL = 12

q20C

sen2(ωt)

U = 12

q20C

= 12CV 2

C0 = UC = 12Li20

6.6 a) d2idt2

+ RL

didt

+ iLC

= V0L

ωcos(ωt)

b) vR(t) = RI0sen(ωt + φ)vL(t) = ωLI0cos(ωt + φ)

vC(t) = − I0ωC

cos(ωt + φ)

c) Imax0 quando ω2LC = 1

ω = 1√LC

= 3, 2 × 105 rad.s−1

f = 5 KHzImax0 = V0

R= 5 mA

i(t) = V0R

sen(ωt)

d) UC(t) = 12LI2

0cos2(ωt)

UL(t) = 12LI2

0sen2(ωt)

U = 12LI2

0

6.7 C = 1, 29 µF

6.8 a) Q = 10 µC; UC = 50 µJ

b.1) VC1 = VC2 = 5 V

b.2) UR = 25 µJ

c) O circuito ficará a oscilar com toda a energia emC1 ou em C2.

6.10 a) ε = 0; I1 = 1 mA; I2 = 10 A

b) dvLdt

+ R1+R2L

vL = 0;vL tem a direcção de cima para baixo na figuramas a força electromotriz tem a direcção de baixopara cima.

c) UR = 0, 5 J

F.Barão, L.F.Mendes Electromagnetismo e Óptica (MEEC-IST) 107

Page 114: Colecção de Problemas de Electromagnetismo e Ópticafcomp.ist.utl.pt/EO.web/Book.29_01_2014.pdf · Preâmbulo Esta colectânea de problemas resulta da experiência dos autores na

Capítulo 6

F.Barão, L.F.Mendes Electromagnetismo e Óptica (MEEC-IST) 108

Page 115: Colecção de Problemas de Electromagnetismo e Ópticafcomp.ist.utl.pt/EO.web/Book.29_01_2014.pdf · Preâmbulo Esta colectânea de problemas resulta da experiência dos autores na

Capítulo 7

Capítulo 7

Equações de Maxwell e ondaselectromagnéticas

F.Barão, L.F.Mendes Electromagnetismo e Óptica (MEEC-IST) 109

Page 116: Colecção de Problemas de Electromagnetismo e Ópticafcomp.ist.utl.pt/EO.web/Book.29_01_2014.pdf · Preâmbulo Esta colectânea de problemas resulta da experiência dos autores na

Capítulo 7

7.1 Exercícios Propostos

Exercício 7.1 : Um condensador plano de placas cir-culares de raio a, em que as armaduras estão separadaspor ar de uma distância d << a possui uma carga ini-cial Q0 e descarrega-se através de uma resistência R,de acordo com a expressão:

Q(t) = Q0e− t

τ

Considere que é válida a aproximação do condensadorde placas infinitas em todo o condensador.

a) Determine o campo eléctrico (EP ) no ponto P.

b) Determine o campo magnético (BP )no ponto P.

c) Determine o vector de Poynting (~S) no ponto Pe o fluxo do vector de Poynting que atravessa asuperfície lateral do condensador.

d) Como se relaciona aquele fluxo do vector de Poyn-ting com a potência fornecida pelo condensadorà resistência?

Exercício 7.2 : Um condutor de comprimento ℓ, secçãocircular de raio a e condutividade σc está ligado a umafonte de tensão V.

a) Determine o vector de Poynting (~S), junto à su-perfície do condutor, no seu exterior.

b) Determine o fluxo do vector de Poynting atra-vés da superfície do condutor e compare-o coma potência dissipada no condutor por efeito deJoule.

Exercício 7.3 : Um cabo coaxial com condutores deraios a eb, liga uma fonte de tensão V a uma resistênciaR.

a) Determine os campos eléctrico e magnético naregião entre os condutores.

b) Determine a magnitude e direcção do vector dePoynting.

c) Determine o fluxo do vector de Poynting atravésda secção recta do cabo.

d) Calcule a potência dissipada por efeito de Joulena resistência.

Exercício 7.4 : Uma onda plana monocromática defrequência f = 50 MHz viaja no vácuo na direcção doszz, estando o campo magnético ~B polarizado segundoa direcção xx com uma amplitude B0.

a) Qual o seu comprimento de onda?

b) Qual a direcção de polarização do campo eléc-trico?

c) Admita que usa uma espira condutora para detec-tar o campo magnético da onda. Em que planodeve ser colocada a espira para que a eficiênciade detecção seja máxima?

d) Se a espira, de diâmetro muito menor que o com-primento de onda, tiver uma área A e uma resis-tência R, qual a amplitude da corrente induzida?

Exercício 7.5 : Uma onda electromagnética plana, mo-nocromática e sinusoisal propaga-se no vácuo segundoo eixo dos zz e tem um comprimento de onda λ = 500nm. O seu campo eléctrico encontra-se polarizado se-gundo o eixo xx. A amplitude do campo eléctrico é 1µV.m−1.

a) Escreva a expressão do campo eléctrico segundoas coordenadas x, y e z.

b) Calcule o campo magnético.

c) Calcule o vector de Poynting.

d) Colocaram-se 3 espiras quadradas de lado λ/4nos planos xy, xz e yz, como indica a figura.Calcule a força electromotriz induzida devido aocampo eléctrico nas três espiras no instante emque o campo é máximo em z = z1.

λ/4

x

y

z

Z1

y

z

λ/4

x

Z1

x

y

z

λ/4Z1

Exercício 7.6 : Uma onda electromagnética plana defrequência desconhecida e polarizada linearmente, propaga-se no sentido positivo do eixo xx. Para determinar a

F.Barão, L.F.Mendes Electromagnetismo e Óptica (MEEC-IST) 110

Page 117: Colecção de Problemas de Electromagnetismo e Ópticafcomp.ist.utl.pt/EO.web/Book.29_01_2014.pdf · Preâmbulo Esta colectânea de problemas resulta da experiência dos autores na

Capítulo 7

sua frequência realizou-se a montagem experimental in-dicada na figura. Por tentativa e erro, concluiu-se quea força electromotriz induzida nas espiras A, B e C eramáxima quando estas estavam paralelas ao plano xy. Fa-zendo variar a distância entre as espiras A e B, verificou-se que a força electromotriz induzida no circuito a quepertencem era nula quando a distância era 3, 0 m. Aomesmo tempo, mediu-se na espira C, cuja coordenadaem x é idêntica à da espira A, uma força electromotrizmáxima, que oscilava entre 31, 4 nV e −31, 4 nV.

a) Calcule a frequência da onda electromagnética.

b) Sabendo que a área da espira C é 0, 01 m2 e ad-mitindo que as espiras são suficientemente estrei-tas para se poder considerar o campo uniformeno seu interior, obtenha a expressão do campomagnético, ~B.Sugestão: comece por calcular a força electro-motriz induzida no circuito da espira C.

c) Obtenha a expressão do campo eléctrico (~E).

d) Calcule o valor médio da potência transportadapela onda através da espira D, que tem a área de0, 05 m2 e é paralela ao plano yz.

Exercício 7.7 : Se a velocidade da luz fosse infinita eo campo eléctrico fosse caracterizado pela mesma cons-tante, (1/4πε0), qual seria o valor do campo magné-tico? Existiriam ímanes? E disquetes ou compact flashdrives?

Exercício 7.8 : Uma onda plana monocromática defrequência f = 1 GHz propaga-se no vácuo e é descritapelo seguinte campo eléctrico:

~E = Ex~ux + Ey~uy

Ex = E0 cos(ωt− κz)

Ey = E0sen(ωt − κz)

Determine, para esta onda:

a) o comprimento de onda e o período;

b) a direcção de propagação;

c) a polarização da onda;

d) o campo magnético;

e) a densidade de energia transportada;

f) o vector de Poynting.

Exercício 7.9 : O campo magnético de uma onda elec-tromagnética plana que se propaga num meio com per-meabilidade magnética µ0 é dada por:

Bx = 7, 5 × 10−9sen(7, 5 × 106 t− 3 × 10−2 y) [T ]

By = 0 [T ]

Bz = −7, 5 × 10−9sen(7, 5 × 106 t − 3 × 10−2 y) [T ]

a) Calcule a velocidade de propagação da onda.

b) Qual a permitividade e o índice de refracção domeio?

c) Qual a direcção de propagação da onda?

d) Descreva o estado de polarização da onda.

Exercício 7.10 : Uma onda electromagnética planapropaga-se num meio não condutor com permeabilidademagnética µ0. O seu campo eléctrico é dado por:

E = 0, 5 cos[6, 5 × 106 t − 3.1 × 10−2

(

0, 5 ×√3 y − 0, 5 z

)]

~ux [V.m−1]

a) Qual a direcção de propagação da onda?

b) Qual o índice de refracção do meio?

c) Determine o campo magnético da onda, ~B.

d) Qual a polarização da onda?

e) Determine o vector de Poynting e a intensidadeda onda.

Exercício 7.11 : Uma fonte de radiação electromagné-tica radia isotropicamente (todas as direcções são equi-valentes) com uma potência média de 1000 W.

a) Determine a intensidade de radiação à distânciade 10 metros.

b) Qual o valor médio da densidade de energia trans-portada pela onda?

c) Relacione os campos eléctrico e magnético exis-tentes a essa distância com a densidade de ener-gia transportada pela onda.

F.Barão, L.F.Mendes Electromagnetismo e Óptica (MEEC-IST) 111

Page 118: Colecção de Problemas de Electromagnetismo e Ópticafcomp.ist.utl.pt/EO.web/Book.29_01_2014.pdf · Preâmbulo Esta colectânea de problemas resulta da experiência dos autores na

Capítulo 7

Exercício 7.12 : Uma onda electromagnética plana emonocromática propaga-se no ar e incide com um ân-gulo de 60 numa placa de cristal de área A = 2m2, sendo a placa totalmente iluminada pela onda. Adensidade média da potência transportada pela onda éI = 10−4

√ε0

µ0W.m−2 e o seu campo eléctrico é

descrito por:

~E = Ex~ux + Ez~uz

Ex = E0cos(ω t− k y)

Ez = E0sen(ω t− k y)

a) Qual a polarização da onda?

b) Quais as equações que descrevem o seu campomagnético ~B?

c) Qual o valor de E0?

d) Sabendo que 50% da potência transportada pelaonda atravessa a placa, qual a energia que a atra-vessou ao fim de 1 hora?

Exercício 7.13 : Um feixe de ondas electromagnéticasde frequência f propaga-se no ar ao longo do eixo dosxx. O seu campo eléctrico tem uma amplitude E0 eencontra-se polarizado linearmente fazendo um ângulode 45 graus com o eixo dos yy. Este feixe é detectadopor um fotomultiplicador cuja eficiência de detecção éde 20%.

a) Escreva as equações que descrevem o campo eléc-trico do feixe.

b) Escreva as equações que descrevem o campo mag-nético do feixe.

c) Sabendo que a intensidade de radiação detectadaé de 200 mW.m−2, determine a intensidade deradiação emitida pela fonte e calcule a amplitudedo campo eléctrico, E0.

Exercício 7.14 : Uma onda electromagnética plana,monocromática e sinusoisal propaga-se no vácuo e pos-sui um campo eléctrico dado pela expressão:

~E =E0 cos (ω t + 2x − 2y + z)(a~ex + ~ey + ~ez√

a2 + 2

)

[V/m]

a) Determine o vector de ondas ~κ e o seu módulo.

b) Calcule o comprimento de onda, λ.

c) Calcule a frequência angular da onda, ω.

d) Determine a constante a existente na expressãodo campo eléctrico.

e) Determine o valor de E0, sabendo que a intensi-dade da onda é de I = 〈|~S|〉 = ε0 c

[W/m2

].

f) Determine o campo magnético ~B da onda (di-recção e amplitude).

Exercício 7.15 : Uma onda electromagnética plana in-cide numa interface ar-água que separa estes dois meiosdieléctricos de permitividades eléctricas relativas εarr =1 e εagua

r = 81. Determine:

a) os índices de refracção dos meios dieléctricos.

b) a velocidade de propagação nos meios dieléctri-cos.

c) o ângulo de Brewster (θB) e o ângulo de refrac-ção correspondente.

d) os coeficientes de reflexão e transmissão, paraθi = θB, para o caso em que a onda possuipolarização paralela ao plano de incidência.

e) os coeficientes de reflexão e transmissão, paraθi = θB, para o caso em que a onda possuipolarização perpendicular ao plano de incidência.

f) a transmitância (T) para ambos os casos (para-lela e perpendicular) de polarização da onda.

Eqs de Fresnel:

(paralelo ao plano de incidência)(

ER

EI

)

= n2cosθi−n1cosθt

n2cosθi+n1cosθt

(ET

EI

)

= 2n1cosθi

n2cosθi+n1cosθt

(perpendicular ao plano de incidência)(

ER

EI

)

⊥= n1cosθi−n2cosθt

n1cosθi+n2cosθt

(ET

EI

)

⊥= 2n1cosθi

n1cosθi+n2cosθt

F.Barão, L.F.Mendes Electromagnetismo e Óptica (MEEC-IST) 112

Page 119: Colecção de Problemas de Electromagnetismo e Ópticafcomp.ist.utl.pt/EO.web/Book.29_01_2014.pdf · Preâmbulo Esta colectânea de problemas resulta da experiência dos autores na

Capítulo 7

7.2 Resoluções

Exercício 7.1

a) Sendo a >> d pode-se considerar a aproximação do condensador de placas infinitas, ou seja, pode-seconsiderar o campo eléctrico uniforme dentro do condensador e até à sua superfície lateral. Podemos de seguidausar a lei de Gauss para calcular o campo dentro do condensador, recorrendo a uma superfície cilíndrica deraio a com tampas dentro e fora do condensador. A tampa de fora não terá fluxo pois E = 0 e as superfícieslaterais do cilindro também não terá fluxo pois ~E ⊥ ~n. Ficamos então com,

~E · ~ndS =Qint

ε0⇐⇒ ε0Eπa2 = Q(t) =⇒ E =

Q(t)

ε0πa2

Considerando como positiva a placa da esquerda e a direcção ~uz como da placa da esquerda para a placa dadireita:

~E =Q0e

− tτ

ε0πa2~uz

b) Dentro do condensador não há corrente eléctrica mas há corrente de deslocamento. Como as linhas de campomagnético são circunferência em trono do eixo zz (centrado nos eixos das placas do condensador), vamosaplicar a lei de Ampère escolhendo como caminho uma linha de campo que passe no ponto P. Para sentidode circulação vamos escolher o que nos dá a normal com a direcção de ~E:

~B · d~ℓ = µ0ε0d

dt

(∫

~E · ~ndS)

⇐⇒ B2πa = µ0ε0d

dt

(Eπa2

)

=⇒ B =µ0

2ε0a

d

dt

(Q(t)

ε0Eπa2

)

=µ0

2πa

dQ

dt

~B = − µ0

2πa

Q0e− t

τ

τ~uθ

c)

~S = ~E × ~H = ~E ×~B

µ0

= EB

µ0

(~ur)

sendo ~ur a direcção radial em relação ao eixo das placas do condensador.

~S =Q0e

− tτ

ε0πa2

µ0

2πaQ0e

−tτ

τ

µ0

(~ur) =

(

Q0e− t

τ

)2

2π2a3ε0τ~ur

O fluxo através da superfície que envolve o condensador só existe na sua superfície lateral, já que o vector dePoynting é radial. Deste modo, e notando que o valor de S é constante nessa superfície:

ΦS =

~S · ~ndS =

(

Q0e− t

τ

)2

2π2a3ε0τA =

(

Q0e− t

τ

)2

2π2a3ε0τ2πad =

(

Q0e− t

τ

)2

πa2ε0τd

d) O resultado da alínea anterior diz-nos que existe uma potência a sair do condensador (fluxo positivo). Istoacontece porque um condensador a descarregar fornece potência ao circuito a que está ligado.

A potência fornecida pelo condensador ao circuito, P = V I, depende da tensão aos terminais do condensadorV = Ed e da corrente eléctrica devido à diminuição da carga nas armaduras do condensador, I = −dQ

dt.

P = V I = −EddQ

dt= − Q0e

− tτ

ε0Eπa2d

d

dt

(

Q0e− t

τ

)

=

(

Q0e− t

τ

)2

πa2ε0τd = ΦS

Exercício 7.4

F.Barão, L.F.Mendes Electromagnetismo e Óptica (MEEC-IST) 113

Page 120: Colecção de Problemas de Electromagnetismo e Ópticafcomp.ist.utl.pt/EO.web/Book.29_01_2014.pdf · Preâmbulo Esta colectânea de problemas resulta da experiência dos autores na

Capítulo 7

a) λ = cf= 3×108

5×107 = 6 m

b) ~n = ~uE × ~uB ⇐⇒ ~uz = ~uE × ~ux =⇒ ~uE = −~uy

c) A detecção do campo magnético da onda faz-se através da corrente induzida na espira. Para que o fluxo docampo magnético seja máximo, o plano da espira deve ser perpendicular ao campo, ou seja, deve estar em yz.

d) Sendo a espira muito pequena em relação ao comprimento de onda (6 m) podemos considerar a amplitude docampo, em cada instante, constante no plano da espira. Assim, sendo o campo magnético, ~B = B0 cos(ωt−kz)~ux, vem:

ε = − d

dt

(∫

~B · ~ndS)

= − d

dt(BA) = −A

dB

dt= −AωB0 (−sen(ωt − kz))

Imax =εmax

R=

AωB0

R

Exercício 7.6

a) Do enunciado depreende-se que λ/2 = 3 m pois assim o campo está em oposição de fase nas espiras A e Be ε = 0. Donde:λ = 6 mv = c

λ= 5 × 107 Hz

b) Do enunciado sabe-se que na espira C a força electromotriz induzida é ε = 31, 4 × 10−9 sen(ωt + α).ε = −dΦ

dt

Φ = 31,4×10−9

ωcos(ωt+ α) = 31,4×10−9

2×31,4×5×107 cos(ωt+ α) = 10−16 cos(ωt + α)

Φ =∫

~B · ~ndS = BA, uma vez que esta força electromotriz é máxima e portanto o campo é perpendicularà superfície da espira.B = Φ

A= 10−14 cos(ωt+ α) [T]

Podemos então concluir que: ~B = B0 cos cos(ωt − ~k · ~r + δ)~uB, com:B0 = 10−14 T~k · ~r = kx, pois a onda progaga-se segundo xx.~uB = ~uz, pois, como já se referiu, a força electromotriz é máxima quando o campo é perpendicular à espira,que está no plano xy.

c) ~ux = ~uE × ~uB =⇒ ~uE = ~uy e E = cB, pelo que:~E = cB0 cos(ωt − ~k · ~r + δ)~uy = 3 × 10−6 cos(ωt− ~k · ~r + δ)~uy V.m−1

d)

I =⟨

~S · ~n⟩

= 〈S~ux · ~ux〉 = 〈EH〉 =

EB

µ0

=

⟨E2

cµ0

=9 × 10−12

3 × 108 × 4π × 10−7

cos2(ωt − ~k · ~r + δ)⟩

=9 × 10−12

2 × 360= 1, 25 × 10−14 W.m−2

< P > = 1, 25× 10−14 × 5 × 10−2 = 6, 25 × 10−16 W

F.Barão, L.F.Mendes Electromagnetismo e Óptica (MEEC-IST) 114

Page 121: Colecção de Problemas de Electromagnetismo e Ópticafcomp.ist.utl.pt/EO.web/Book.29_01_2014.pdf · Preâmbulo Esta colectânea de problemas resulta da experiência dos autores na

Capítulo 7

7.3 Soluções

7.2 a) ~S = −σV 2a2ℓ2

~ur

b) ΦS = V 2σπa2

ℓ= V 2

R

7.3 a) ~E = Vrln(b/a)

~ur ; ~B = µ02π

Ir~uθ

b) ~S = V I2πr2ln(b/a)

~uz

c) ΦS = V I

d) P = V I

7.5 a) ~E = 10−6 cos(ωt − kz)~ux V.m−1

ω = 12π × 1014 rad.s−1

k = 4π × 106 rad.m−1

b) ~B = 3, 3 × 10−15 cos(ωt − kz)~uy T

c) ~S = 2, 6 × 10−15 cos2(ωt − kz)~uz W.m−2

d) yz : ε = 0; xz : ε = 0, 125 pV; xy : ε = 0.

7.7 B = 0

7.8 a) λ = 0, 3 m; T = 10−9 s.

b) ~uk = ~uz

c) polarização circular direita ()

d) ~B = 1c(Ex~uy − Ey~ux)

e) u = ε0E20

f) ~S =√

ε0µ0

E20~uz

7.9 a) v = 0, 83c

b) ε = 1, 44ε0; n = 1, 2.

c) ~uy

d) polarização linear no plano xz, fazendo um ângulode −45 com o eixo xx.

7.10 a) ~uk =√

32

~uy − 12~uz

b) n = 1, 43

c) Bx = 0

By = −0,52v

cos(ωt − ~k · ~r) [T]

Bz = −0,5√

32v

cos(ωt − ~k · ~r) [T]

d) linear

e) ~S =√

εµ0

0, 52 cos2(ωt − ~k · ~r)~uk W/m2

I ≡< ~S >=√

εµ0

0, 52 12

W/m2

7.11 a) I = 104π

W.m−2

b) < uT >= 10−7

12πJ.m−3

c) < E2 >= <uT >ε

7.12 a) circular esquerda

b) ~B = 1c(Ez~ux − Ex~uz)

c) E0 = 10−2 V.m−1

d) Energia =180√

ε0µ0

mJ

F.Barão, L.F.Mendes Electromagnetismo e Óptica (MEEC-IST) 115

Page 122: Colecção de Problemas de Electromagnetismo e Ópticafcomp.ist.utl.pt/EO.web/Book.29_01_2014.pdf · Preâmbulo Esta colectânea de problemas resulta da experiência dos autores na

Capítulo 7

F.Barão, L.F.Mendes Electromagnetismo e Óptica (MEEC-IST) 116

Page 123: Colecção de Problemas de Electromagnetismo e Ópticafcomp.ist.utl.pt/EO.web/Book.29_01_2014.pdf · Preâmbulo Esta colectânea de problemas resulta da experiência dos autores na

Capítulo 8

Capítulo 8

Óptica

8.1 Introdução

F.Barão, L.F.Mendes Electromagnetismo e Óptica (MEEC-IST) 117

Page 124: Colecção de Problemas de Electromagnetismo e Ópticafcomp.ist.utl.pt/EO.web/Book.29_01_2014.pdf · Preâmbulo Esta colectânea de problemas resulta da experiência dos autores na

Capítulo 8

8.2 Exercícios Propostos

Exercício 8.1 : Uma onda electromagnética monocro-mática de frequência f = 1014 Hz propaga-se no ar eentra num material com um índice de refracção n = 2.

a) Qual a velocidade de propagação da onda no ma-terial?

b) Qual a frequência da onda no material?

c) Qual o comprimento de onda no material?

Exercício 8.2 : Um raio de luz monocromático incidecom um ângulo de 30 numa das faces de uma placade vidro com índice de refracção n = 1, 5 (para essecomprimento de onda).

a) Qual o ângulo da onda transmitida através dovidro? (faça um esquema)

b) O que se alterava se em vez de vidro a luz atraves-sasse uma placa diamante de índice de refracção2, 4?

Exercício 8.3 : Um raio de sol (luz branca) incide sobreuma janela de vidro de 4 mm com um ângulo de 45.Sabendo que o índice de refracção do vidro para a corvermelha é de 1, 5885 e para a cor azul é de 1, 5982,determine a separação espacial das duas cores após oraio atravessar o vidro.

Exercício 8.4 : Uma onda electromagnética plana mo-nocromática propaga-se dentro de um material caracte-rizado por εr = 1, 5 e µr = 1.

a) Verifique em que condições de incidência da ondana superfície de separação do material com o arnão existe onda propagada no ar.

b) O fenómeno descrito em a) (reflexão total) poder-se ia verificar se a onda incidisse na superfície deseparação dos meios mas propagando-se no ar?

Exercício 8.5 : Um pescador procura observar na água(ngua >> 1, 5) um peixe, sendo este visível caso a luznele reflectida atinja os olhos do pescador. Considereque a luz proveniente do ponto P de um peixe (imóvel)está linearmente polarizada, com o seu campo eléctricono plano xy (ver figura).

a) Determine a velocidade da luz na água e escrevauma expressão para o campo eléctrico associadoà componente da luz com λ = 500nm da ondaque se propaga na direcção do ponto P2. Consi-dere que a amplitude do campo eléctrico é E0.

b) Determine as direcções de propagação da luz trans-mitida para o ar dos raios luz que incidem nasuperfície da água nas posições P1 e P2.

c) Qual o ângulo de incidência máximo que a luzproveniente do peixe pode ter para que possa serobservada pelo pescador?

Exercício 8.6 : Uma onda electromagnética plana mo-nocromática e polarizada circularmente desloca-se no are incide segundo um dado ângulo de incidência θi so-bre a superfície plana de um dieléctrico (εr = 2, 7 eµr = 1). Determine o ângulo de incidência para o quala onda reflectida está polarizada linear mente.

Exercício 7 : Um avião de estudos oceanográficos pre-tende medir o comprimento de onda das ondas do marenviando um feixe de radar de frequência variável e ve-rificando em que condições se obtêm máximos de inter-ferência.

a) Qual o ângulo de incidência do feixe sobre a su-perfície da água para que o feixe reflectido volteao ponto de partida? (De outra forma, comopoderia o avião detectar o feixe reflectido?!)

b) Calcule, em função do comprimento de onda dasondas do mar, λmar, e do ângulo que o feixefaz nas condições da alínea a) com a vertical dolugar, θ, os comprimentos de onda da radiaçãodo feixe que produzem máximos de interferênciano receptor do avião.

F.Barão, L.F.Mendes Electromagnetismo e Óptica (MEEC-IST) 118

Page 125: Colecção de Problemas de Electromagnetismo e Ópticafcomp.ist.utl.pt/EO.web/Book.29_01_2014.pdf · Preâmbulo Esta colectânea de problemas resulta da experiência dos autores na

Capítulo 8

c) Designando por n o índice de refracção da águado mar, qual o comprimento de onda do feixeemitido pelo avião que é transmitido para a água?Qual o desvio angular desse feixe na passagem doar para a água?

Exercício 8.8 : Duas fendas são iluminadas por umalâmpada de sódio filtrada (λ = 589 nm). Num alvo a1 m de distância formam-se riscas que na zona centraldo alvo estão distanciadas de 1 cm. Qual a distânciaentre as fendas?

Exercício 8.9 : Uma rede de difracção com 6000 li-nhas por centímetro é iluminada por luz verde de com-primento de onda λ = 500 nm.

a) Quantas riscas poderão ser vistas num alvo?

b) Qual a posição angular dessas riscas?

Exercício 8.10 : Uma rede de difracção com 4000 linhaspor centímetro é iluminada com luz branca que contémcomprimentos de onda que vão desde o violeta, λ1 =400 nm, ao vermelho, λ2 = 700 nm.

a) Quais as separações angulares dos espectros de1a, 2a e 3a ordem? Faça um esquema.

b) Que conclui quanto à visibilidade desses espec-tros?

Exercício 8.11 : A luz do Sódio é composta por ra-diação que contém dois comprimentos de onda muitopróximos: λ1 = 589, 59 nm e λ2 = 589, 00 nm.

a) Que número mínimo de linhas de uma rede dedifracção deverá ser iluminado para se conseguirseparar as duas riscas de primeira ordem (critériode Rayleigh)?

b) Este número de riscas iluminadas separava tam-bém as riscas de 2a ordem?

c) Como variava o espectro de riscas da luz do Só-dio se se utilizasse uma rede de difracção como mesmo número de riscas iluminadas mas maispróximas entre si?

d) Como variava o espectro de riscas da luz do Sódiose se utilizasse uma rede de difracção com asriscas à mesma distância entre si mas com maisriscas iluminadas?

Exercício 8.12 : A luz emitida por uma lâmpada de altorendimento tem dois comprimentos de onda próximos λ1

e λ2. Ao passar por uma rede de difracção formam-seduas riscas de primeira ordem (m = 1) a 1 cm e a

1, 1 cm do centro de um alvo. Considerando a aproxi-mação dos pequenos ângulos senθ ∼ θ e cos θ ∼ 1),determine:

a) a razão entre os comprimentos de onda λ1 e λ2.

b) o número mínimo de fendas iluminadas para queas duas riscas pareçam separadas (critério de Ray-leigh)?

Exercício 13 : Numa sala à escuras foi possível ob-servar a figura de difracção, resultante da luz vermelhade um ponteiro de laser que entrava por uma frinchade 6, 3 µm de largura (invisível a olho nu), projectadanuma parede a 1 m de distância da frincha. A largurada mancha luminosa central na parede é 20 cm.

a) Qual a frequência da luz do laser?

b) Se em vez de passar pela frincha a luz passassepor uma dupla fenda com aberturas distanciadasde 6, 3 µm, a figura de interferência seria igualou diferente da observada na parede? Justifique.

c) Se, ao entrar na sala, alguém acendesse duas lâm-padas que, por absurdo, estivessem distanciadasde 6, 3 µm, haveria figura de interferência (zonasclaras e escuras na sala)? Justifique.

Exercício 8.14 : Uma fenda com 400 nm de largura éiluminada com luz monocromática de 400 nm de com-primento de onda. O padrão resultante da difracção éprojectado num alvo.

a) Qual a posição angular do primeiro mínimo dedifracção? Qual a largura angular do máximocentral?

b) Repita a alínea anterior para o caso de a aberturater 4000 nm.

Exercício 8.15 : Um cabelo humano é iluminado comuma luz monocromática de comprimento de onda λ =500 nm. Num alvo situado a 2 m obtém-se o primeiromínimo do padrão de interferência a 2 cm do seu centro.Qual a espessura do cabelo?

Exercício 8.16 : A difracção limita a nitidez das ima-gens formadas nos instrumentos ópticos e na retina hu-mana. Sabendo que o diâmetro médio da retina do olhodurante o dia é 2, 5 mm, determine:

a) a largura angular da imagem de um ponto queemita com a cor laranja (λ = 600 nm).

b) a separação mínima entre dois pontos situados a10 m do observador para que este possa obterduas imagens distintas (critério de Rayleigh).

F.Barão, L.F.Mendes Electromagnetismo e Óptica (MEEC-IST) 119

Page 126: Colecção de Problemas de Electromagnetismo e Ópticafcomp.ist.utl.pt/EO.web/Book.29_01_2014.pdf · Preâmbulo Esta colectânea de problemas resulta da experiência dos autores na

Capítulo 8

Exercício 8.17 : A distância entre planos atómicosadjacentes na Calcite (CaCO3 cristalizado) é d =3×1010 m. Se um feixe de raios X de comprimento deonda igual a 0, 3 × 1010 m incidir sobre o cristal, qualo ângulo mínimo de incidência em relação aos planos docristal para o qual existe interferência construtiva?

Exercício 8.18 : Uma mancha de óleo no chão da ruatem uma espessura d = 0, 5 µm. Qual o ângulo mí-nimo entre um transeunte e a luz solar para que esteveja uma mancha verde (λ = 500 nm)?

F.Barão, L.F.Mendes Electromagnetismo e Óptica (MEEC-IST) 120

Page 127: Colecção de Problemas de Electromagnetismo e Ópticafcomp.ist.utl.pt/EO.web/Book.29_01_2014.pdf · Preâmbulo Esta colectânea de problemas resulta da experiência dos autores na

Capítulo 8

8.3 Exercícios Resolvidos

Resolução do exercício 8.7

a) O feixe tem que incidir perpendicularmente à superfície da água para que volte para trás com a mesmadirecção, para o avião que o enviou.

b) Para existir interferência construtiva entre o feixe reflectido em duas ondas, a diferença de percurso tem que serum múltiplo inteiro de comprimentos de onda. Convém notar também que as condições de perpendicularidadeentre o feixe e a superfície da água se repetem a cada comprimento de onda do mar. Assim, a diferença depercurso é duas vezes a distância d marcada na figura, ou seja, 2λmarsenθ = mλ =⇒ λ = 2λmar

msenθ

c) λ′

= vf= c

nf= λ

n

O ângulo de transmissão é calculável a partir da lei de SnellDescartes. Como neste caso a incidência é normal,a transmissão também o é, ou seja, não há qualquer desvio angular.

Figura 136.1

Resolução do exercício 8.13

a) Se a largura da mancha central é 20 cm, a posição dos primeiros mínimos de difracção é 10 cm para cadalado do centro. A posição angular do primeiro mínimo é então arctg(0,1), que é um ângulo suficientementepequeno para se poder fazer a aproximação

λ = asenθmin = 6, 3 × 10−6 × 0, 1 = 630nm

λ = cf=⇒ f = 3×108

630×10−9 = 4, 76× 1014 Hz

b) A expressão da posição angular dos mínimos da figura de difracção é igual à da posição angular dos máximosda figura de interferência pelo que a figuras são diferentes: apesar da posição do máximo central ser igual, asua largura não o é e todos os mínimos da difracção vão coincidir com os máximos da interferência.

c) Não pois as duas lâmpadas não são coerentes: a sua fase varia aleatoriamente e independentemente uma daoutra.

F.Barão, L.F.Mendes Electromagnetismo e Óptica (MEEC-IST) 121

Page 128: Colecção de Problemas de Electromagnetismo e Ópticafcomp.ist.utl.pt/EO.web/Book.29_01_2014.pdf · Preâmbulo Esta colectânea de problemas resulta da experiência dos autores na

Capítulo 8

8.4 Soluções

8.1 a) 1, 5 × 108 m.s−1

b) 1014 Hz

c) λ = 1, 5 µm

8.2 a) 30

b) Saía com o mesmo ângulo mas mais perto doponto de incidência.

8.3 d = 14, 8 µm

8.4 a) θi = 54, 7

b) não

8.5 a) ~E = Ex~ux + Ey~uy

Ex =√

32

E0 cos(

8π × 1014t−4π × 106( 1

2x +

√3

2y)

)

Ey = −12E0 cos

(

8π × 1014t−4π × 106( 1

2x +

√3

2y)

)

b) ~nar1 = ~uy

~nar2 = 0, 75~ux + 0, 66~uy

c) θmax = 41, 8

8.6 58, 7

8.8 d = 58, 9 µm

8.9 a) 7 riscas.

b) θ = 0; θ = ±17; θ = ±37; θ = ±64;

8.10 a) m = 1; θ1 = 9; θ2 = 16;

m = 2; θ′

1 = 19; θ′

2 = 34;

m = 3; θ′

1 = 29; θ′

2 = 57;

b) Os espectros de 2ae 3a ordem estão sobrepostos.

8.11 a) 1000

b) Sim, pois bastam 500 riscas iluminadas para sepa-rar.

c) Obtinham-se riscas mais separadas.

d) Obtinham-se riscas mais finas.

8.12 a) λ2λ1

= 1, 1

b) N = 10

8.14 a) θ = 90. Todo o alvo está iluminado: ∆θ =180

b) θ = 5, 7; ∆θ = 1, 5

8.15 50 µm

8.16 a) θ = 49, 5

b) 2, 4 mm

8.17 θ = 2, 9

8.18 120

F.Barão, L.F.Mendes Electromagnetismo e Óptica (MEEC-IST) 122